NPTAE Pocket Prep Questions

Réussis tes devoirs et examens dès maintenant avec Quizwiz!

The physical therapist assistant assesses the pulse of a 50-year-old male patient and finds it to be 70 beats per minute (bpm), which is within a normal range. What is the normal range for heart rate for an adult? A. 60-100 bpm B. 70-140 bpm C. 70-170 bpm D. 40-60 bpm

A. 60-100 bpm The normal adult heart rate is 70 beats per minute, with a normal range between 60-100 bpm. In pediatric patients, the normal range is 70-170 bpm, with the new born average heart rate at 120 bpm. Tachycardia is considered anything above 100 bpm and can be exercise induced. Bradycardia is considered anything below 60 bpm.

In regards to proper body mechanics, the back should be held in what position to protect the spine when the physical therapist is assisting for a transfer? A. A neutral position with abdominals braced B. A kyphotic position with abdominals braced C. A lordotic position with abdominals braced D. The back should move from flexion to extension and perform the majority of the lift.

A. A neutral position with abdominals braced When assisting a transfer, the back should be held in a neutral position to maintain the spine's natural curves. The neutral position of the spine protects the back, and abdominal bracing provides additional support to the spine. Lifting should never be done with movement from the back as this can cause back injury.

Which is NOT a typical characteristice of a venous ulcer? A. Well-demarcated lesion B. Little to no pain C. Yellow fibrous covering with granulation D. Located over medial malleolus

A. Well-demarcated lesion Characteristics of a venous ulcer include location over the medial malleolus, typically partial thickness in depth, shaggy and irregular edges, a base with yellow fibrous covering with granulation, and little to no pain. An ulcer with well-demarcated lesions is typically of an arterial ulcer.

The physical therapist assistant is delivering joint mobilization intervention at a patient's shoulder. The therapist delivers 3 sets of 20 seconds of continuous large amplitude oscillation that pushes into tissue resistance. Which grade of joint mobilization is being performed? A. Grade 2 B. Grade 3 C. Grade 4 D. Grade 5

B. Grade 3 Grade 3 joint mobilization is being performed in this example. Grade 3 and 4 are intended for increasing capsular mobility. Grade 1 and 2 are intended for joint nutrition and pain reduction. The grades of joint mobilization are determined by dosage of oscillation technique. Grade 1: small amplitude oscillation at the beginning of range. Grade 2: large amplitude oscillation before the point of tissue resistance Grade 3: large amplitude oscillation that pushes into tissue resistance Grade 4: small amplitude oscillation near the end of passive range Grade 5: small amplitude high-velocity manipulation past the end of passive range

When using ultrasound modality, excessively high temperatures may produces a strong ache caused by overheating which type of tissue? A. Blood B. Periosteal tissue C. Ligaments D. Muscle fibers

B. Periosteal tissue The patient may experience periosteal pain as a result of excessively high temperatures over a directed surface area of treatment. The clinician should reduce the intensity of the ultrasound or increase the surface area of the treatment if the patient experiences periosteal pain

A physician has ordered the physical therapist to utilize ultrasound to drive medications through the skin into the deeper soft tissues. Which treatment modality does this describe? A. Diathermy B. Phonophoresis C. Massage application D. Iontophoresis

B. Phonophoresis Phonophoresis is the use of ultrasound to drive medications through the skin into the deeper tissues. Local pain relievers and anti-inflammatory drugs are often used with phonophoresis such as lidocaine, dexamethasone, and salicylates. Iontophoresis utilizes electric current to drive medication through the skin. Diathermy utilizes electromagnetic energy to create heat. Massage is mechanical manipulation of soft tissue by the hands.

Two physical therapist assistants are asked to perform the same functional outcome measurement tool, the Berg Balance Assessment, on the same patient. The results show a difference in scoring between the two therapists. However when the same therapist repeats the test, his and her test scores remain the same between their two trials. This is an example of a problem with: A. Patient performance B. Clinic setup for testing environment C. Interrater reliability D. Intrarater reliability

C. Interrater reliability Interrater reliability is the degree to which two or more testers are able to obtain the same score or rating. In this case, the two therapists assistants found different scores with the same patient. However, they obtain the same score in their second trial, showing good intrarater reliability. Intrarater reliability is the degree to which the same tester will obtain the same score in subsequent testing.

A patient suffered a peripheral nerve injury that results in foot drop of the left lower extremity during gait. The physical therapist has included functional electrical stimulation fr ortostatic substitution. Where should the PTA position the electrical stimulation to stimulate contraction of the appropriate muscles? A. Over ankle plantar flexors B. Over back extensors C. Over ankle dorsiflexors D. Over hip abductors

C. Over ankle dorsiflexors Foot drop is a gait deviation that results in weakness or inactivation of the ankle dorsiflexors. As a result, the functional electrical stimulation should be placed over the ankle dorsiflexors to activate this muscle group. This muscle group should be activated during the swing phase in order to allow for foot clearance during swing.

Which position is the BEST indicated for stretching of the hip flexors? A. supine hip flexion and knee extension B. side-lying top leg adduction C. prone hip extension D. hip flexion and adduction

C.Prone Hip Extension Placing the patient prone with hip extension will stretch the hip flexor muscle group When the patient is placed supine with the hip flexed and knee extended, the hamstrings will be stretched. When the patient is placed in side-lying and the top leg is adducted, the hip abductors are stretched. The piriformis is stretched when the patient's lower extremity is positioned in hip flexion and adduction.

When assessing end feels for a typical joint, which would have a normal physiological end feel that may feel soft, with soft tissue approximation? A. Shoulder abduction with a patient who has a torn rotator cuff B. Knee extension C. Elbow extension D. Knee flexion

D. knee flexion a soft end feel due to soft tissue approximation occurs when two soft tissues, typically muscle and/or fat, meet and restrict restrict any further movement. In knee flexion, the hamstring approximates with the gastrocnemius and prevents any further motion at the knee. Knee extension would typically have a firm/capsular end feel,which indicates capsular and ligamentous stretching. Elbow extension would typically have a hard/bony end feel, which indicates when bone and/or cartilage meet. If the patient has a torn rotator cuff and the clinician is assessing the end feel for shoulder abduction, then it would likely be an empty end feel. This occurs when the clinician is unable to bring the joint through the full range of motion due to guarding by the patient, which is typically due to pain.

If a goal is to achieve maximum muscular strength, which resistive training protocol is MOST appropriate? A. Using 40% of 1 RM and performing 3 sets of 20 reps B. Using 70% of 1 RM and performing 1 set of 10 reps C. Using 80% of 1 RM and performing 1 set of 10 reps D. Using 95% of 1 RM and performing 1 set of 8 reps.

D.Using 95% of 1 RM and performing 1 set of 8 repetitions. To achieve maximum strength in a muscle high-resistive loads need to be used but only with relatively low repetitions. Therefore, using 95% of a patient's 1 repetition maximum for 1 set of 8 repetitions is the appropriate choice. To improve muscular endurance, low-load resistance exercise for many repetitions is performed. Using 40% of a patient's 1 repetition maximum is a low-load and performing 3 sets of 20 repetitions is a high amount of repetitions. 70% and 80% of 1 repetition maximum will both achieve a goal of muscle hypertrophy and strength.

After ACL reconstruction, intervention will focus on restoring appropriate knee range of motion. What is the allowed and expected knee motion by the end of the sixth week of rehabilitation? A. 0-120 degrees B. 20-70 degrees C. Knee flexion is contraindicated D. 0-70 degrees

A. 0-120 degrees There are six phases of rehabilitation following an ACL reconstruction: postoperative, maximum protection, controlled motion, moderate protection, minimum protection, and return to activity. Immediately following surgery, a continuous passive motion unit is utilized with PROM form 0-70 degrees of flexion. Motion is increased to 0-120 degrees knee flexion by the sixth week. Initially, reconstruction is protected with a hinged brace set at 20-70 degrees.

At what age can the physical therapist assistant first expect a pediatric patient to creep on hands and knees? A. 8-9 months B. 10-11 months C. 6-7 months D. 12-15 months

A. 8-9 months At age 8-9 months, the infant can typically creep on hands and knees position. At this age, the infant is also able to move from hands and knees to sitting position, sit without hand support, pull to stand, walk along furniture, and stand briefly without support. At 6-7 months, the infant is able to roll from supine to prone and prone to supine, hold weight on one hand in prone to reach for an object, lift head in supine, sit without support, and stand holding onto a surface. At 10-11 months, the infant can pull to stand through half-kneeling, stand without support for longer periods, pick up object from the floor while holding on, walk with two-hand support, and may begin to walk without help a few steps. At 12-15 months, the infant can walk alone, walk backward and sideways, bend over to look between stairs, creep up stairs, and throw a ball in a sitting position.

All prosthetic devices contain a socket and terminal device with varying components in between. What is the terminal device of a lower-limb prosthesis? A. A foot B. A hand C. A hook D. A sock

A. A foot The terminal device of a prosthesis provides an interface between the amputee's prosthesis with the external environment. For a lower-limb prosthesis, the terminal device is a foot. For an upper extremity prosthesis, the terminal device is a hook or hand. A sock is used for providing a soft interface between the residual limb and the socket.

When rising to standing from the floor, which patient will likely display use of upper extremities to push on knees and "walk" up the legs? A. A patient with Duchenne's muscular dystrophy B. A patient with osteogenesis imperfecta C. A patient with developmental dysplasia of the hip D. A patient with cerebral palsy.

A. A patient with Duchenne's muscular dystrophy Duchenne's muscular dystrophy is a sex-linked neurologic condition. The patient with this condition has lower extremity weakness, so would have difficulty rising to stand from the floor. To stand up from the floor, the patient must have enough strength in the hip and knee musculature. To compensate for this, the patient will use the upper extremities to push on the lower extremities and "walk" the arms up the legs unti; standing upright. This is referred to as a Gower's sign.

In a conventional below-elbow prosthesis, which movement controls opening of the terminal device such as a hook? A. Bilateral scapular abduction or ipsilateral flexion of the humerus B. Contralateral humeral extension and internal rotation C. Contralateral flexion of the humerus and unilateral scapular abduction D. Ipsilateral humeral extension and external rotation

A. Bilateral scapular abduction or ipsilateral flexion of the humerus In order to control the movement of the terminal device (hook or hand) in a conventional below-elbow prosthesis, the individual moves in bilateral scapular abduction or ipsilateral humeral flexion. This motion is used to pull on the cable and force opening of the hook or hand.

What special test of the upper extremity evaluates for pathology of structures that pass through the thoracic inlet? A. Adson's Test B. Yergason's Test C. Drop Arm Test D. Clunk Test

A. Adson's Test Adson's Test identifies pathology of the structures that pass through the thoracic inlet. It is used when identifying individuals with thoracic outlet syndrome. The test is positive when the examiner finds the loss of the radial pulse in the arm when the patient rotates the head to the ipsilateral side with neck extension, following deep inspiration. The clunk test identifies a glenoid labrum tear. The drop arm test identifies a tear of the rotator cuff. Yergason's test tests the integrity of the transverse ligament and may identify bicepital tendonitis

Which special test of the upper extremity evaluates for pathology of structures that pass through the thoracic inlet? A. Adson's test B. Yergason's test C. Drop arm test D. Clunk test

A. Adson's test Adson's test identifies pathology of the structures that pass through the thoracic inlet. It is used when identifying individuals with thoracic outlet syndrome. The test is positive when the examiner finds the loss of the radial pulse in the arm when the patient rotates the head to the ipsilateral side with neck extension, following deep inspiration. The clunk test identifies a glenoid labrum tear. The drop arm test identifies a tear in the rotator cuff. Yergason's test tests the integrity of the transverse ligament and may identify bicipital tendonitis.

Which degenerative disease of the central nervous system is characterized as a progressive upper and lower motor neuron disease in which there is muscle fiber atrophy as well as degeneration and scarring of the motor neuron in part of the spinal cord, brainstem, and cerebral cortex? A. Amyotrophic lateral sclerosis B. Multiple Sclerosis C. Meningitis D. Parkinson's disease

A. Amyotrophic lateral sclerosis Amyotrophic lateral sclerosis (ALS) is a progressive upper and lower motor neuron disease in which the muscle fibers atrophy from peripheral nerve involvement. Degeneration and scarring of the motor neurons occur in the lateral aspect of the spinal cord, brainstem, and cerebral cortex. Patients with ALS display asymmetrical, distal weakness with atrophy, facial weakness with difficulty swallowing, as well as hyeractive tendons and reflexes. They have normal cognitive function. They have eventual decline in endurance and aerobic capacity and motor function gradually diminishes until they reach total dependency. Parkinson's disease is a chronic, progressive, degenerative disease of the CNS. It is a basal ganglia disorder characterized by dopamine deficiency and degeneration of the substantia nigra. Multiple sclerosis is a degenerative disease of the CNS characterized by chronic, progressive demyelinating lesions that impair neural transmission and cause nerves to fatigue rapidly. Meningitis is an infectious disorder characterized by inflammation of the meninges of the spinal cord or brain. It is due to a bacterial or viral infection.

Which is NOT a pressure-tolerant area of the typical transtibial residual limb? A. Anterior tibial crest B. Tibial and fibular shafts C. Patellar tendon D. Medial tibial plateau

A. Anterior tibial crest The anterior tibial crest is a pressure-sensitive area of a typical transtibial residual limb. Other pressure-sensitive areas include the anterior tibia, fibular head and neck, and fibular nerve. Pressure tolerant areas include the patellar tendon, medial tibial plateau, tibial and fibular shafts, as well as the the distal end. It is important to know pressure-sensitive versus pressure-tolerant regions of the residual limb in order to make a properly fitting socket and monitor for potential skin breakdown when wearing the socket.

In which direction do MOST dislocations of the glenohumeral joint occur? A. Anterior-inferior B. Posterior-inferior C. Posterior D. Superior

A. Anterior-inferior Most dislocations (95%) occur in anterior-inferior direction. These occur when the abducted upper extremity is forcefully moved into external rotation. This causes tearing of the inferior glenohumeral ligament, anterior capsule, and sometimes the glenoid labrum. Posterior dislocations are rare and occur with multi-directional joint laxity. Posterior dislocations occur with horizontal adduction and internal rotation of the glenohumeral joint. The other options are incorrect and not directions of shoulder dislocations.

Which pediatric disorder is characterized by multiple joint contractures at birth? A. Arthrogryposis Multiplex Congenita B. Legg-Calve'-Perthes Disease C. Slipped Capital Femoral Epiphysis D. Osteogenesis Imperfecta

A. Arthrogryposis Multiplex Congenita Arthrogryposis Multiplex Congenita is a non-progressive neuromuscular disorder that causes multiple joint contractures at birth. There is variability in clinical picture but typically include severe joint contractures and absence of muscle development. Children will utilize splinting, surgery, and assistive devices to compensate and correct for deformities. Legg-Calve'-Perthes Disease is a condition of self-limiting degeneration of the femoral head due to a disturbance in blood supply. It most commonly occurs in boys 4-7 years of age. It is less frequent in girls with onset at older ages. Osteogenesis Imperfecta is characterized by fragile bones that break easily and often for no reason. It is due to an abnormality in the collagen gene causing problems with the amount and quality of collagen in the body. There are four types with varying degrees of severity. Slipped Capital Femoral Epiphysis is a hip deformity related to slippage of the femoral epiphysis. It occurs in children who are tall, with delayed skeletal maturity, obese, and near onset of puberty. It is more common in boys than girls. In mild or moderate cases, surgery is required to pin the hip and prevent further slippage. Severe cases require various osteotomy.

The physical therapist is working with a patient who has HIV. Through which of the following body fluids is HIV transmissible? A. Blood B. Sweat C. Urine D. Vomit

A. Blood HIV is transmissible through contact with infected body fluids such as blood, saliva, semen, cerebral spinal fluid, breast milk, and vaginal/cervical secretions. It is not contracted through respiratory inhalation, skin contact, or human waste of urine, feces, sweat, or vomit.

Which airway clearance technique is MOST appropriate to use for a patient with a spinal cord injury unable to generate enough force to produce a an effective cough? A. Assisted cough B. Huffing C. Tracheal stimulation D. Endotracheal suctioning

A. Assisted Cough Assisted cough is performed when the patient lacks abdominal strength to generate an effective cough such as a patient with a spinal cord injury. The therapist's hands become the force behind the patient's exhaled air. Huffing is an airway clearance technique more effective in patients with collapsible airways such as COPD. It prevents the high intra-thoracic pressure that causes premature airway closure. The technique is performed by taking a moderate or deep breath and forcibly expelling the air, saying "ha ha," such as breathing out to fog the lens of eye glasses. Tracheal stimulation is used with patients unable to cough on command such as infants or individuals who have had a stroke or traumatic brain injury. The therapist's finger is placed above the superasternal notch and a quick inward and downward pressure on the trachea elicits a cough. Endotracheal suctioning is used only when other airway clearance techniques are not adequate. A catheter is fed into an artificial airway, oral airway or nares to the carina. Suction is applied intermittently (10-15 seconds).

Which type of muscle response is noted in a denervated muscle when utilizing electric stimulation modality? A. Asynchronous or vermicular B. There is no contraction C. Brief muscle twitch D. Tetanic or sustained

A. Asynchronous or vermicular An asynchronous or vermicular (worm-like) muscle response is noted in denervated muscle. Low-frequency pulse stimulation produces a breif muscle twitch with each stimulus. Increasing the frequency of the pulse produces a tetanic or sustained muscle contraction in innervated muscles.

Which pulmonary condition is characterized by a collapsed or airless alveolar unit? A. Atelectasis B. Pleural effusion C. Pulmonary emboli D. Pulmonary edema

A. Atelectasis Atelectasis is a collapsed or airless alveolar unit. It is caused by hypoventilation secondary to pain during the ventilator cycle, internal bronchial obstruction, external bronchial compression, low tidal volumes, or neurologic insult. Pleural effusion is excessive fluid between the visceral and parietal pleura caused by increased inflammatory disease, neoplastic disease, or congestive heart failure. Pulmonary emboli are a thrombus from the peripheral venous circulation that becomes embolic and lodges in the pulmonary circulation. Pulmonary edema is excessive seepage of fluid from the pulmonary vascular system into the interstitial space. It can eventually cause alveolar edema. There are cardiogenic and non-cardiogenic forms of pulmonary edema.

A patient has recently undergone a total hip replcaement with the posterior-lateral approach. The PTA is instructing the patient in hip precautions. All of the following are hip precautions after this surgery EXCEPT: A. Avoid hip extension beyond neutral B. Avoid hip internal rotation C. Avoid hip flexion past 90 degrees D. Avoid hip adduction past midline

A. Avoid hip extension beyond neutral After undergoing the total hip replacement, the patient should avoid the position of hip flexion over 90 degrees with adduction and internal rotation. This could place unnecessary pressure on the capsule and risk dislocation.

When working with a client, the therapist notices a raised irregularity on her patient's face, which she is concerned may need further evaluation by a dermatologist. Which form of malignant tumor is characterized by a raised patch with ivory appearance and rolled border with indented center? A. Basal cell carcinoma B. Squamous cell carinoma C. Kaposi's sarcoma D. Malignant melanoma

A. Basal cell carcinoma A basal cell carcinoma is a low-growing epithelia basal cell tumor. It is characterized by a raised ivory color patch with a rolled border and indented center. It rarely metastasized and is common on the face in fair-skinned people. It is associated with prolonged sun exposure. Squamous cell carcinomas have a poorly defined margin. They present as a flat red area, ulcer or nodule. Squamous cell carcinoma grown more quickly and is common on sun-exposed areas including the face, neck, and back of hands. It can metastasize. Malignant melanoma is a tumor that arises melanocytes. Superficial spreading melanoma is the most common type. It is characterized by uneven edges, irregular borders, variations in color, larger than 6 mm, and typically elevated presentation. Kaposi's sarcoma is lesion of endothelial cell origin with red or dark purple macules that progress to nodules or ulcers. It is associated with itching and pain. It is common in lower extremities.

What is the MOST appropriate position for a physical therapist assistant, guarding a patient ascending stairs for the first time using crutches and is partial weight bearing on the left side? A. Behind and slightly to the left side B. In front and slightly to the right side C. Behind and slightly to the right side D. In front and slightly to the left side

A. Behind and slightly to the left side The correct guarding position is always a behind (lower step) and slightly on the involved side, which is the left side in this case. During descending, the therapist should be in front (lower step) and slightly to the involved side.

Which upper extremity condition may lead to pain in the anterior aspect of the shoulder and difficulty with resisted lifting motions below the shoulder level? A. Bicipital tendinitis B. Impingement syndrome C. Adhesive capsulitis D. Thoracic outlet syndrome

A. Bicipital tendinitis Bicipital tendinitis is an inflammation of the long head of the biceps. It results for mechanical impingement of the proximal tendon between the anterior acromion and the bicipital groove of the humerus. This patient description involves pain with muscle activity (which would impact the biceps) as well as pain in the anterior aspect of the shoulder (which is where the biceps tendon inserts). The other options are not characteristic of these symptoms. Adhesive capsulitis is also referred to as frozen shoulder. It is characterized by restriction in the shoulder motion as a result of inflammation and fibrosis of the shoulder capsule. This usually follows disuse after an injury or repetitive microtrauma. Restriction follows capsular pattern of limitation, with the greatest limitation in external rotation followed by abduction and flexion. Impingement syndrome is characterized by soft tissue inflammation of the shoulder from impingement against the acromion with repetitive overhead motion. Thoracic outlet syndrome is compression of the neurovascular bundle in the thoracic outlet between bony and soft tissue structures. The bundle includes the brachial plexus, subclavian artery and vein, vagus and phrenic nerves, and the sympathetic trunk.

A therapist monitors his patient's respiratory rate as 22 breaths per minute. This respiratory rate is considered: A. Normal B. Tachypnea C. Dyspnea D. Bradypnea

B. Tachypnea The normal respiratory rate for an adult is 12-20 breaths per minute. Tachypnea is an increase in rate of breathing greater than 20 breaths per minute. Bradypnea is a decreased rate of breathing less than 12 breaths per minute. Dyspnea is a term for shortness of breath.

The physical therapy evaluation describes a patient having sustained a spinal cord injury due to a traumatic knife injury at T6, resulting in hemisection of the spinal cord. What is this spinal cord injury syndrome known as? A. Brown-Sequard Syndrome B. Central Cord Lesion C. Anterior Cord Syndrome D. Posterior Cord Syndrome

A. Borwn-Sequard Syndrome Brown-Sequard Syndrome is a hemisection of the spinal cord. It typically is caused by penetration wounds (gunshot or knife) with asymmetrical symptoms. Characteristics include ipsilateral loss of dorsal columns with loss of tactile discrimination, pressure, vibration, and proprioception as well as ipsilateral loss of corticospinal tracts with loss of motor function and spastic paralysis below the lesion. There is contralateral loss of sponithalamic tract with loss of pain and temperature below the level of the lesion. At the level of the lesion, there is bilateral loss of pain and temperature. Central Cord Syndrome is characterized by cavitation of central cord in cervical section, loss of spinothalmic tracts with bilateral loss of pain and temperature, loss of ventral horn with bilateral loss of motor function, preservation of proprioceptive, and discriminatory sensation. Anterior Cord Syndrome is a loss of anterior cord with loss of lateral corticospinal tracts with bilateral loss of motor function, spastic paralysis below the level of the lesion, as well as loss of spinothalmic tracts with loss of spinothalmic tracts with bilateral loss of pain and temperature. There is preservation of dorsal columns with proprioception, kinesthesia, and vibratory sense. Posterior Cord Syndrome is a loss of dorsal columns bilaterally with bilateral loss of proprioception, vibration, pressure, and epicritic sensations. There is preservation of motor function, pain, and light touch.

Which of the following hand deformities is a result of rupture a digit's central tendinous slip of the extensor hood? A. Boutonniere deformity B. Mallet finger C. Swan neck deformity D. Ape hand deformity

A. Boutonniere deformity A boutonniere deformity results from the rupture of the central tendinous slip of the extensor hood. The observed deformity is extension of the metacarpophalangeal and distal interphalangeal joint with flexion of the proximal interphalangeal joint. It is a result of trauma or rheumatoid arthritis. A swan neck deformity results from contracture of intrinsic muscles with dorsal subluxation of lateral extensor tendons. The observed deformity is flexion of the metacarpophalangeal and distal interphalangeal joint with extension of the proximal interphalangeal joint. It is a result of trauma or rheumatoid arthritis. An ape hand deformity is observed as thenar muscle wasting, with the first digit moving dorsally in line with the second digit. It is a result of median nerve dysfunction. A mallet finger deformity is due to a rupture or avulsion of the extensor tendon at its insertion into the distal phalanx of a digit. The observed deformity is flexion of the distal interphalangeal joint. It is usually a result of trauma.

What skin color change can be expected in the lower legs for an individual with chronic venous insufficiency? A. Brown B. Yellow C. Skin color changes do not occur with venous insufficiency D. Liver spots

A. Brown A brown color is due to increased pigmentation that can be associated with venous insufficiency. A yellow color indicates jaundice or liver disease. It can also be associated with increased carotene intake. Liver spots are brownish yellow and can be due to aging, uterine and liver malignancies, as well as pregnancy. Changes in skin color are a worthwhile method for tissue assessment.

A patient sustained a spinal cord injury and presents with ipsilateral loss of tactile discrimination, pressure and vibration, as well as ipsilateral loss of motor function and spastic paralysis below the lesion. She has contralateral loss of pain and temperature below the lesion and at the level f the lesion, she has bilateral loss of pain and temperature. Which clinical syndrome does this patient likely have? A. Brown-Sequard Syndrome B. Central Cord Lesion C. Anterior Cord Syndrome D. Posterior Cord Syndrome

A. Brown-Sequard Syndrome Brown Sequard Syndrome is a hemisection of the spinal cord. It typically is caused by penetration wounds (gunshot or knife) with asymmetrical symptoms. Characteristics include ipsilateral loss of dorsal columns with loss of tactile discrimination, pressure, vibration and proprioception as well as ipsilateral loss of corticospinal tracts with loss of spinothalamic tract with loss of pain and temperature below the level of the lesion. At the level of the lesion, there is bilateral loss of pain and temperature. Central Cord Syndrome is characterized by cavitation of central cord in cervical section, loss of spinothalamic tracts with bilateral loss of pain and temperature, loss of ventral horn with bilateral loss of motor function, preservation of proprioceptive and discriminatory sensation. Anterior Cord Syndrome is a loss of anterior cord with loss of of lateral corticospinal tracts with bilateral loss of motor function, spastic paralysis below level of the lesion, as well as loss of spinothalmic tracts with bilateral loss of pain and temperature. There is preservation of dorsal columns with proprioception, kinesthesia, and vibratory sense. Posterior Cord Syndrome is a loss of dorsal columns bilaterally with bilateral loss f proprioception, vibration, pressure, and epicritic sensations. There is preservation of motor function, pain, and light touch.

The physical therapist assistant is working with a patient who has sustained a closed traumatic brain injury. The patient complains of a severe headache and is suddenly lethargic. What is the MOST appropriate action by the PTA? A. Call 911 for emergency services B. Have the patient take an acetaminophen for headache pain C. Give the patient a seated rest with water D. Monitor vitals through the remainder of the therapeutic exercises

A. Call 911 for emergency services The physical therapist must be aware of signs of increased intracranial pressure when working with a patient with a closed traumatic brain injury. Signs of increased intracranial pressure include severe headache and rapid changes in levels of consciousness. This is considered a medical emergency, and the patient should be sent to the hospital for evaluation.

A physical therapist assistant works with another physical therapist to write a study, describing the treatment interventions and outcomes of a patient with a unique condition and unusual outcomes after physical therapy. What type of study is this considered? A. Case report B. Cohort study C. Systemic review D. Randomized control trial

A. Case report A case report is a type of descriptive research in which only one patient is studied in depth, and a retrospective report is written about the case. When writing a report about a specific patient retrospectively, it would be considered a case report. A cohort study is a prospective, forward-in-time, study in which a cohort, group of participants, with a similar condition is followed for a defined amount of time and compared to a matched group without that condition. A randomized control trial is an experimental study in which participants are randomly assigned to either an experimental or control group, receiving the variable intervention or placebo. A systematic review is a review in which many primary studies about a similar topic are combined and summarized.

The physical therapy evaluation indicates a positive Lachman's test of the right knee of a soccer player. The patient has opted to avoid surgery at this time. What is the BEST initial treatment intervention to build strength? A. Closed chain terminal knee extension exercises B. Plyometrics C. Open chain terminal knee extension exercises D. Lateral cutting coordination activities

A. Closed chain terminal knee extension exercises A positive Lachman's test indicates laxity of the anterior cruciate ligament (ACL). Closed chain terminal knee extension exercises are safe and effective for knee complex strengthening due to inherent stability with the exercise. Open chain terminal knee extension would place excessive load on the ACL. Plyometric and quick cutting agility training is not indicated due to heavy joint loading and stress on the ACL in early rehab.

During which stage of motor learning does the learner develop an understanding of the task and determine "what to do"? A. Cognitive stage B. Practice stage C. Autonomous stage D. Associated stage

A. Cognitive stage The three stages of motor learning include the cognitive, associated, and autonomous stages. The practice stage is not a stage of motor learning. The cognitive stage is when the learner develops an understanding of the task, assesses abilities, and task demands and learns "what to do". The associated stage is when the learner practices movements, refines motor programming, and decreases errors, learning "how to do" the task. The autonomous stage is when the learner practices movements, continues to refine movements, displays highly organized control, and is mostly error-free, learning "how to succeed" at the task.

During application of ice massage, the patient will experience what sequence of physiological response stages? A. Cold, burning, aching, numbness B. Aching, burning, cold, numbness C. Cold, aching, burning, numbness D. Numbness, cold, aching, burning

A. Cold, burning, aching, numbness During the application of ice massage, the patient experiences the sequence of cold, burning, aching, and numbness to the application region. Ice cylinders are formed by freezing water in a paper or Styrofoam cup r with a wooden tongue depressor stick. An ice massage is done with direct placement of ice over the treatment area.

In regards to wound assessment, what are the characteristics of wound exudate described as sanguineous? A. Containing blood B. Containing eschar C. Watery serum D. Containing pus

A. Containing blood Wound exudate type should be determined in the assessment of wounds. Sanguineous drainage contains blood. Serous drainage is a watery serum. Purulent drainage contains pus. Eschar is dried necrotic tissue.

Which of the following is a sign of left-sided heart failure? A. Cough B. Nausea C. Anorexia D. Weight gain

A. Cough Signs associated with left-sides heart failure include fatigue, cough, shortness of breath, dyspnea on exertion, orthopnea, paroxysmal nocturnal dyspnea, and diaphoresis. Left heart failure (forward heart failure) is a condition in which blood is not adequately pumped into systemic circulation. It is due to an inability of the left ventricle to pump and increases in ventricular end-diastolic pressure and left artial pressures. The other answer options are signs associated with right-sided heart failure. Signs associated with right-sided heart failure include nausea, anorexia, weight gain, ascites, and right upper quadrant pain. Right heart failure (backward heart failure) is a condition in which the blood is not adequately returned from the systemic circulation to the heart. It is due to failure of the right ventricle and increased pulmonary artery pressures.

A patient has impaired ability to smell. Which cranial nerve may be involved? A. Cranial nerve I B. Cranial nerve II C. Cranial nerve III D. Cranial nerve IV

A. Cranial nerve I Cranial nerve I is the olfactory nerve. It is a sensory nerve responsible for the function of smell. Cranial nerve II is the optic nerve. It is a sensory nerve responsible for the function of visual acuity. Cranial nerve III is the oculomotor nerve. It is a motor nerve responsible for turning the eye up, down, and in. Cranial nerve IV is the trochlear nerve. It is a motor nerve responsible for turning the adducted eye down

A patient has impaired visual acuity. Which cranial nerve may be involved? A. Cranial nerve II B. Cranial nerve IV C. Cranial nerve III D. Cranial nerve I

A. Cranial nerve II Cranial nerve II is the optic nerve. It is a sensory nerve responsible for the function of visual acuity. Cranial nerve I is the olfactory nerve. It is a sensory nerve responsible for the function of smell. Cranial nerve III is the oculomotor nerve. It is a motor nerve responsible for turning the eye up, down, and in. Cranial nerve IV is the trochlear nerve. It is a motor nerve responsible for turning the adducted eye down.

When working with a child with spastic cerebral palsy (high tone), the goal for intervention is to: A. Decrease motor output B. Increase environmental stimuli C. Increase motor output D. Increase stimulation of child

A. Decrease motor output For a child with high tone, the goal is to decrease motor output. Handling techniques include rocking, firm touch, rhythmic movements, slow movements, stroking, warm water, and wrapping/swaddling. Environmental stimuli should provide consistent sensory input, relaxing music, and quiet voice. Increasing environmental or sensory stimulation may further increase tone. For a child with low tone, the goal is to increase motor output.

Which is NOT a physiologic effect of massage? A. Decreased venous and lymphatic flow B. Muscle relaxation C. Pain Modulation D. Reduction of edema

A. Decreased venous and lymphatic flow Massage has the following physiological effects: increased venous and lymphatic flow,stretching of adhesions, edema reductions, sedation, muscle relaxation, and pain modulation.

Which arterial disease is marked by an inappropriate elevation of blood glucose levels and accelerated atherosclerosis? A. Diabetic angiopathy B. Arteriosclerosis obliterans C. Raynaud's disease D. Thromboangiitis obiterans

A. Diabetic angiopathy Diabetic angiopathy is an inappropriate elevation of blood glucose levels and accelerated atherosclerosis. Neuropathy is a major problem, and patients may develop neurotrophic ulcers. Thromboangiitis obiterans is a chronic, inflammatry vascular occlusive disease of small arteries and also veins. It commonly occurs in young adults who smoke. Patients have paresthesias, pain, cyanotic cold extremities, deminished temperature sensation, fatigue, and risk of ulceration. Arteriosclerosis obliterans is a chronic, occlusive arterial disease of medium- and large-sized vessels. It is a result of peripheral atherosclerosis and is associated with hypertension and hyperlipidemia. Raynaud's disease is and episodic spasm of small arteries and arterioles. It is marked by abnormal asconstrictor reflex exacerbated by exposure to cold or emotional stress. Ith tips of fingers develop pallor, cyanosis, numbness, and tingling.

The therapist is treating a pregnant woman who is 30 weeks gestation. The patient complains of a headache and lower energy. The therapist assesses her blood pressure as 145/90 and notes edema in the lower legs. What is the BEST treatment intervention? A. Discontinue the session and initiate an immediate referral to see her physician B. Educate the patient in the role of lower extremity elevation to reduce edema and review medication list. C. Give the patient water and a seated rest D. Provide the patient elastic support hose to reduce edema

A. Discontinue the session and initiate an immediate referral to see her physician These are all signs of preeclampsia, which is a serious pregnancy-induced acute hypertension condition. Preeclampsia is associated with high blood pressure and protein in urine. The signs of preeclampsia include hypertension, edema, sudden excessive weight gain, headache, visual disturbances, or hyperreflexia. The patient should immediately see her physician for further evaluation and care.

A pulmonary patient has developed respiratory acidosis. What should the therapist monitor for? A. Disorientation B. Dizziness C. Convulsions D. Numbness and tingling

A. Disorientation A patient with respiratory acidosis has carbon dioxide retention and impaired alveolar ventilation. Signs include dyspnea, hyperventilation cyanosis, restlessness, headache, and it can lead to disorientation, stupor, or coma, and death. The other options are signs of respiratory alkalosis, which is diminished carbon dioxideand aveolar hyperventilation. Signs include tachypnea, dizziness, anxiety, numbness and tingling, blurred vision, diaphoresis, muscle cramps, twitching, weakness, arrhythmias and convulsions.

Which of the following is NOT effective communication strategy for a physical therapist assistant when communicating with a patient? A. Displaying pity toward the patient and his situation B. Using open body language such as facing the sender at eye level C. Utilization of "I statements" to clarify what was heard D. Speaking in even and moderate tones sensitive to the patient's background

A. Displaying pity toward the patient and his situation The physical therapist assistant should resist pitying the patient. This conveys an inequality between the provider and the patient and that the patient is somehow beneath the provider due to his condition. There is a difference between showing empathy and pity. The other options are all appropriate forms of good communication.

Which special test of the wrist and hand would be performed to identify a patient with de Quervain's tenosynovitis? A. Finkelstein's test B. Tinel's sign C. Phalen's test D. Froment's sign

A. Finkelstein's test Finkelstein's test is used to indentify de Quervain's tenosynovitis. It is performed by grasping the patient's thumb and sharply moving the hand into ulnar deviation. A positive test will cause pain over the distal radius and forearm. Froment's sign identifies ulnar nerve dysfunction. Both Tinel's sign and Phalen's test identify carpal tunnel compression of the median nerve.

Which form of resistance exercise uses dynamic muscle lengthening and is used to prepare muscles for functional activities such as descending stairs? A. Eccentric B. Isometric C. Isotonic D. Isokinetic

A. Eccentric Eccentric resistance exercise is performed with dynamic lengthening contraction. This is useful to prepare muscles for functional activities and for muscles unable to perform concentric contraction. Maximum eccentric contraction produces more force than maximal concentric contraction. Isometric resistance exercise is performed as a static muscle contraction, with no muscle length change. The patient holds a muscle contraction for at least 6 seconds. This is beneficial to strengthen a muscle at a specific point in the ROM. Isotonic exercises use dynamic muscle contraction. There is constant (free weights) or variable (machine) resistance and uses concentric and eccentric contractions. The patient moves a resistance through the available range of motion and speed can be variable. Isokinetic resistance exercise is dynamic and has a speed control for muscle shortening and lengthening. Resistance is accommodating and variable. The individual uses isokinetic equipment, and it provides maximum resistance at all points of the range of motion.

For patients with burns, it is important that the therapy team work to prevent or reduce complications of immobilization through positioning and splinting to prevent contractures. Which is the correct position to place a patient with an anterior neck burn? A. Emphasize hyperextension and position with plastic cervical orthosis B. Emphasize flexion and position with chin to chest C. Positioning does not matter in this burn injury D. Emphasize neutral spine with plastic cervical orthosis

A. Emphasize hyperextension and position with plastic cervical orthosis In an anterior neck burn, the common deformity is neck flexion contracture due to shortening of scar tissue in the anterior neck. For this reason, the patient should be positioned in a way to emphasize neck hyperextension. Positioning with a firm (plastic) cervical orthosis is indicated. Positioning in neck flexion or neutral spine is not appropriate as both could promote soft tissue shortening.

The medical history reveals that a patient has sustained a second-degree, deep partial thickness burn. What anatomical layers are affected in this type of injury? A. Epidermis, dermis, nerve endings, hair follicles, and sweat glands B. Epidermis, dermis, subcutaneous tissue, muscle, and bone C. Epidermis, dermis, and subcutaneous tissue D. Epidermis and upper layers of the dermis

A. Epidermis, dermis, nerve endings, hair follicles, and sweat glands In a second-degree, deep partial thickness burn, the layers affected include the epidermis, dermis, nerve endings, hair follicles, and sweat glands. In a first-degree, superficial burn, the layers affected include the epidermis. In a second-degree, superficial partial thickness burn, the layers affected include the epidermis and upper layers of the dermis. In a third-degree, full thickness burn, the layers affected include the epidermis, dermis, and subcutaneous tissue. In a fourth-degree, subdermal burn wound, the layers afected include the epidermis, dermis, subcutaneous tissue, muscle, and bone.

A patient has sustained a fourth-degree subdermal burn wound. What skin and/or tissue layers are affected? A. Epidermis, dermis, subcutaneous tissue, muscle, and bone B. Epidermis, dermis, subcutaneous tissue C. Epidermis, dermis, nerve endings, hair follicles, sweat glands, D. Epidermis and upper layers of the dermis

A. Epidermis, dermis, subcutaneous tissue, muscle, and bone In fourth-degree, subdermal burn wound, the layers affected include the epidermis, dermis, subcutaneous tissue, muscle, and bone. In a third-degree, full thickness burn, the layers affected include the epidermis, dermis, and subcutaneous tissue. In a second-degree, deep partial thickness burn, the layers affected include the epidermis, dermis, nerve endings, hair follicles, and sweat glands. In a second-degree, superficial partial thickness burn, the layers affected include the epidermis and the upper layers of the dermis. In a first-degree, superficial burn, the layers affected include the epidermis.

Which component of physical therapy care CANNOT be completed by the PTA and requires completion by the overseeing PT? A. Establishing plan of care B. Educating family members on home safety C. Execution of interventions D. Administering and educating of a home exercise program

A. Establishing plan of care The role of the physical therapist is to examine patients, evaluate data, establish diagnosis, establish prognosis, establish plan of care, execute interventions, and supervise treatment. The physical therapist assistant is not to establish plan of care. However, the other three answers are within scope of practice for a physical therapist assistant.

What is NOT a sign or symptom of hyperglycemia? A. Excessive Hunger B. Dry Mouth C. Deep, rapid respirations D. Weakness

A. Excessive Hunger Excessive Hunger is a sign of hypoglycemia (low blood sugar), not hyperglycemia. For patients in a hyperglycemic state, there is decreased appetite, nausea or vomiting, and abdominal tenderness. Hyperglycemia is when the blood glucose is > 300 mm/dL and has a gradual onset over days. Additional signs of hyperglycemia include weakness, increased thirst, dry mouth, frequent but scant urination, dulled senses, deep and rapid respirations, rapid and weak pulse, fruity odor to the breath, and hyperglycemic coma. Immediate medical treatment is needed for hyperglycemia.

Which is NOT an expected gait deviation due to increased extensor tone in a patient s/p CVA? A. Excessive knee flexion during stance B. Inadequate hip flexion C. Decreased knee flexion during swing D. Circumduction

A. Excessive knee flexion during stance Excessive knee flexion during stance is a result of weakness or flaccidity in the lower extremity, or increased flexor tone in the lower extremity. Increased extensor tone in the lower extremity can contribute to: circumduction, inadequate hip flexion, decreased knee flexion during swing, knee hyperextension during stance, and foot drop.

Which physical therapy intervention for urinary incontinence aims to re-educate pelvic floor muscles if a patient cannot initiate active contractions? A. Functional electrical stimulation B. Biofeedback C. Behavioral training D. Progressive strengthening

A. Functional electrical stimulation Functional electrical stimulation is used for muscle reeducation if a patient is unable to initiate active contractions in the pelvic floor. Biofeedback uses pressure recordings to reinforce active contractions and relax the bladder. Progressive strengthening uses weighted vaginal cones for home exercises to increase the strength of the pubococcygeus muscles. Behavioral training includes record keeping of voiding and education for lifestyle changes to avoid Valsalva's maneuver and heavy resistance exercises.

Which soft tissue condition is characterized by crystal deposits into peripheral joints as a result of a disorder of purine metabolism? A. Gout B. Torticollis C. Busitis D. Myositis ossificans

A. Gout Gout is a disorder of purine metabolism characterized by elevated serum uric acid. The uric acid changes into crystals and deposits into peripheral joints and tissues. It is most frequently observed in the knee and great toe. Myositis ossificans is a painful condition with abnormal calcification in a muscle belly. It is usually caused by direct trauma to the muscle, resulting in hematoma and calcification. It can also be induced by early aggressive physical therapy following direct muscle trauma. It most frequently affects the quadriceps, brachialis, and biceps brachii muscles. Torticollis is a spasm or tightness of the sternocleidomastoid muscle. It is seen by side-bending toward and rotation away from the affected side. Physical therapy aims to correct for the SCM spasm or tightness and restore normal range and head motion. Bursitis is an inflammation of the bursa secondary to overuse , trauma, gout, or infection. It is characterized by pain at rest and limited range of motion due to pain, not capsular restriction. Physical therapy aims to reduce inflammation, reduce pain, and improve flexibility.

The physical therapist assistant is delivering joint mobilization intervention at a patient's shoulder. The therapist delivers 30 seconds of continous small amplitude oscillation at the beginning of the range. Which grade of joint mobilization is being performed? A. Grade 1 B. Grade 2 C. Grade 3 D. Grade 4

A. Grade 1 Grade 1 joint mobilization is being performed in this example. The grades of joint mobilization are determined by dosages of oscillation techniques. Grade 1: small amplitude oscillation at the beginning of range Grade 2: large amplitude oscillation before the point of tissue resistance Grade 3: large amplitude oscillation that pushes against tissue resistance Grade 4: small amplitude oscillation near the end of passie range Grade 5: small ampliude, high-velocity manipulation past the end of passive range

A physical therapist assistant draws a letter in the palm of a patient's hand to test sensation. He is testing what? A. Graphesthesia B. Stereognosis C. Barognosis D. Kinesthesia

A. Graphesthesia Graphesthesia is a test of cortical sensation. It is done when the therapist draws a letter in the palm of the patient's hand. The patient should be able to identify the letter. Barognosis is when similar objects with varying weights are placed in the patient's hand. The patient should be able to identify differences in weight. Stereognosis is when an pbject is placed in a patient's hand, and the patients should be able to identify the object by touch only. Kinesthesis is tested by moving the patient's limb in various directions, and the patient identifies the direction of the motion the limb is being moved.

A patient is walking in a physical therapy clinic. The PTA wants to challenge the patient's balance by making a change to the vestibular system. Which modification would achieve this? A. Have the patient turn his head side to side and up and down when walking B. Have the patient walk with his eyes closed C. Have the patient walk over uneven terrain, oudoors D. Turn the lights off in the clinic and have the patient walk

A. Have the patient turn his head side to side and up and down when walking Sensory training can challenge the balance system. Vestibular changes include having the patient stand and walk with head position changes such as moving side o side or up and down. Additionally, another vestibular change would be to have the patient on a moving surface such as an elevator or bus. Closing the eyes and walking in a dark room are both visual changes. Walking over uneven terrain is a somatosensory change.

What are the two MOST significant milestones in gross motor child development in the first 12 months? A. Head control and sitting B. Rolling and walking C. Rolling and cruising D. Sitting and walking

A. Head control and sitting During the first year, there is a planned and predictable outline of milestones. Head control typically develops around 4 months, rolling occurs between 6-8 months, sitting occurs at 8 months, cruising at 10 months, and walking at 12 months. The two most significant milestones are head control and sitting. Without head control, infants are unable to fix their gaze at something or maintain stability to manipulate an object. Without sitting, the infants are unable to hold their trunk against gravity and begin to move toward mobility.

A physical therapist assistant is working with a patient s/p total knee replacement who is receiving treatment for prevention of venous thrombosis formation. The therapist notes an area of ecchymosis on the skin, secondary to which medication? A. Heparin B. Acetaminophen C. Dilantin D. Hydrocortisone

A. Heparin Heparin is a blood thinner. It is used for prevention of blood-clot formation. Caution mus be used with patients as they may be more prone to bleeding. They can bruise more easily as well. Hydrocortisone is used to decrease inflammatory processes associated with psoriasis, eczemia, and other skin conditions. Acetaminophen is a medication used for pain relief and fever reduction. Dilantin is used to decrease seizure activity.

When utilizing physical agents, which form of hydrotherapy would be BEST indicated to treat a patient with burns and open wounds over 50% of his body surface including his back and trunk? A. Hubbard Tank B. Warm water pool C. Cool water pool D. Whirlpool

A. Hubbard Tank The Hubbard Tank is a total immersion bath in which the water is agitated and mixed with air to be directed around the affected part. The tank is filled with water to the proper level and temperature, adding disinfectants if there are open wounds present. The patient is positioned on a stretcher or pneumatic lift and slowly lowered into the tank. The agitator is turned on and adjusted to the proper direction and force over the treatment area. Hubbard tanks are indicated for decubitus ulcers, open burns and wounds, post-hip fractures, post-surgical hip conditions, chronic musculoskeletal conditions of the neck, shoulders, back, and rheumatoid arthritis. The Whirlpool is incorrect because it would not cover enough body surface area. The warm water and cool water pools are incorrect because they would not provide a sterile environment for the open wounds.

A patient is referred for physical therapy to treat low back pain due to disc herniation. Upon reviewing the patient's medications, the therapist sees he is taking an ACE Inhibitor. What other medical condition does this patient likely have? A. Hypertension B. Congestive heart failure C. Diabetes D. Atrial fibrillation

A. Hypertension ACE inhibitors work to decrease blood pressure. Therefore, this medication is indicated for hypertension. Medications to reduce blood sugar, such as insulin, would be expected for diabetes. Diuretics are common medications for congestive heart failure. Antiarrhythmics are used to control heart conductivity for atrial fibrillation.

A patient has suffered damage to the basal ganglia. What is a likely impact of this injury? A. Impaired motor planning B. Loss of vision C. Hypotonia D. Impaired speech

A. Impaired motor planning The basal ganglia is responsible for converting general motor activity into specific and goal-oriented action plans for movement. Dysfunction or injury to basal ganglia results in problems with motor planning and scaling of movements. Parkinson's disease is an example of a disease affected by the basal ganglia. These patients have difficultly initiating movements. Hypotonia is decreased muscle tone. Injury to the basal ganglia would not result in impaired vision or speech.

A patient has undergone a total knee arthroplasty and is in the acute hospital setting, using a continous passive movement (CPM) machine. The CPM machine will assist in promoting all of the following except what? A. Increased Edema B. Decreased postoperative pain C. Increased range of Motion D. Decreased risk of deep vein thrombosis

A. Increased Edema The CPM machine will assist in decreasing postoperative pain by keeping the lower extremity moving. Additionally, it will assist in increasing range of motion, decrease risk of DVT, and DECREASE edema (not increase) in the surgical limb.

Which federal act mandates that eligible students receive free related services (such as physical, occupational, and speech therapy services) to meet their educational needs? A. Individuals with Disabilities Education Act (IDEA) B. Health Insurance Portability and Accountability Act (HIPAA) C. Joint Commission on the Accreditation of Health care Organizations (JCAHO) D. American Disabilities Act (ADA)

A. Individuals with Disabilities Education Act (IDEA) The IDEA was originally enacted in 1975 and allows children with disabilities from birth to 21 years to recieve free, appropriate education in the least restrictive environment. Additionally, it mandates that eligible students receive related services such as physical, occupational, and speech therapy in order to meet their educational needs. The services must be provided by qualified personnel. The services such as physical therapy are intended to allow the student to meet their educational needs and allows access to the school environment.

The physical therapist assistant is working with a 20-year-old male with spastic cerebral palsy who is ambulatory. What is the common gait deviation that can be expected? A. Internal rotation of the hips and flexed knees B. Decreased stance time unilaterally and slow cadence C. Limited trunk rotation, limited arm swing, and shuffling gait D. Weakness on one side of the body with instability

A. Internal rotation of the hips and flexed knees. Internal rotation of the hips and flexed knees is common for an individual with specific cerebral palsy. The other options are more characteristic of other conditions. In patients with Parkinson's disease, common gait deviations include minimal trunk rotation, decreased arm swing, decreased step length, shuffling gait, forward stooped posture, and instability. Weakness on one side of the body with instability is common for an individual after a stroke. Decreased stance time unilaterally and slow cadence is common for an individual after an orthopedic surgery such as a total hip arthroplasty.

The physical therapist is assisting in selecting wheelchair positioning components for a child with spastic cerebral palsy. Which component functions to provide upper extremity positioning and support to assist with trunk extension? A. Lap tray B. Butterfly strap C. Chest strap D. Pelvic belt

A. Lap tray A lap tray provides upper extremity positioning and support to assist with trunk extension.It also provides a surface for activities of the upper extremities. A pelvic belt maintains pelvic positioning in the chair and helps reduce falls from the chair. A chest strap maintains trunk positioning and prevents falling forward. A butterfly strap provides a broad surface to promote anterior chest support for upright sitting

Which type of cancer primarily affects the blood? A. Leukemia B. Sarcoma C. Lymphoma D. Carcinoma

A. Leukemia Leukemia affects the blood with unrestrained growth of leukocytes. A sarcoma is a malignant tumor that originates in connective and mesodermal tissues (such as the muscle, bone, or fat). A carcinoma is a malignant tumor that originates from epithelial tissues (such as the skin, stomach, colon, breast, and rectum). A lymphoma is a malignant tumor that affects the lymphatic system (such as Hodgkin's disease and lymphatic leukemia)

In order to accomplish postural drainage, a patient is positioned on her right side with her head down and rotated 1/4 turn backward. The therapist places a pillow behind her back and the patient's knees are flexed. What is being drained in this position? A. Lingular segments of the left upper lobe B. Right middle lobe C. Anterior basal segments of the lower lobe D. Superior segments of the lower lobes

A. Lingular segments of the left upper lobe The left upper lobe lingular segments are drained by positioning the patient head down on the right side and 1/4 turn backward. The right middle lobe is drained by positioning the patient head down on the left side and rotated 1/4 turn backward. The anterior basal segments of the lower lobe are drained by positioning the patient on her side, head down, with a pillow under the knees. The superior segments of the lower lobes are drained by placing the patient on her abdomen with two pillows under the hips.

The PTA is providing fall risk reduction education to an elderly patient. Which of the following is NOT an appropriate home strategy? A. Low lighting is utilized B. Phone is within easy reach C. All cords are tucked away D. Scatter rugs removed

A. Low lighting is utilized Rooms should be well lit, and night lights should be used at night. This will provide for the best visualization, and visual input is a component of balance. Low lighting will change visual perception. Other safety strategies in the home include rugs removed, stairs with sturdy handrails, bathroom adaptations in place, phone within easy reach, clean and clutter free environment, chairs are sturdy and have armrests, and cords are all tucked away.

A patient presents with chronic pain in the back that wakes him at night and does not follow typical musculoskeletal patterns. He has a history of prostate cancer, so the therapist refers him to his physician for further workup, as prostate cancer is known to metastasize to the bone. Which form of cancer is also know to metastasize to the bone? A. Lung cancer B. Pancreatic cancer C. Ovarian cancer D. Stomach cancer

A. Lung cancer Lung cancer metastasizes to the bone. Additionally, breast, prostate, thyroid, and lymphatic cancers commonly metastasize to the bone. Pancreatic, ovarian, and and stomach cancers all metastasize to the liver, lung and peritoneum.

Which positioning or mobility device provides a child free floor mobility in a safe environment? A. Mat B. Ball C. Scooter board D. Sidelyer

A. Mat A mat allows for free floor mobility and provides a safe, cushioned surface for a child. It can be utilized in a therapy session or incorporated into a child's school or home environment. A sidelyer gives a child visual access to his hands and keeps the head in midline to decrease asymmetrical tonic neck reflex (ATNR), as well as allow for flexion of the hips and knees to decrease extensor tone. A ball is used to stimulate balance and postural reflexes and aides in development of postural control musculature. A scooter board is used to promote floor-level mobility in prone, supine, or quadruped positions.

A patient s/p CVA presents with left-sided sensory motor loss with face and upper extremity affected more than the lower extremity. He has perceptual deficits and expressive and receptive aphasia. Additionally, he has homonymous hemianopsia. Which CVA syndrome is likely? A. Middle cerebral artery syndrome B. Anterior cerebral artery C. Posterior cerebral artery syndrome D. Vertebrobasilar artery syndrome

A. Middle cerebral artery syndrome CVA syndromes are characterized by the impacted cerebral artery. Middle cerebral artery syndrome is characterized by contralateral sensory motor loss with face and upper extremity affected more than lower extremity. These patients can have perceptual deficits, homonymous hemiopsia, Broca's aphasia, Wernicke's aphasia, and global aphasia. The middle cerebral artery supplies the lateral cerebral hemispheres including the frontal, temporal, and parietal lobes. The anterior cerebral artery supplies the medial part of the frontal and parietal lobe, basal ganglia, and corpus collosum. With anterior cerebral artery syndrome, the patient can experience contralateral sensory and motor loss with lower extremities affected more than upper extremities. The patient can have mental impairment, urinary incontinence, apraxia, slow movement, and behavioral changes. The posterior cerebral artery supplies the occipital lobe, medial and inferior temproal lobe, talamus and midbrain. With posterior cerebral artery syndrome, the patient can experience contralateral sensory and motor loss, homonymous hemianopsia, visual agnosia, oculomotor nerve palsy, involuntary movement, Pusher syndrome, and thalamic pain syndrome. The vertebrobasilar artery supplies the medulla, pons, and cerebellum. Patients with vertebrobasilar artery syndrome have a wide variety of symptoms, ipsilaterally, and contralaterally. They can have cranial nerve involvement, ataxia, Wallenberg's syndrome, and Locked-in syndrome.

Which type of training strategies are used to improve coordination and balance and to assist the central nervous system in adaptation for movement control? A. Motor learning strategies B. Environmental adaptions C. Compensatory strategies D. Remedial strategies

A. Motor learning strategies Motor learning strategies are important to assist the central nervous system in adaption for movement control. Motor learning requires repetition. Sensory cues and feedback are used to enhanced motor performance. A compensatory strategy is one that is used to promote safety and early resumption of functional skills. Safety can be improved by use of appropriate assistive devices, as well as by substitution and alteration of postural strategies. It can also include compensation through adaptation of the environment. Remedial strategies focus on use of the involved body segments (such as a paretic limb in a person with a stroke). Control is developed first in isolated movements and progressed to more complex movements. Control in stability is achieved before moving toward mobility.

The physical therapist assistant is working with a patient who has peripheral vascular disease. Which is NOT a sign or symptom of chronic VENOUS insufficiency? A. Nail changes B. Minimal to moderate steady aching pain in lower leg C. Stasis dermatitis D. Dark, cyanotic thickened skin

A. Nail changes This is a sign of chronic arterial insufficiency. Signs of chronic venous insufficiency include minimal to moderate steady aching pain in the lower leg. There is muscle compartment tenderness and venous dilation or varicosity is present. Patients have dark, cyanotic thickened skin, and it may lead to static dermatitis. Signs of chronic arterial insufficiency include sever muscle pain, intermittent claudication, resting pain, muscle fatigue, and pain in he calf and lower leg. There is decreased or absent pulses, and the skin is dry, pale, and shiny. There is loss of hair and nail changes.

A therapist monitors his patient's respiratory rate as 18 breaths per minute. This respiratory rate is considered: A. Normal B. Tachypnea C. Dyspnea D. Bradypnea

A. Normal The normal respiratory rate for an adult is 12-20 breaths per minute. Tachypnea is an increase in rate of breathing greater than 20 breaths per minute. Bradypnea is a decreased rate of breathing less than 12 breaths per minute. Dyspnea is a term for shortness of breath.

Which condition of the knee is commonly referred to as "jumper's knee" and is mechanical dysfunction resulting in traction apophysitis of the tibial tubercle at the patellar tendon insertion? A. Osgood-Schlatter B. Patella alta C. Patella Baja D. Pes anserine bursitis

A. Osgood-Schlatter Osgood-Schlatter (jumper's knee) is a mechanical dysfunction resulting in traction apophysitis of the tibial tubercle at the patellar tendon insertion. Physical therapy goals include modification of activities to reduce stress. Pes anserine bursitis is typically caused by overuse or a contusion. It is an inflammatory condition of the medial knee, as a result of inflammation of the anserine bursa. Patella alta is a patellar malalignment in which the patella tracks superiorly in the femoral intercondylar notch. Patella Baja is a patellar malalignment in which the patella tracks inferiorly in the femoral intercondylar notch.

What condition is an inflammatory response within a bone caused by an infection? A. Osteomyelitis B. Osteogenesis imperfecta C. Osteomalacia D. Osteoporsis

A. Osteomyelitis Osteomyelitis is an inflammatory response within the bone caused by an infection. The infection is usually caused by Staphylococcus aureus. Medical treatment is antibiotic medication. Osteomalacia is a condition of decalcification of bones due to vitamin D deficiency. Symptoms include severe pain, fractures, weakness, and bone deformities. Patients are given calcium, viatmin D, and calciferol as medical treatment. Osteogenesis imperfecta is an abnormality in the collagen gene that causes problems with the amount and quality of collagen in the body. It is characterized by fragile bones that break easily, dental problems, scoliosis, kyphosis, short stature, and hearing loss. Medical Intervention includes orthotic devices, calcium, and treatment of fractures. Osteoporosis is a disease process that results in reduction of bone mineral density. It is a failure of bone formation to keep pace with bone reabsorption and destruction. Osteoporosis is more common in women, as there is a hormonal deficiency associated with menopause. Treatment includes calcium and vitamin D intake, as well as weight-bearing and strengthening exercises.

Where is the auscultation landmark for the pulmonic valve located? A. Over the second left intercostal space at the sternal border. B. Over the fourth left intercostal space at the sternal border C. Over the fifth left intercostal space at the midclavicular line D. Over the second right intercostal space at the sternal border

A. Over the second left intercostal space at the sternal border. The auscultation landmark for the pulmonic valve is at the second left intercostal space at the the sternal border. The auscultation landmark for the aortic valve is found over the second right intercostal space at the sternal border. The auscultation landmark for the tricuspid valve is found over the fourth left intercostal space at the sternal border. The auscultation landmark for the mitral valve is found over the fifth left intercostal space at the midclavicular line.

Where should the therapist position her hands when instructing a patient to perform diaphragmatic breathing? A. Over the subcostal angle of the thorax B. Over the lateral chest wall bilaterally C. Over the sternum D. Over the posterior aspect of the thoracic ribs

A. Over the subcostal angle of the thorax Diaphragmatic breathing is used to increase ventilation and improve gas exchange, as well as promote relaxation and decrease work of breathing. The patient should be semi-reclined. The therapist's hands are placed over he subcostal angle of the thorax. The therapist applies gentle pressure through exhalation, increased to firm pressure at the end of exhalation, and releases pressure during inhalation. The other options are incorrect because they do not facilitate diaphragmatic breathing.

In reviewing the physical therapist's evaluation, the physical therapist assistant notes her patient tested positive for Clarke's sign. What does this test indicate? A. Patellofemoral dysfunction B. Meniscal tear C. Torn posterior cruciate ligament D. Torn anterior cruciate ligament

A. Patellofemoral dysfunction Clarke's sign indicates patellofemoral dysfunction, a common condition amongst athletes and runners. The examiner places the patient in long sitting with approximately 30 degrees of flexion. The examiner presses down slightly on the patella and asks the patient to contract their quadriceps. Positive test is pain or inability to hold the contraction. Lachman's test is for identifying the integrity of the anterior cruciate ligament. The posterior drawer test exams the integrity of the posterior cruciate ligament. McMurray's test identifies meniscal tears.

Full knee extension is the loose-packed position for which joint? A. Patellofemoral joint B. Hip Joint C. Talocrural joint D. Tibiofemoral joint

A. Patellofemoral joint The loose-packed position is the position in which the capsule and other soft tissues are maximally relaxed. The close-packed position of a joint is the position in which the capsule and other tissues are maximally tensed. Full knee extension is the loose-packed position for the patellofemoral joint. The closed-packed position for the patellofemoral joint is full knee flexion. The loose-packed position for the talocrural joint is 10 degrees plantar flexion of the ankle. The loose-packed position for the tibiofemoral joint is 25 degrees flexion of the knee. The loose-packed position for the hip joint is 30 degrees flexion, 30 degrees abduction, and slight external rotation.

Which member of the health care team would be responsible for ensuring that policies and procedures of the department and facility are carried out? A. Physical therapy director B. Physical therapist assistant C. Physical therapy aide D. Staff physical therapist

A. Physical therapy director Th physical therapy director oversees the functions of the department. He or she ensures that policies and procedures are carried out effectively. He or she acts as a liaison with the facility administration and sets goals and plans for the department. The staff physical therapist is a licensed professional responsible for many aspects of physical therapy patient care including examination, evaluation, establishment of plan of care, treatment and supervision of treatment of treatment by PTAs. The physical therapist assistants are licensed providers who work under the supervision of a physical therapist and perform selected interventions within their scope of practice and the patient's plan of care. The physical therapy aide is a non-licensed worker who works under the continuous supervision of a physical therapist or physical therapist assistant, performing operational duties and patient transport tasks.

Which foot/ankle condition causes acute pain with loading of the foot, especially when first weight bearing in the morning? A. Plantar fasciitis B. Charcot-Marie-Tooth disease C. Talipes equinovarus D. Metatarsus adductus

A. Plantar fasciitis Plantar fasciitis is a chronic irritation of the plantar fascia from excessive pronation. There is limited range of motion of the first metatarsalphalangeal and talocrural joint with tight triceps surae. It results in microtears at the attachment of the plantar fascia. Metatarsus adductus is a deformity that is either congenital or a result of a neuromuscular disease (polio). It results in medial subluxation tarsometatarsal joints, hindfoot slightly valgus, and navicular lateral to the head of the talus in the rigid type. The flexible type results in adduction of all five metatarsals at the tarsometatarsal joints. Charcot-Marie-Tooth disease is a peroneal muscular athrophy that affects the motor and sensory nerves. Talipes equinovarus is clubfoot. Talipes equinovarus is an abnormaldevelopment of the head and neck of the talus as the result of heredity or neuromuscular disorders.

Patients with diabetes should be educated in proper foot care by the treating therapist. Which of the following should the patient avoid? A. Walking barefoot B. Daily inspection of feet C. Daily application of lotion D. Soaking feet in tepid water

A. Walking barefoot Walking barefoot can result in skin abrasions and lesions to the foot, which may not be felt due to the effects of diabetic neuropathy. If a lesion occurs, it may not heal weel due to impaired circulation. The other options should be encouraged, as they all assist in keeping good skin integrity of the feet.

When providing relaxation training to relieve muscular tension, what is the FIRST important step? A. Position patient in a comfortable resting position with all body parts supported B. Instruct the patient in focused relaxation through guided imagery C. Instruct the patient in conscious contraction followed by relaxation of the facial muscles D. Provide active range of motion to the extremities in a pain-free range

A. Position patient in a comfortable resting position with all body parts supported A common error associated with relaxation training is a lack of awareness of the effects a patient's position has and the influence of the environment. It is important to first position the patient properly so his body is fully supported. The other answer choices are all examples of forms of relaxation intervention

The physical therapist assistant is working with a 4-month-old infant with torticollis. Which of the following interventions is appropriate to build strength in the head and neck musculature? A. Position the child in supported sitting and facilitate visual tracking of toy B. Supine position, reaching for a toy C. Prone on elbows position, reaching for a toy D. Seated without support and visually tracking a rattle

A. Position the child in supported sitting and facilitate visual tracking of toy. By age 4-5 months, the head is steady in supported sitting. By 2-3 months, the head bobs in supported sitting. Therefore, this is a good position to promote head and neck musculature strengthening. Visual tracking of a toy will also facilitate the strengthening of the head and neck musculature. Prone on elbows position, reaching for a toy is not appropriate , as infants are not able to hold weight on one hand in prone to reach for a toy until they are 6-7 months of age. Seated without support and visually tracking a rattle is not appropriate , as infants cannot sit unsupported until age 6-7 months. Supine position, reaching for a toy is not appropriate , as the head and neck are supported, not encouraging strengthening in this position.

Regarding stages of motor learning, which is NOT a true stage? A. Practice stage B. Associative stage C. Cognitive stage D. Autonomous stage

A. Practice stage Stages of motor learning include the cognitive stage, associative stage, and autonomous stage. In the cognitive stage, the learner understands the goals of the tasks, develops strategies to perform the tasks, and need to specifically think about how to perform the task. In the associative stage, the actions are perfected to produce the most efficient action. The learner thinks about performing the steps to accomplish the task with greater refinement and precision. The autonomous stage, the learner is able to accomplish the task with very little thinking. The task becomes automatic.

Regarding proprioceptive techniques, which technique is considered inhibitory? A. Prolonged, slowly applied stretch B. Resistance C. Quick stretch and tapping of muscle belly D. Joint traction

A. Prolonged, slowly applied stretch A prolonged and slowly applied stretch inhibits the agonist muscle and decreases tone. Inhibitory pressure is another inhibitory technique, in which firm pressure is provided along tendons. This inhibits muscle and decreases tone. Facilitation techniques include all other options. Quick stretch and tapping of muscle belly facilitates agonist muscle and inhibits antagonist. Resistance recruits motor units and facilitates and strengthens agonist contraction. Joint traction enhances joint awareness and actions of flexors.

Which is NOT one of the three zones of a burn wound? A. Zone of healing B. Zone of hyperemia C. Zone of coagulation D. Zone of stasis

A. Zone of healing The three zones of a burn wound include: Zone of coagulation: Cells are irreversibly injured, and cell death occurs. Zone of stasis: Cells are injured and may die within 24-48 hours without specialized treatment . Zone of hyperemia: Minimal cell injury, cells should recover.

Which speech and communication disorder is characterized by severely impaired auditory comprehension but spontaneous speech is preserved and fluent? A. Receptive aphasia B. Expressive aphasia C. Global aphasia D. Dysarthria

A. Receptive aphasia Receptive aphasia (also known as Wernicke's or fluent aphasia) occurs from damage to Wernicke's area. Spontaneous speech is preserved and fluent, however, auditory comprehension is severely impaired. Expressive aphasia occurs with damage to Broca's area. It results in speech that requires a great deal of effort to produce and when speech occurs, the words are typically restricted, interrupted, and awkward. Global aphasia is a combination of expressive and receptive aphasia resulting in major difficulty with comprehension and expression of language. Dysarthria is characterized by a limited ability to control movements of the jaw, tongue, and respiratory structures needed for speech control. The patient can understand spoken language and use the correct words, but spoken words are hard to understand.

Which pulmonary condition is characterized by alveolar collapse in a premature infant resulting from lung immaturity, inadequate level of pulmonary surfactant? A. Respiratory distress syndrome B. Bronchiectasis C. Cystic fibrosis D. Bronchopulmonary dysplasia

A. Respiratory distress syndrome Respiratory distress syndrome is a condition of alveolar collapse in a premature infant. It is a result of lung immaturity and inadequate levels of pulmonary surfactant. Physical findings are within a few hours of birth and include respiratory distress, crackles, tachypnea, hypoxemia, cyanosis, accessory muscle use, expiratory grunting, and flaring nares. Bronchopulmonary dysplasia is an obstructive pulmonary disease. It is often a sequela of premature infants with respiratory distress syndrome, who were given high pressures of mechanical ventilation, or infection. Lungs show pulmonary immaturity and hyperinflation. Brochiectasis is a chronic congenital or acquired disease characterized by abnormal dilation of the bronchi and excessive sputum production. Cystic fibrosis is a genetically inherited disease that affects the mucus-producing glands of the lungs, sweat glands, digestive tract, and genitourinary system. It is characterized by thickening of secretions of all exocrine glands, leading to obstruction.

A physical therapist assistant is working with a patient who has a recent diagnosis of pancreatic cancer and has begun chemotherapy treatments, resulting in weakness and low activity tolerance. The assistant checks the vital signs of this patient to find: heart rate is 60 beats per minute, respiratory rate is 9 breaths per minute, blood pressure is 116/72, and abdominal pain is rated as 2/10. Which vital sign is of greatest concern? A. Respiratory rate B. Pain rating C. Blood pressure D. Heart rate

A. Respiratory rate The normal respiratory rate for a adult is 12-18 breaths per minute. A respiratory rate of nine breaths per minute is outside normal parameters and will negatively impact other vitals over time including decreased oxygenation. The blood pressure and heart rate are within normal limits. A pain rating 2/10 is not alarming at this time, especially as cancer patients tend to have cancer-related pain. However, the pain rating should be monitored and addressed if possible, possibly with the physician or with repositioning strategies

The patient is placed head down on her left side and rotated 1/4 turn backward in order to accomplish postural drainage. What segment is being drained? A. Right middle lobe B. Left upper lobe, lingular segments C. Anterior basal segments of the lower lobe D. Superior segments of the lower lobes

A. Right middle lobe The right middle lobe is drained by positioning the patient head down on the left side and rotated 1/4 turn backward. A pillow may be placed behind from the shoulder to the hip and the knees should be flexed. The left upper lobe lingular segments are drained by positioning the patient head down on the right side and rotated 1/4 turn backward. The anterior basal segments of the lower lobe are drained by positioning the patient on her side, head down, with a pillow under the knees. The superior segments of the lower lobes that are drained by placing the patient on her abdomen with two pillows under the hips.

In regards to arthrokinematics, joint accessory motion, what is the movement between the femoral and tibial articular surfaces of the knees? A. Roll B. Spin C. Glide D. Flexion

A. Roll Roll consists of one joint surface rolling n another such as a tire rolling over a road. An example is the movement between the femoral and tibial articular surfaces in the knee. Glide or slide consists of a pure translatory motion of one surface gliding on another surface. An example is the movement of the proximal phalanx at the head of a metacarpal bone. Spin consists of a rotation of the movable component of a joint. An example is between the radial head with the humerus. Flexion is a term used to describe osteokinematics, which is physiological motion between two bones.

Regarding neoplasms, which type of malignant tumor arises from the connective tissues? A. Sarcoma B. Carcinoma C. Lymphoma D. Leukemia

A. Sarcoma A sarcoma is a malignant tumor that originates in connective and mesodermal tissues (such as the muscle, bone, or fat) A carcinoma is a malignant tumor that originates from epithelial tissues (such as the skin, stomach, colon, breast, and rectum). A lymphoma is a malignant tumor that affects the lymphatic system (such as Hodgkin's disease and lymphatic leukemia). Leukemia affects the blood with unrestrained growth of leukocytes

Which of the following is NOT a movement pattern associated with an extension synergy? A. Scapula retraction and elevation B. Wrist and finger flexion C. Shoulder adduction and internal rotation D. Forearm pronation

A. Scapula retraction and elevation Scapular retraction and elevation are part of a flexion synergy pattern. Extension synergy patterns include protraction of scapula, shoulder adduction and interal rotation, elbow extension, forearm pronation, wrist and finger flexion, hip extension and adduction, knee extension, ankle plantarflexion, and inversion and toe plantarflexion. Flexion synergy patterns include retraction and elevation of the scapula, shoulder abduction and external rotation, elbow flexion, forearm supination, wrist and fingerflexion, hip flexion/abduction/external rotation, knee flexion, ankle dorsiflexionand inversion, and toe dorsiflexion.

Which position should a patient be placed for postural drainage of the posterior segments of the upper lobes? A. Seated, leaning over a folded pillow at a 30-degree angle B. The patient is on her abdomen with two pillows under her hips C. The patient is on her back with a pillow under her knees D. The patient is on her abdomen in a head-down position with a pillow under her hips

A. Seated, leaning over a folded pillow at a 30-degree angle To clear the posterior segments of the upper lobes, the patient sits, leaning over a folded pillow at 30 degrees. The therapist performs percussions over the upper back on both both sides. To clear the superior segments of the lower lobes, the patient is prone on the abdomen with two pillows under her hips. In order to promote postural drainage of the anterior segments of the upper lobes, the patient is on her back with a pillow under her knees. To clear to posterior basal segments of the lower lobes, the patient is on the abdomen in a head-down position with a pillow under her hips.

The physical therapist assistant is evaluating a patient's exteroceptive sensation. Which is a test of exteroceptive sensation? A. Sharp/dull discrimination B. Two-point discrimination C. Vibration D. Kinesthesia

A. Sharp/Dull discrimination Exteroceptive sensation is a test of superficial sensation. These tests include sharp/dull discrimination, temperature, and light touch. Proprioceptive (deep) sensation testing includes kinesthesia, proprioception, and vibration. Combined (cortical) sensation testing includes stereognosis, two-point discrimination, texture recognition, tactile localization, graphesthesia, and barognosis.

What is the expected intervention for a patient after total hip replacement with partial weight-bearing and pain well controlled on post-surgery day two? A. Short distance ambulation with a rolling walker B. Ambulation without an assistive device C. Stair navigation over 10 stairs D. Bed mobility only

A. Short distance ambulation with a rolling walker On post-surgery day two the therapist should be able to initiate gait training with an appropriate assistive device, educating in weight bearing restrictions. Ambulation without an assistive device cannot be properly done while maintaining partial weight bearing. Stairs training over 10 stairs is too advanced for day two. Bed mobility only is too limited for intervention, unless there is a medical complication or high pain/low endurance.

When performing gait analysis of a patient with an above-knee amputation, the PTA notices the patient displays circumduction during the swing phase of gait on the prosthetic limb. Which condition is NOT cause of this deviation? A. Short prosthesis B. Abduction contracture C. Long prosthesis D. Inadequate suspension

A. Short prosthesis There are many factors that can contribute to noticeable circumduction of the prosthetic limb during swing phase. Factors include long prosthesis, locked knee unit, loose friction, inadequate suspension, small or loose socket, abduction contracture, and poor knee control. A short prosthesis can contribute to lateral bending in the stance phase of gait.

When assessing end feels for a typical joint, which would have a normal physiological end feel that would be described as empty? A. Shoulder abduction in a patient with a torn rotator cuff B. Knee extension C. Elbow extension D. Knee flexion

A. Shoulder abduction in a patient with a torn rotator cuff If the patient has a torn rotator cuff and the clinician is assessing the end feel for shoulder abduction, then it would likely be an empty end feel. This occurs when the clinician is unable to bring the joint through the full range due to guarding by the patient, which is typically due to pain. Elbow extension would typically have a bony/hard end feel, which indicates when bone and/or cartilage meet. This bony end feel occurs when the humerus and ulna meet. Knee extension would typically have a firm/capsular end feel, which indicates capsular and ligamentous stretching. This feels like stretching a leather belt. A soft end feel due to soft tissue approximation occurs when two soft tissues, typically muscle and/or fat, meet and restrict any further movement. In knee flexion, the hamstring approximates with the gastrocnemius and prevents any further motion of the knee.

Which special test of the shoulder identifies bicipital tendonitis? A. Speed's test B. Neer's impingement test C. Drop arm test D. Clunk test

A. Speed's test The speed's test identifies bicipital tendonitis. It is performed by having the patient hold his straight arm forward at 90-degrees flexion and resisting the therapist's downward pressure. A positive test is if the patient experiences pain over the area of the biceps tendon. Neer's impingement test identifies impingement of soft tissue structures of the shoulder complex. The drop arm test identifies a tear in the rotator cuff. The clunk test identifies glenoid labrum tears.

The physical therapist is working with a patient who has chronic low back pain. He notes the pain decreases with flexed positions and increases with extension positions. This pattern is MOST closely related to which condition? A. Spinal stenosis B. Vascular claudication C. Neoplastic disease D. Degenerative joint disease

A. Spinal stenosis Spinal stenosis pain is associated with position. Flexed positions decrease pain while extended positions increase pain. The pain is typically described as numbness, tightness, or cramping. Walking any distances brings on symptoms of pain. With vascular claudication, pain is consistent in all spinal positions and brought on by physical exertion. With neoplastic disease, pain is not resolved by changes in position or activity level and pain will wake the patient. The pain is described as gnawing, intense, and penetrating. With degenerative joint disease, the patient experiences pain and stiffness upon rising and easing of pain through the morning. Pain increases with repetitive bending and is described as soreness and nagging.

The physical therapist is working with a patient who has chronic low back pain. He notes the pain is decreased with flexed positions and increased with extension positions. This pattern is MOST closely related to which condition? A. Spinal stenosis B. Vascular claudication C. Neoplastic disease D. Degenerative joint disease

A. Spinal stenosis Spinal stenosis pain is associated with position. Flexed positions decrease pain while extension positions increase pain. The pain is typically described as numbness, tightness, or cramping. Walking any distances brings on symptoms of pain. With vascular claudication, pain is consistent in all spinal positions and brought on by physical exertion. With neoplastic disease, pain is not resolved by changes in position or activity level and pain will wake patient. The pain is described as gnawing, intense, and penetrating. With degenerative joint disease, the patient experiences pain and stiffness upon rising and easing of pain through the morning. Pain increases with repetitive bending and is described as soreness and nagging.

The initial evaluation evaluation reveals a patient has an antalgic gait pattern. She has recently had a total knee replacement on her right lower extremity and is full weight bearing. Which gait deviation can be expected? A. Stance time is abbreviated on the right leg,with a shortened step length on the left leg B. Stance time is abbreviated on the left leg, with a shortened step length on the left leg C. Stance time is abbreviated on the right leg, with a shortened step length on the right leg D. Stance time is abbreviated on the left leg, with a shortened step length on the right leg

A. Stance time is abbreviated on the right leg,with a shortened step length on the left leg An antalgic gait pattern is a painful gait pattern. The stance time is shortened on the painful limb. This results in uneven stepping and an uneven gait pattern. The uninvolved leg has a shortened step length since it needs to bear weight sooner than normal. For this situation with the right total knee replacement, the right lower extremity is the painful limb. The right leg has a shortened stance time, and the left leg has the shortened step length.

The PTA is working to improve standing balance. Which sensory training intervention would be MOST challenging? A. Standing over dense foam with eyes closed B. Standing over dense foam with eyes open C. Standing over carpet with eyes open D. Standing over tile floor with eyes open

A. Standing over dense foam with eyes closed Sensory training to improve balance includes introducing various changes to the balance system. Visual changes are eyes open to eyes closed and full lighting to reduced lighting. Somatosensory changes include even terrain to more unstable terrain. Vestibular changes include moving head side-to-side, up and down, and standing or walking on a moving surface such as an escalator or elevator. Introducing sensory conflict situations will further challenge a patient as when the patient stands on an unstable foam surface with the eyes closed.

The physical therapist assistant is educating family members about pressure ulcer formation and prevention. In which position would a pressure ulcer MOST LIKELY form over the heels? A. Supine B. Sidelying C. Sitting D. Standing

A. Supine The heels would have the most pressure placed on them when the patient is positioned in supine. The greater trochanter and lateral malleolus are most susceptible when the patient is positioned in side-lying. The ischial tuberosities are most susceptible in sitting. Typically, for patients able to stand, pressure ulcers do not form. Bony prominences are susceptible to pressure ulcers. Risk factors for pressure ulcer formation include immobility and inactivity, sensory impairment, cognitive deficits, impaired circulation, poor nutrition, and incontinence.

The physical therapist assistant is educating family members about pressure ulcer formation and prevention. In which position position would a pressure ulcer MOST LIKELY form over the sacrum? A. Supine B. Side-lying C. Seated in wheelchair D. Standing

A. Supine The sacrum would have the most pressure placed on it when the patient is positioned supine. The greater trochanter and lateral malleolus are most susceptible when the patient is positioned in side-lying. The ischial tuberosities are most susceptible in sitting. Typically, for patients able to stand, pressure ulcers do not form. Bony prominences are susceptible to pressure ulcers. Risk factors for pressure ulcer formation include immobility and inactivity, sensory impairment, cognitive deficits, impaired circulation, poor nutrition, and incontinence.

Which massage technique describes rapid striking with the palmar surface of the hand in an alternating pattern in order to promote stimulation of the treatment area? A. Tapotement B. Effleurage C. Vibration D. Petrissage

A. Tapotement Tapotement is rapid striking of the surface of the palm, cupped hand, or edge of hand over the patient's skin. It is used to stimulate the desired treatment effect. The clinician must utilize clinical judgement to avoid too much intensity when delivering this technique. Effleurage is a technique by which the clinician provides gliding movement of the hands over the surface of the skin. Vibration is shaking of the tissue with short, rapid motions. Petrissage is a kneading that grasps and lifts the tissues.

Which is a false statement in regards to the role is the PT/PTA team? A. The PTA may adjust the plan of care if the patient displays a change in status in order to reach established goals B. The PTA must follow the plan of care established by the PT C. The PT is the only individual who can clinically direct and supervise a PTA D. The PTA must practice under the direction of a PT

A. The PTA may adjust the plan of care if the patient displays a change in status in order to reach established goals The PTA may not adjust the plan of care established by a PT. If the PTA identifies a possible need for change or update to the plan of care, then the PTA must discuss this with the PT. If the PT finds a change necessary, then the PT will need to update the plan of care. The other options are true statements regarding PT/PTA roles.

Which position would a patient be placed to promote postural drainage of the superior segments of the lower lobes? A. The patient is on her abdomen with two pillows under her hips B. The patient is on her back with a pillow under her knees C. The patient sits, leaning over a folded pillow at 30 degrees D. The patient is on her abdomen in a head-down position with a pillow under her hips

A. The patient is on her abdomen with two pillows under her hips To clear the superior segments of the lower lobes, the patient lies prone on abdomen with two pillows under her hips. The therapist performs percussions over the middle of the back tip of the scapula on either side of the spine. In order to promote postural drainage of the anterior segments of the upper lobes, the patient is on her back with a pillow under her knees. To clear the posterior segments of the upper lobes, the patient sits and leans over a folded pillow at 30 degrees. To clear the posterior basal segments of the lower lobes, the patient is on the abdomen in a head down position with a pillow under her hips.

Which of the following would NOT be considered an example of negligence? A. The patient was sitting in his wheelchair in the rehab department, waiting for his therapist for 1 hour B. A cognitively impaired patient fell and broke his hip after he was left unsupervised in his room without any alert system C. A patient was injured after a therapist failed to utilize a gait belt when performing stair training D. Patient was harmed when he fell due to a slippery floor

A. The patient was sitting in his wheelchair in the rehab department, waiting for his therapist for 1 hour. Although this would be considered poor customer service and may make the patient upset, this is not an example of negligence. For a situation to qualify as negligence, there must be: failure to do what a reasonably competent practitioner would have done under similar circumstances and harm must have occurred to the patient. Additionally, the institution can be found negligent if a patient was harmed due to an environmental problem such as a slippery floor.

In reviewing the physical therapist's evaluation, the physical therapist assistant notes his patient tested positive for Ober's test. What does this test indicate? A. Tightness of the tensor fascia latae and/or iliotibial band B. Tightness of hip flexion C. Piriformis syndrome D. Tightness of rectus femoris

A. Tightness of the tensor fascia latae and/or iliotibial band Ober's test indicates tightness of the tensor fascia latae and/or the iliotibial band. It is performed by placing the patient in sidelying while the examiner passively extends the lop lower extremity backward. Then the examiner brings the leg down toward the table. Thomas's test identifies tightness of hip flexion. Ely's test identifies tightness of rectus femoris. Piriformis test identifies piriformis syndrome.

Which condition is commonly occurring in running athletes due to irritation from a tight iliotibial band? A. Trochanteric busitis B. Avascular necrosis C. Slipped capital femoral epiphysis D. Piriformis syndrome

A. Trochanteric bursitis Tight iliotibial bands, which commonly occur in running athletes can result in inflammation of the trochanteric bursa. Treatment includes reduction in pain and inflammation, correcting muscle imbalance and flexibility issues, gait, and running training. Piriformis syndrome is a condition in which the tightness or spasm of the piriformis muscle can result in compression of the sciatic nerve and contribute to sacroiliac dysfunction. Avascular necrosis is a condition of impaired blood supply to the femoral head. Slipped capital femoral epiphysis is a hip deformity related to slippage of the femoral epiphysis. It can be genetic or hormonal and lead to weak growth plates. It occurs in children near the onset of puberty age.

What form of rigidity is common for individuals who have sustained a traumatic brain injury with lesion of the brainstem? A. UE and LE in extension B. UE and LE in flexion C. UE in extension and LE in flexion D. UE in flexion and LE in extension

A. UE and LE in extension Decerebrate rigidity is a form of rigidity common with lesions of the brainstem. The UE and LE are held in extension. Decorticate rigidity is a form of rigidity common with lesions above the brainstem. It is characterized by UE flexion and LE in extension. The other options are not typical forms of rigidity.

The physical therapist assistant is working with a patient that has class IV congestive heart failure. What is the maximal METS for physical activity for this patient? A. Up to 1.5 METS B. UP to 4.5 METS C. Up to 6.5 METS D. Up to 3 METS

A. Up to 1.5 METS Patients with congestive heart failure (CHF) demonstrate significant ventricular dysfunction, decreased cardiac output, and low functional capacities. METS are metabolic equivalents of oxygen consumption. CHF can be classified as class I-IV. Class I: Mild CHF; no limitation on physical activity, up to 6.5 METS. Patients are comfortable at rest; ordinary activity does not cause undue fatigue, dyspnea, or angina pain. Class II: Mild CHF; slight limitation in physical activity, up to 4.5 METS; Patients are comfortable at rest, and ordinary physical activity results in fatigue, palpitation, dyspnea, or angina pain. Class III: Moderate CHF; marked by limitation of physcial activity, up to 3 METS. Patients are comfortable at rest , but less than ordinary activities cause fatigue, palpitation, dyspnea, or angina pain. Class IV: Severe CHF; patients are unable to carry out any physical activity, 1.5 METS, without discomfort. Symptoms of ischemia, dyspnea, or angina pain are present even at rest and symptoms increase with exercise.

A patient with right-side hemiplegia is gait training with a hemiwalker. The patient is having difficulty advancing her right lower extremity forward. The BEST initial feedback that the physical therapist assistant should give is: A. Verbally cue to shift weight to the left B. Physically lift the right lower extremity and place it forward C. Verbally cue the patient to shift weight posteriorly D. Verbally cue to shift weight to the right

A. Verbally cue to shift weight to the left The patient must shift the weight toward the left in order to unweight the R LE and allow for advancement of the R LE. The most appropriate feedback is to cue the patient to weight shift left. Cueing the patient to shift weight to the right or posteriorly would hinder her ability to take a step forward. Physically lifting the right lower extremity forward would be indicated if the patient is not able to complete the task with a verbal cue

The medical history reveals that a patient suffered a full thickness, third-degree burn to his right lower leg in a work-related injury. What can the therapist expect based on this description? A. White, gray, or black appearance, dry surface, edema, eschar, and little pain. B. Blisters, inflammation, and severe pain C. White or gray appearance, wet surface, edema, and severe pain D. Red or white appearance, edema, blisters, and severe pain

A. white, gray, or black appearance, dry surface, edema, eschar, and little pain A full thickness burn is classified as a third-degree burn. The skin layers affected include the epidermis, dermis, and subcutaneous tissue. It is characterized by white, gray, or black appearance, dry surface, edema, eschar, and little pain. The healing requires removal of eschar and grafting. A superficial partial thickness burn is a second-degree burn. The skin layers affected include the epidermis and upper layers of the dermis. It is characterized by blisters, inflammation, and severe pain. Healing typically occurs in 7-21 days. A deep partial thickness burn is also a second-degree burn. The skin layers affected include the epidermis, dermis, nerve endings, hair follicles, and sweat glands. It is characterized by a red or white appearance, edema, blisters, ans severe pain. Healing typically occurs in 21-28 days.

When assessing the strength of the shoulder in an elderly patient, the patient displays the ability to move the limb through the full range of motion but is unable to resist any pressure from the therapist. What grade would this muscle be documented as? A. 2/5 B. 3/5 C. 4/5 D. 5/5

B. 3/5 When assessing for muscle strength using manual muscle testing, the muscle can be graded on a 5-point scale. 0: absent 1: trace muscle contraction 2: Muscle contraction, only moves through partial range or in the gravity eliminated position 3: moves through full range of motion against gravity 4: good strength, able to resist for some therapist testing 5: normal strength, able to resist all therapist testing

Which of the following is NOT a peripheral cause of vertigo? A. Otoscelerosis B. Neuroma C. Meniere's Disease D. BPPV

B. Neuroma causes for vestibular dysregulation can be central or peripheral. The symptoms can be unilateral or bilateral. Neuroma is a central cause of vestibular disorder. It is a tumor or growth that occurs on a nerve. Specifically, a vestibular schwannoma is a benign tumor of the myelin-forming cells on the vestibulocochlear nerve. This can result in vertigo symptoms. Otoscerosis, Meniere's Disease, and BPPV are all peripheral causes of vertigo.

The therapist is assessing the heart rate of a newborn infant and finds it to be 120 beats per minute (bpm), which is normal. What is the normal range for heart rate in pediatric patient? A. 60-100 bpm B. 70-170 bpm C. 40-60 bpm D. 150-210 bpm

B 70-170 bpm In pediatric patients, the normal range is 70-170 bpm, which the newborn average heart rate is 120 bpm. The normal adult heart rate is 70 bpm, with a normal range between 60-100 bpm.

Which would be BEST for building strength in a 5-year-old boy with Down Syndrome? A. Progressive resistive exercise B. Climbing playground equipment C. Instructing in PNF patterns to be performed as home exercise program D. Supine open chain lower extremity strengthening exercises

B Climbing playground equipment Using familiar and fun objects and activities for therapeutic intervention would be the best way to promote strengthening for a young child with Down syndrome. A 5-year-old child will not have the attention span to follow instruction or perform repeated exercises. Additionally, a 5-year-old will not be taught a home exercise program. Instead, a home exercise program would be taught to the parent or caregiver. Additionally, the child with Down syndrome will likely have some degree of cognitive impairment, so a lot of verbal instruction may not be appropriate.

The rate of percussion should be _____ when performing percussion for postural drainage. A. 40+ times per minute B. 100+ times per minute C. 60+ times per minute D. 80+ times per minute

B. 100+ times per minute Percussion is a force rhythmically applied with the therapist's cupped hands to the specific specific area of the chest wall that corresponds to the involved lung segment. The therapist percusses over the area of the thorax with a rate of percussion of 100 + times per minutes. Three to five minutes of percussion per postural drainage position is a typical guideline.

Which medications or medications are typically used to control cerebral edema for an individual with traumatic brain injury? A. Botox B. Baclofen C. Barbiturates D. Oxycodone

C. Barbiturates Barbiturates are used to control cerebral edema and increased intracerebral pressure with severe traumatic brain injury. They are a group of medications that act as a central nervous system depressant. Botox and baclofen are used to treat spasticitiy. Various medications are used for pain medication, one of which may include oxycodone.

Which patient is LEAST appropriate for electrical stimulation modality to modulate low back pain? A. 28-year-old female nurse with chronic low back pain from transferring and lifting patients B. 30-year-old female with acute low back pain related to pregnancy C. 50-year-old male with acute back pain after performing yard work at home D. 75-year-old male with chronic lumbar spinal stenosis

B. 30-year-old female with acute low back pain related to pregnancy Electrical stimulation is considered contraindicated over the abdomen or low back during pregnancy due to potential and unknown effects there may be on the developing baby. It is considered a precaution during labor and delivery. The other patients cases are appropriate for electrical stimulation as long as their past medical history did not include and precautions or contraindications, like metal implants in the area, pacemakers, or impaired sensation.

How long must an athlete participate in strength training before he sees hypertrophy of muscle? A. 1-2 weeks of resistance training B. 6-8 weeks of resistance training C. 4-6 weeks of resistance training D. 2-4 weeks of resistance training

B. 6-8 weeks of resistance training Hypertrophy is an increase in muscle size as a result of resistance training. It can be seen after at least 6-8 weeks of resistance training, Individual muscle fibers are enlarged with more actin and myosin. Additionally, sarcomeres are increased.

In an average adult, what is considered a normal adult blood pressure? A. <160 mm Hg systolic; <100 mm Hg diastolic B. <120 mm Hg systolic; <80 mm Hg diastolic C. <140 mm Hg systolic; <80 mm Hg diastolic D. <130 mm Hg systolic; <90 mm Hg diastolic

B. <120 mm Hg systolic; <80 mm Hg diastolic Blood pressure should be assessed prior to, during, and after intervention. A normal adult blood pressure is <120 mm Hg systolic and <80 mm Hg diastolic. Prehypertension is considered 120-139 mm Hg systolic; 80-89 mm Hg diastolic. Hypertension Stage 1 is 140-159 mm Hg systolic; 90-99 mm Hg diastolic. Hypertension Stage 2 is >160 mm Hg systolic; >100 mm Hg diastolic. It is important note that the majority of patients with hypertension are without symptoms, which is why monitoring by the therapist is so important.

Which of the following patients would be MOST susceptible to catching a head cold within a healthcare clinic? A. A patient who had a recent traumatic brain injury B. A patient undergoing chemotherapy C. A patient with a recent ankle fracture D. A patient in an acute stage of rheumatoid arthritis

B. A patient undergoing chemotherapy A patient receiving chemotherapy is in an immunosuppressed state. The chemotherapy compromises the patients immune system and places him at higher risk for illness. If a clinician has a cold or other illness, he or she should not work with this patient. The other patient examples are not in a state of compromised immune system.

For which patient is the use of a sliding board BEST indicated to perform a transfer from a wheelchair to bed? A. A patient with a grade 3 ulcer over the left side buttocks B. A patient with a level C6 complete spinal cord injury C. A patient with a level C2 complete spinal cord injury D. A patient with stage 2 Parkinson's disease

B. A patient with a level C6 complete spinal cord injury. A sliding board is used for assisted transfers for patients with good sitting balance who can lift most but not all off the weight of the buttocks. A person with a level C6 spinal cord injury would be able to utilize their upper extremities to assist in the slide board transfer. A person with an ulcer over the buttocks would not be appropriate, as the friction of the transfer would further irritate the pressure ulcer. A person with a C2 spinal cord injury would not be appropriate as he would not have the upper extremity strength to perform the transfer. A person with with stage 2 Parkinson's disease is not appropriate, as the patient would still have the functional strength and balance to perform a standing transfer without need for assistance.

For which patient is the use of sliding board BEST indicated to perform a transfer from a wheelchair to bed? A. A patient with a grade 3 ulcer over left side buttocks B. A patient with level C6 complete spinal cord injury C. A patient with a level C2 complete spinal cord injury D. A patient with stage 2 Parkinson's disease

B. A patient with level C6 complete spinal cord injury A sliding board is used for assisted transfers for patients with good sitting balance who can lift most but not all of the weight off the buttocks. A person with a level C6 spinal cord injury would be able to utilize their upper extremities to assist in the slide board transfer. A person with an ulcer over the buttocks would not be appropriate, as the friction of the transfer would further irritate the pressure ulcer. A person with a C2 spinal cord injury would not be appropriate as he would not have the upper extremity strength to perform the transfer. A person with stage 2 Parkinson's disease is not appropriate, as the patient would still have the functional strength and balance to perform a standing transfer without need for assistance.

Which clinical sign is associated with a cerebral stroke? A. Ataxia B. Abnormal synergy patterns C. Hypotonia D. Clinical signs are the same for a cerebral and cerebellar stroke.

B. Abnormal Synergy Patterns Abnormal synergy patterns are associated with cerebral stroke. Ataxia and hypotonia are associated with cerebellar stroke. There is a difference in symptoms depending on the location and extent of the infarct.

Which lymphatic disease is marked by an acute bacterial infection that spreads throughout the lymph system? A. Primary lymphatic disease B. Acute lymphanagitis C. Lymphadenopathy D. Secondary lymphatic disease

B. Acute lymphanagitis Acute lymphanagitis is an acute bacterial infection spreading throughout the lymph system. It is usually caused by streptococcal. Secondary lymphatic disease is acquired and can be due to trauma, radical mastectomy, femoral popliteal bypass, radiation, or disease. Lymphadenopathy is a condition of enlargement of nodes with or without tenderness. Primary lymphatic disease is congenital.

The plan of care includes soft tissue management for the treatment of a patient with de Quervain's tenosynovitis. Which structure should NOT be included? A. Abductor pollicis longus muscle B. Adductor pollicis muscle C. Extensor pollicis brevis muscle D. Extensor pollicis brevis tendon

B. Adductor pollicis muscle The adductor pollicis muscle is not associated with de Quervain's tenosynovitis. This condition is an inflammation of the abductor pollicis longus and extensor pollicis brevis tendons. Soft tissue work on the muscle and tendons of these two structures can help decrease pain and improve soft tissue adhesions.

Patients with multiple sclerosis may experience Lhermitte's sign. What does this entail? A. A drop in blood pressure when moving from sitting to standing B. An electric, shock-like sensation throughout the body produced with flexing the neck C. Rapid weight gain unrelated to caloric intake D. A Rapidly increasing respiratory rate with minimal physical exertion

B. An electric, shock-like sensation throughout the body produced with flexing the neck Lhermitte's sign is an electric shock-like sensation throughout the body produced when the individual flexes the neck. Other common components of presentation include diplopia, dysarthria, paresthesias, hyperpathia, trigeminal neuralgia, nystagmus, spasticity, paresis, ataxia, fatigue, and impaired functional mobility.

A patient with spinal cord injury presents with bilateral loss of motor function, spastic paralysis below the lesion level and bilateral loss of pain and temperature. She has preservation of proprioception, kinesthesia, and vibratory sense. Which clinical syndrome is likely? A. Brown-Sequard Syndrome B. Anterior Cord Syndrome C. Posterior Cord Syndrome D. Central Cord Syndrome

B. Anterior Cord Syndrome Anterior Cord Syndrome is a loss of anterior cord with loss of lateral corticospinal tracts with bilateral loss of motor function, spastic paralysis below level of lesion, as well as loss of spinothalamic tracts with bilateral loss of pain and temperature. There is preservation of dorsal columns with proprioception, kinesthesia, and vibratory sense. Posterior Cord Syndrome is a loss of dorsal colums bilaterally with bilateral loss of proprioception, vibration, pressure, and epicritic sensations. There is a preservation of motor function, pain, and light touch. Brown-Sequard Syndrome is a hemisection of the spinal cord. It typically is caused by penetration wounds, like gunshot or knife wounds, with asymmetrical symptoms. Characteristics include ipsilateral loss of dorsal columns with loss of tactile discrimination, pressure, vibration and proprioception as well as ipsilateral loss of corticospinal tracts with loss of motor function and spastic paralysis below the lesion. There is contralateral loss of spinothalamic tract with loss of pain and temperature below the level of the lesion. At the level of the lesion, there is a bilateral loss of pain and temperature. Central Cord Syndrome is characterized by cavitation of central cord in cervical section, loss of spinothalamic tracts with bilateral loss of pain and temperature, loss of ventral horn with bilateral loss of motor function, preservation of proprioceptive and discriminatory sensation.

The patient is sitting and leaning back on a pillow at a 30-degree angle against the therapist in order to accomplish postural drainage. What is being drained? A. Posterior segments of the upper lobes B. Apical segments of the upper lobes C. Anterior segments of the upper lobes D. Superior segments of the lower lobes

B. Apical segments of the upper lobes In order to drain the apical segments of the upper lobes, the patient leans back at a 30-degree angle against the therapist. The therapist percusses with markedly cupped hands over the area between the clavicle and the top of the scapula on each side. The posterior segments of the upper lobes are drained by having the patient lean forward over a folded pillow at a 30-degree angle. The anterior segments of the upper lobes are drained by having the patient on her back with a pillow under the knees. The superior segments of the lower lobes are drained by placing the patient on her abdomen with two pillows under the hips.

Which arterial disease is marked by chronic, occlusive arterial disease of medium- and large- sized vessels? A. Diabetic angiopathy B. Arteriosclerosis obliterans C. Raynaud's disease D. Thromboangiitis obliterans

B. Arteriosclerosis obliterans Arteriosclerosis obliterans is a chronic, occlusive arterial disease of medium- and large- sized vessels. It is the result of peripheral atherosclerosis and is associated with hypertension and hyperlipidemia. Thromboangiitis obliterans is a chronic, inflammatory vascular occlusive disease of small arteries and also veins. It commonly occurs in young adults who smoke. Patients have paresthesias, pain, cyanotic cold extremities, diminished temperature sensation, fatigue, and risk of ulceration. Diabetic angiopathy is an inappropriate elevation of blood glucose levels and accelerated atherosclerosis. Neuropathy is a major problem, and patients may develop neurotrophic ulcers. Raynaud's disease is episodic spasm of small arteries and arterioles. It is marked by abnormal vasoconstrictor reflex exacerbated by exposure to cold or emotional stress. The tips of fingers develop pallor, cyanosis, numbness, and tingling.

Regarding cardiovascular hemodynamics, which is the term for the volume of blood discharged from the right or left ventricle per minute? A. Stroke volume B. Cardiac output C. Left ventricular end-diastolic pressure D. Ejection fraction

B. Cardiac Output Cardiac output (CO) is the volume of blood discharged from the left or right ventricle per minute, which is approximately 4-6L per minute in a typical adult. Stroke volume is the amount of blood ejected with each myocardial contraction, which is approximately 70 mL. Left ventricular end-diastolic pressure (LVEDP) is the pressure in the left ventricle during diastole. Ejection fraction (EF) is the percentage of blood emptied from the ventricle during systole.

Which is a condition in which there is impaired blood supply to the femoral head, which can lead to symptoms of pain and tenderness to palpation in the groin and hip region? A. Trochanteric bursitis B. Avascular necrosis C. Slipped capital femorial epiphysis D. Piriformis syndrome

B. Avascular Necrosis Avascular necrosis is a condition of impaired blood supply to the femoral head. Hip range of motion is decreased in flexion, internal rotation, and abduction. Symptoms include pain in the groin and thigh, as well as tenderness to palpation at the hip joint. The patient presents with coxalgic gait. Tight iliotibial bands, which commonly occur in running atheletes can result in inflammation of the trochanteric bursa. Treatment includes reduction in pain and inflammation, correcting muscle imbalance and flexibility issues, gait, and running training. Piriformis syndrome is a condition in which the tightness or spasm of the piriformis muscle can result in compression of the sciatic nerve and contribute to sacroiliac dysfunction. Slipped capital femoral epiphysis is a hip deformity related to slippage of the femoral epiphysis. It can be generic or hormonal and lead to weak growth plates. It occurs in children near the onset of puberty age.

A patient is referred to physical therapy for vestibular dysfunction. The patient has episodes of vertigo brought on by stimulation of the vestibular sense organs when rolling over in bed and moving the head in certain positions. The patient has unilateral symptoms and noted nystagmus. What is the likely cause of vertigo? A. Meniere's disease B. BPPV - benign positional paroxysmal vertigo C. CVA with ischemia to the vertebrobasilar system D. Vestibular neuritis

B. BPPV - benign positional paroxysmal vertigo BPPV is a peripheral cause of vertigo in which the vertigo is brought on by a change in head position and stimulation of the vestibular sense organs. Typically, the symptoms occur with rolling in bed, lying down, or looking overhead. The symptoms are unilateral. Canalith Repositioning Technique is performed as treatment for the intervention. Meniere's disease is a peripheral cause of vertigo, which can also accompany hearing loss, tinnitus. It is caused by increased volume of endolymph in the semicircular canals. It does not collow this patient description of symptoms. Otosclerosis is an abnormal growth of bone in the middle ear, which can be another cause of peripherial neuopathy, however, it is not position dependent. A CVA causing ischemia to the vertebrobasilar system is a central cause of vertigo and will not bring about vertigo systoms due to position changes.

What is the MOST common cause of work-related injury and lost of work days for health care workers including physical therapist assistants? A. Dehydration and fatigue B. Back injury C. Carpal tunnel symptoms D. Wrist or hand injury

B. Back injury Back injuries are one of the highest causes of lost work days for health care workers. Patient lifts and patient transfers are the most common causes of back injury amongst health care providers. Proper body mechanics are essential to prevent injury. Additionally, appropriate assistive devices should be utilized when necessary including Hoyer lifts and slide boards.

Which physical therapy intervention for urinary incontinence uses pressure recordings to reinforce active pelvic floor muscle contractions? A. Functional electrical stimulation B. Biofeedback C. Behavioral training D. Progressive strengthening

B. Biofeedback Biofeedback uses pressure recordings to reinforce active contractions and relax the bladder. Functional electrical stimulation is used for muscle reeducation if a patient is unable to initiate active contractions in the pelvic floor. Progressive strengthening uses weighted vaginal cones for home exercises to increase the strength of the pubococcygeus muscles. Behavioral training includes record keeping of voiding and education for lifestyle changes to avoid Valsalva's maneuver and heavy resistance exercises.

Which handling technique is indicated when working with a child who has low tone and the goal is to increase motor recruitment? A. Rocking B. Bouncing C. Swaddling D. Stroking

B. Bouncing Bouncing will assist in increasing motor output and facilitating increased muscle tone. Other handling strategies that will achieve this goal include tapping, brushing, vibrating, quick movements, deep pressure, spinning, and swinging. Swaddling, stroking, and rocking are all handling techniques that would be appropriate when working with a child who has spasticity or dystonia, as they would decrease motor output.

The initial evaluation indicates the patient has impaired sensation over the second and third digits. Which dermatome is this distribution? A. C6 B. C7 C. C8 D. T1

B. C7 C7 dermatome includes digits 2 and 3, the pointer and middle finger. C6 dermatome includes the 1st digit, the thumb. The C8 dermatome includes digits 4 and 5, the ring and little finger. The T1 derma tome includes the anterior aspect of the upper and lower arm.

Which type of disc bulge or herniation is MOST common in the cervical spine? A. Disc bulge/herniation does not occur in the cervical spine B. Central posterior bulge/herniation C. Posteriolateral bulge/herniation D. Anterior bulge/herniation

B. Central posterior bulge/herniation Central posterior bulge/herniation is more commonly observed in the cervical spine but can be seen in the lumbar spine. Posterolateral bulge/herniation is the most commonly observed disc disorder of the lumbar spine due to three structural deficiencies that include posterior disc is narrower in height that anterior disc, posterior longitudinal ligament is not as strong and only centrally located in the lumbar spine, and the posterior lamellae of the annulus are thinner. The anterior bulge/herniation is rare due to structural integrity of the anterior intervertebral disc.

Which condition is typically NOT indicated for manually assisted airway clearance technique? A. Atelectasis B. Chronic bronchitis C. Aspiration D. Cystic fibrosis

B. Chronic bronchitis Conditions that do not respond to manual secretion removal techniques are those without large amounts of drainage. Chronic bronchitis, pneumonia, and post-operative patients typically do not have large amounts of drainage and are not suited of manual secretion removal. Conditions indicated for manually assisted airway clearance are those with increased pulmonary secretions such as cystic fibrosis, bronchiectasis, as well as aspirations. Additionally, conditions that cause mucus plugs such as atelectasis and individuals on mechanical ventilation are appropriate.

Which type of research study is one in which a group of participants with a similar condition is followed for a defined period of time and comparison is made to a matched group that does not have the condition? A. Case report B. Cohort study C. Systematic review D. Randomized control study

B. Cohort study A cohort study is a prospective (forward-in-time) study in which a cohort (group of participants) with a similar condition is followed for a defined amount of time and compared to a matched group without that condition. It is generally a type of observational study. A randomized control trial is an experimental study in which participants are randomly assigned to either an experimental or control group, receiving the variable intervention or placebo. A case report is a type of descriptive research in which only one patient is studied in depth, and a retrospective report is written about the case. A systematic review is a review in which the primary studies arecombined and summarized.

Teaching a patient with balance instability to ambulate with a rolling walker is an example of which type of strategy? A. Disability strategy B. Compensatory strategy C. Remedial strategy D. Motor learning strategy

B. Compensatory strategy A compensatory strategy is one that is used to promote safety and early resumption of functional skills. Safety can be improved by use of appropriate assistive devices, as well as by substitution and alteration of postural strategies. It can also include compensation through adaptation of the environment. Remedial strategies focus on use of the involved body segments (such as a paretic limb in a person with a stroke). Motor learning strategies assist the central nervous system in adaptation for movement control. Disability strategy is not a training strategy for balance.

For a patient with a CVA, which form of intervention is intended to promote forced use of the paretic extremity? A. Functional electrical stimulation B. Constraint-induced movement therapy C. Biofeedback training D. Passive range of motion

B. Constraint-induced movement therapy Constraint-induced movement therapy is intended to increase the functional use of the paretic limb. It is performed by restraining the sound limb and forcing the involved limb to perform functional movement tasks. Functional electrical stimulation is used to stimulate muscle action and reduce spasticity. Biofeedback training aids with increasing firing of paretic muscles and improved muscle control. Passive range of motion can be used to increase awareness of a neglected limb and maintain range of motion.

A patient presents with a lower motor neuron lesion. Which is NOT a structure that would be involved in a condition of a lower motor neuron lesion? A. Cranial Nerve B. Peripheral nerve C. Corticospinal tract D. Spinal root

C. Corticospinal tract. Lower motor neuron lesions include lesions located in the peripheral nervous system. The structures involved may include the anterior horn cells, spinal roots, peripheral nerves, and cranial nerves. Upper motor neuron lesions include lesions located in the central nervous system. The structures involved may include the cortex, brainstem, corticospinal tracts, and spinal cord.

The physical therapist incorporates PNF diagonals into treatment in order to improve shoulder range of motion for bringing the hand to mouth in order to self-feed. Which PNF pattern follows this pattern? A. D1 extension B. D1 flexion C. D2 extension D. D2 flexion

B. D1 flexion D1 flexion for the upper extremity involves flexion-adduction-external rotation of the shoulder. The therapist cues the patient to "close your hand, turn, and pull your arm up across your face." This motion resembles bringing the hand to mouth in order to eat. D2 flexion for upper extremity involves flexion-abduction-external rotation of the shoulder. D1 extension for the upper extremity involves extension-abduction-internal rotation of the shoulder. D2 extension for the upper extremity involves extension-adduction-internal rotation of the shoulder.

Which is NOT a sign or symptom of hypoglycemia? A. Pallor B. Decreased appetite C. Sweating D. Weakness

B. Decreased appetite Decreased appetite is a sign of hyperglycemia, which is abnormally high blood sugar. Signs and symptoms of hypoglycemia, low blood suger, include low glucose < 70 mg/dL, pallor, shakiness, sweating, tachycardia, fainting, dizziness, weakness, fatigue, poor coordination. Excessive hunger is another early sign of hypoglycemia. If the patient is awake, he should be given sugar from juice, candy, or glucose tablets. If the patient is unresponsive, immediate emergency medical treatment is needed.

Which is not an expected gait deviation due to increased flexor tone in the lower extremity in a patient s/p CVA? A. Instability during stance B. Decreased knee flexion during swing C. Excessive knee flexion during stance D. Toe clawing

B. Decreased knee flexion during swing Decreased knee flexion during swing can be caused by increased lower extremity extensor tone (or weakness in hip flexion). Increased flexor tone in the lower extremity is associated with excessive knee flexion during stance, instability in stance ,and toe clawing

Which of the following is considered a contraindication to the use of local superficial thermotherapy? A. Edema B. Deep vein thrombophlebitis C. Pregnancy D. Cardiac insufficiency

B. Deep vein thrombophlebitis Contraindications to the use of local superficial thermotherapy include: deep vein thrombophlebitis, acute traumatic and inflammatory conditions, decreased sensation, malignant tumors, very young and very old patients, as well as impaired cognitive function. Precautions for use of superficial thermotherapy include: cardiac insufficiency, edema, impaired circulation, pregnancy, and areas where topical counterirritants have be recently applied.

Which of the following is NOT a sign of metabolic acidosis? A. Nausea and vomiting B. Depressed respirations C. Muscular twitch D. Weakness

B. Depressed respirations Metabolic acidosis is when there is depletion of bases or an accumulation of acids. The blood pH falls below 7.35. It is caused by diabetes, renal insufficiency, and diarrhea. Signs of metabolic acidosis include hyperventilation, deep respirations, weakness, muscular twitching, malaise, nausea, vomiting, diarrhea, headache, dry skin, poor skin turgor, and coma. Depressed respirations occurs in metabolic alkalosis. Metabolic alkalosis is when there is an increase in bases or a reduction in acids. The blood pH rises above 7.45. It is caused by excess vomiting, excess diuretics, hypokalemia, peptic ulcers, and excessive antacid intake. Signs of metabolic alkalosis include hypoventilation, depressed respirations, dysrhymias, prolonged vomiting, diarrhea, weakness, muscle twitching, irritability, agitation, convulsions, and coma.

Which strategy for therapeutic intervention is LEAST appropriate for an individual with a right hemisphere injury? A. Focus on safety B. Develop communication plan of gestures C. Give frequent feedback D. Avoid environmental clutter

B. Develop communication plan of gestures For a patient with a right hemisphere injury, it is appropriate to use verbal cues. Demonstrations and gestures may be confusing with the presence of visual-perceptual deficits. For a right hemisphere injury, appropriate strategies for intervention include using verbal cues, focusing on slowing down and controlling movements, focus on safety, avoiding environmental clutter, giving frequent feedback and support as well as not under estimating the patient's ability to learn.

What is the primary muscle or muscle group for inspiration? A. Abdominal muscles B. Diaphragm C. Sternocleidomastoid and scalenes D. Trapezius

B. Diaphragm The primary muscle of inspiration is the diaphragm. The diaphragm is made of two hemidiaphragms, each with a central tendon. When the diaphragm is resting, the hemidiaphragms are arched high into the thorax. When the muscle contracts, the central tendon pulls downward and flattens the diaphragm. This results in protrusion of the abdominal wall during inhalation. The scalenes and sternocleidomastoid are considered accessory muscles of inspiration, needed when a person is taking a more rapid or deeper breath or in a diseased state. The trapezius can be considered an accessory muscle of inspiration when the shoulder gridle is fixed, used in forced inspiration. The abdominal muscles are muscles of forced expiration.

What is the frequency of falls in individuals age 65 years and older? A. Each year 50% of persons over the age of 65 fall. B. Each year 30% of persons over the age of 65 fall. C. Each year 10% of persons over the age of 65 fall. D. Each year 15% of persons over the age of 65 fall.

B. Each year 30% of persons over the age of 65 fall. Falls and fall injury are a major health concern for the geriatric population. Each year 30% of persons over the age of 65 fall. Falls result in injury, as well as increased caution and fear of falling, loss of confidence, in independence, reduced activity, and increased risk of recurrent falls.

Heart failure is a condition in which the heart is unable to maintain adequate circulation of the blood to meet the metabolic needs of the body. It is termed CONGESTIVE HEART FAILURE when _____ is present. A. Pain B. Edema C. Nausea D. Shortness of breath

B. Edema Heart failure is a condition in which the heart is not able to maintain adequate blood circulation to meet the body's needs. When edema is present, it is termed congestive heart failure. Right heart failure (backward heart failure) is a condition in which blood is not adequately returned from the systemic circulation to the heart. Left heart failure (forward heart failure) is a condition in which blood is not adequately pumped into systemic circulation.

Which form of massage application describes a technique in which the clinician provides gliding movements of the hands over the surface of the skin with varying light to deep pressure? A. Tapotement B. Effleurage C. Vibration D. Petrissage

B. Effleurage Efflerage is a technique by which the clinician provides gliding movement of the hands over the surface of the skin. It is a form of massage that usually initiates and ends treatment and is used to move from one area of the body to another. It is generally a relaxing stroke. Petrissage is a kneading technique that grasps and lifts the tissues. Tapotement is rapid striking of the surface of the palm, cupped hand, or edge of hand over the patient's skin. Vibration is shaking of the tissue with short, rapid motions.

Regarding cerebral vascular accidents, which etiology is characterized by travelling bits of matter that produce occlusion and infarction in the cerebral arteries? A. Subarachnoid hemorrhage B. Embolus C. Subdural hemorrhage D. Thrombus

B. Embolus An embolus is defined as travelling bits of matter (thrombi, tissue, fat, air, bacteria) that produce occlusin and infarction in the cerebral arteries. Other etiologies include thrombus and hemorrhage. A thrombus is a formation or development of a blood clot within the cerebral arteries or their branches. A hemorrhage is abnormal bleeding as result of rupture of a blood vessel. A subarachnoid hemorrhage is blood between the arachnoid layer and the pia matter. A subdural hemorrhage is blood between the dura matter and the arachnoid layer.

For patients with burns is it important that the therapy team work to prevent or reduce complications of immobilization through positioning and splinting to prevent contractures. Which is the correct position to place a patient with a shoulder burn? A. Emphasize adduction and internal rotation B. Emphasize abduction, flexion, and external rotation C. Emphasize anatomical position D. Emphasize shoulder extension

B. Emphasize abduction, flexion and external rotation The common contracture deformity for a patient with a shoulder burn is adduction and internal rotation. Therefore, when positioning there should be an emphasis on abduction, flexion, and external rotation. The patient is typically positioned with an axillary splint known as an airplane splint. The other options would not sufficiently protect against contracture development. The first and third choices would promote contracture development.

An initial evaluation notes the patient weakness in the hip and knee flexors. Which is NOT a gait deviation that can be expected? A. Insufficient hip and knee flexion B. Equinovarus C. Hip hiking D. Circumduction

B. Equinovarus Equinovarus is the result of spasticity of the posterior tibialis and/or gastrocnemius-soleus. It can also be a developmental abnormality. Circumduction, hip hiking, and insufficient hip and knee flexion are all gait deviations associated with weakness in the hip and knee flexors.

For medicare patients, what is the minimum requirement for completion of reevaluation or reassessments by the the supervising physical therapist? A. Every 14 days B. Every 30 days C. Every 60 days D. Only at the start of care and discharge

B. Every 30 days The supervising physical therapist must complete a reevaluation/reassessment minimally every 30 days for Medicare patients. The reevaluation/reassessment must include progress or regression since last assessment, rationale for continued care, revision of goals, and revision of plan of care. The physical therapist assistant should discuss the progress and goals with the supervising PT. The reassessment must be written by the supervising PT.

For Medicare patients, what is the minimum requirement for completion of reevaluation or reassessments by the supervising physical therapist? A. Every 14 days B. Every 30 days C. Every 60 days D. Only at the start of care and discharge

B. Every 30 days The supervising physical therapist must completes a reevaluation/reassessment minimally every 30 days for Medicare patients. The reevaluation/reassessment must include progress or regression since last assessment, rationale for continued care, revision of goals, and revision of plan of care. The physical therapist assistant should discuss the progress and goals with the supervising PT. The reassessment must be written by the supervising PT.

The physical therapist assistant is working with a patient who is participating in two weeks of lower extremity strengthening prior to undergoing a total knee replacement surgery. The patient asks the PTA to describe the surgery and expected rehabilitation after the surgery, as she is feeling anxious. What is the BEST course of explanation by the PTA? A. Explain the surgical and anesthesia procedures in full detail for the patient B. Explain how patients typically respond to a total knee replacement and describe the typical rehabilitation protocol with exercise progression C. Refer the patient to the overseeing physical therapist who is an orthopedic clinical specialist D. Explain that this is outside your scope of practice, as you are not the surgeon and refer the patient to discuss this with the orthopedic physician

B. Explain how patients typically respond to a total knee replacement and describe the typical rehabilitation protocol with exercise progression The physical therapist assistant should provide appropriate information about the rehabilitation protocol after the total knee replacement. PTAs should have clear knowledge of the protocol following orthopedic surgeries. This explantion would not require the input from a clinical specialist and is within the scope of practice of the PTA. Information about the details of surgery and anesthesia are within the scope of practice of the orthopedic surgeon and anesthesiologist. Rehabilitation is within the scope of practice of the therapy team including the PTA.

Transcutaneous electrical nerve stimulation is designed to provide afferent stimulation with the goal of achieving what? A. Muscle contraction B. Pain management C. Joint range of motion D. Wound healing

B. Pain management Transcutaneous electrical nerve stimulation (TENS) is designed for pain management. It provides afferent stimulation and modulates pain through activation of central inhibition of pain transmission. This is done with the gate theory. Muscle contraction is achieved with high-voltage pulsed current, IFC, and Russian current.Joint range of motion can be achieved with IFC. Wound healing is achieved through high voltage pulsed current.

Which type of speech and communication disorder is characterized by the ability to comprehend spoken but significant difficulty producing speech and when speech does occur, words are restricted, interrupted and awkward? A. Receptive aphasia B. Expressive aphasia C. Global aphasia D. Dysarthria

B. Expressive aphasia Expressive aphasia occurs with damage to Broca's area. It results in speech that requires a great deal of effort to produce and when speech occurs, the words are typically restricted, interrupted, and awkward. The individual is able to comprehend the spoken words from others. Receptive aphasia, also known as Wernicke's or fluent aphasia, occurs from damage to Wernicke's area. Spontaneous speech is preserved and fluent, however, auditory comprehension is severely impaired. Global aphasia is a combination of expressive and receptive aphasia resulting in major difficulty with comprehension and expression of language. Dysarthria is characterized by a limited ability to control movements of the jaw, tongue, and respiratory structures needed for speech control. The patient can understand spoken language and use the correct words, but spoken words are hard to understand.

Which of the following is NOT a contraindication to aquatic exercise intervention? A. Bowel or bladder incontinence B. Fear of water C. Water and airborne infections D. Severe epilepsy

B. Fear of water Contraindications to aquatic therapy include: bowel or bladder incontinence, severe kidney disease, severe epilepsy, severe cardiac disease, severe peripheral vascular disease, large open wounds, bleeding or hemorrhage, and water and airborne infections. Precautions to aquatic therapy include: fear of water, inability to swim, patients with heat intolerances, and use of waterproof dressings over small wounds.

The physical therapist assistant is treating a patient 8 months after suffering third-degree burns to the upper extremity with a treatment focus on improving an elbow flexion contracture. What is the MOST appropriate soft tissue technique to release soft tissue adhesions? A. Effleurage B. Friction C. Vibration D. Petrissage

B. Friction This patient has high risk for developing hypertrophic scarring. Shortening of the tissue caused the elbow contracture to form. Therefore, massage needs to improve how supple the tissue is in order to improve the range of motion. Friction massage is used to break up scar tissue. It is performed with compression of the tissue using small circular or long stroking movements. Petrissage and effleurage are relaxation forms of massage that address muscle tightness. Vibration is primarily used for chest wall mobilization to excrete secretions.

During which phase of healing in a musculoskeletal injury are modalities LEAST LIKELY to be used in treatment? A. Modalities are always indicated B. Functional restoration phase C. Subacute phase D. Acute (inflammatory) phase

B. Functional restoration phase During this phase, patients should not have any pain and should be returning to previous levels of activity. Modalities are unlikely because the patient is without pain and should be without inflammation. During the acute and subacute phases, modalities are indicated for pain relief, reduction of inflammation, and to promote tissue healing.

The physical therapist assistant is delivering joint mobilization intervention. The therapist delivers 30 seconds of continuous large amplitude oscillation before the point of tissue resistance. Which grade of joint mobilization is being performed? A. Grade 1 B. Grade 2 C. Grade 3 D. Grade 4

B. Grade 2 Grade 2 joint mobilization is being performed in this example. The grades of joint mobilizations are determined by dosages of oscillation techniques. Grade 1: small amplitude oscillation at the beginning of the range Grade 2: large amplitude oscillation before the point of tissue resistance Grade 3: large amplitude oscillation that pushes into tissue resistance Grade 4: small amplitude oscillationnear the end of passive range Grade 5: small amplitude, high-velocity manipulation past the end of passive range

The physical therapist assistant is teaching a patient to ascend a curb using a large base quad cane. The patient has weakness noted in the right quadriceps, which limits walking stability. Which is the appropriate sequencing for ascending the curb? A. Hold cane in right side, lead with left lower extremity B. Hold cane in left side, lead with left lower extremity C. Hold cane in left side, lead with right lower extremity D. How cane in right side, lead with right lower extremity

B. Hold cane in left side, lead with left lower extremity When using a device in one hand, the device is held opposite the weaker lower extremity. Therefore , the quad cane should be held with the left hand. When ascending a curb or stair, the patient should lead with the stronger lower extremity in order to best have the strength to lift the weight of the body onto the curb or stair. In this case, the patient's stronger leg is the left lower extremity, so he should lead with this leg when ascending.

A PTA is working to reduce discomfort related to trigger point symptoms in a patient's neck. Where is the trigger point for the sternocledomastoid MOST LIKELY palpated? A. Just superior to the superior angle of the vertebral border of the scapula B. Just inferior to the mastoid process C. Medial to the superior portion of the acromion process. D. Medial to the inferior angle of the vertebral border of the scapula

B. Just inferior to the mastoid process. The sternocleidomastoid has a trigger point located just inferior to the mastoid process. The trapezius has trigger points medial to the acromion process on the superior portion as well as at the inferior angle of the vertebral border of the scapula. The levator scapula has a trigger point just superior to the superior angle of the scapula.

Which airway clearance technique is MOST appropriate for patient with collapsible airways such as chronic obstructive disease? A. Assisted cough B. Huffing C. Tracheal stimulation D. Endotracheal suctioning

B. Huffing Huffing is an airway clearance technique more effective in patients with collapsible airways such as COPD. It prevents the high intra-thoracic pressure that causes premature airway closure. The technique is performed by taking a moderate or deep breath and forcibly expelling the air, saying "ha ha," such as breathing out to fog the lens of eyeglasses. Assisted cough is performed when the patient lacks abdominal strength to generate an effective cough such as a patient with a spinal cord injury. The therapist's hands become the force behind the patient's exhaled air. Tracheal stimulation is used with patients who are not able to cough on command such as infants or individuals who have had a stroke or traumatic brain injury. The therapist finger is placed above the suprasternal notch, and a quick inward and downward pressure on the trachea elicits a cough. Endotracheal suctioning is used only when other airway clearance techniques are not adequate. A catheter is fed into an artificial airway, oral airway, or the nares to the carina. Suction is applied intermittently (10-15 seconds).

Which types of wound dressings are characterized as adhesive wafers containing absorptive particles that interact with wound fluid to form a gelatinous mass over the wound bed and are indicated for protection of partial thickness wounds? A. Foams B. Hydrocolloids C. Hydrogels D. Alginates

B. Hydrocolloids Hydrocolloids are adhesive wafers that contain absorptive particles that interact with wound fluid to form a gelatinous mass over the wound bed. They can be occlusive or semi-occlusive. They are indicated for protection of partial thickness wounds and wounds with mild exudate, maintaining a moist wound environment. Hydrogels are water or glycerine-based gels insoluble in water. They are available as solid sheets, amorphous gels, or impregnated gauze. They are used for partial and full thickness wounds and those with necrosis and slough. They are soothing and cooling to the patient and rehydrate dry wound beds. Alginates are soft, absorbent, nonwoven dressings derived from seaweed and often used for wounds with moderate to large amounts of exudate. Alginates react with wound exudate to form a viscous hydrophillic gel mass over the wound area. They are able to absorb up to 20 times their weight in drainage. Foams are semiperimeable membranes that are either hydrophillic or hydrophobic. They vary in thickness, absorptive capacity, and adhesive properties. They are indicated for partial and full thickness wounds with minimal or moderate exudate.

Which physical property of water describes the circumferential water pressure exerted on an immersed body part and is responsible for decreased edema with aquatic therapy? A. Density B. Hydrostatic pressure C. Buoyancy D. Cohesion

B. Hydrostatic pressure Hydrostatic pressure is the circumferential water pressure exerted on an immersed body or body part. A pressure gradient is established between the surface waters and the deeper waters due to the increased density in the deeper water. Increased pressure reduces edema and enhances peripheral circulation. Cohesion is the tendency of water molecules to adhere to each other. The resistance that is felt while moving through water is due to cohesion. Force is required to separate the water molecules. This can create resistance for exercise. Buoyancy is the upward force of water on an immersed or partially immersed body or body part. It is equal to the weight of the water that it displaces. The buoyancy creates an apparent decrease in body weight and joint unloading, which allows for easier movement in the water. Density is the mass per unit volume of a substance. The density of water is proportional to its depth. The deeper the water, the greater the density. This can be incorporated to challenge or regress exercise.

The physical therapist assistant assesses her patient's blood pressure and finds it to be 145/92 mm Hg. This is considered: A. Normal B. Hypertension Stage 1 C. Hypertension Stage 2 D. Prehypertension

B. Hypertension Stage 1 Hypertension Stage 1 is 140-159 mm Hg systolic; 90-99 mm Hg diastolic Hypertension Stage 2 is >160 mm Hg systolic; >100 mm Hg diastolic. Prehypertension is 120-139 mm Hg systolic; 80-89 mm Hg diastolic. Blood pressure should be assessed prior to, during, and after intervention. A normal adult blood pressure is <120 mm Hg systolic and <80 mm Hg diastolic. It is important to note that the majority of patients with hypertension are without symptoms, which is why monitoring by the therapist is so important. It is not within the scope of the PTA's practice to diagnose a patient with hypertension, but the physician should be informed of the blood pressure.

Which of the following is NOT a sign of metabolic alkalosis? A. Irritability B. Hyperventilation C. Muscle twitching D. Dysrythmia

B. Hyperventilation Metabolic alkalosis is when there is an increase in bases or a reduction in acids. The blood pH rises above 7.45. It is caused by excess vomiting, excess diuretics, hypokalemia, peptic ulcers, and excessive antacid intake. Signs of metabolic alkalosis include hypoventilation, depressed respirations, dysrhymias, prolonged vomiting, diarrhea, weakness, muscle twitching, irritability, agitation, convulsions, and coma. Hyperventilation occurs in metabolic acidosis. Metabolic acidosis is when there is a depletion of bases or an accumulation of acids. The blood pH falls below 7.35. It is caused by diabetes, renal insufficiency, and diarrhea. Signs of metabolic acidosis include hyperventilation, deep respirations, weakness, muscular twitching, malaise, nausea, vomiting, diarrhea, headache, dry skin, poor skin turgor, and coma.

The therapist is performing sit-to-stand training for a patient with right-sided hemiplegia who requires maximum assist to stand. Where should the therapist be positioned for safe guarding? A. To the right side and slightly behind the patient B. In front of the patient and slightly to the right side C. To the left side and slightly behind the patient D. In front of the patient and slightly to the left side

B. In front of the patient and slightly to the right side During sit-to-stand transfers, the therapist should stand to one side and slightly behind the patient in most cases. However, for patients who require increased levels of assistance, the therapist should stand in front of the patient. Additionally, the therapist should stand to the involved side. This way, the therapist can block the limb from buckling.

Which nerve root level is being tested with the quadriceps deep tendon reflex? A. C7-C8 B. L2-L4 C. L5-S3 D. S1-S2

B. L2-L4 The quadriceps deep tendon reflex testing examines the function of the nerve root level L2-L4. A deep tendon reflex is tested with the muscle positioned at mid-range. The tendon is tapped with a reflex hammer. S1-S2 is tested with the Achilles deep tendon reflex. The L5-S3 nerve root level is tested with the hamstring deep tendon reflex. The C7-C8 nerve roots are tested with the triceps deep tendon reflex.

When performing lymphatic drainage exercises for an upper extremity affected with lymphedema, which is NOT appropriate? A. Perform with affected extremity elevated, supine position B. Include resisted strengthening with 5-10 pound weights to build strength C. Begin with active range of motion of the shoulder and then isometric exercises of the shoulder D. Progress to distal musculature (elbow, forearm, wrist, hand) after shoulder

B. Include resisted strengthening with 5-10 pound weights to build strength. Pumping exercises for lymphatic drainage are designed to follow a sequence to move lymph away from congested areas. During this exercise intervention, resisted exercises with weights are not appropriate and will hinder lymphatic flow. Exercise routines for an upper extremity are as follows: The patient should perform in supine with the limb elevated. Initiate exercise with diaphragmatic breathing followed by active range of motion exercises for the shoulder. Then include isometric exercises for the shoulder, followed by active range and isometric exercises for the elbow, forearm, wrist, and fingers. Then, the patient should rest with the limb elevated for 20-30 minutes.

When working with a child who has Down Syndrome with low muscle tone, the goal of intervention is to: A. Decrease motor output B. Increase motor output C. Provide appropriate wheelchair positioning D. Promote relaxtion

B. Increase motor output For a child with low tone, the goal is to increase motor output. This can be done through handling techniques of tapping, brushing, vibrating, quick movements, deep pressure, spinning, swinging, and bouncing. For a child with high tone, the goal is to decrease motor output. Relaxation would not be correct as this would further decrease muscle tone. Children with Down syndrome are typically ambulatory and rarely utilize wheelchairs, so that answer is incorrect.

Which of the following is NOT an example of a goal and indication for superficial thermotherapy? A. Accelerate tissue healing B. Increase muscle tone C. Increase connective tissue extensibility D. Modulate pain

B. Increase muscle tone The goals and indications for superficial thermotherapy include: modulation of pain, increase connective tissue extensibility, reduce soft-tissue swelling, accelerating rate of tissue healing, reduced joint restriction, and reduced muscle spasm.

A physical therapist assistant is working with a child with severe extensor tone. Which type of wheelchair positioning would be helpful in reducing the extensor tone? A. Place the seat in a position to increase anterior pelvic tilt B. Increase the flexion of the hips and knees to greater than 90 degrees when seated in chair C. Decrease the flexion of hips and knees to less than 90 degrees when seated in chair D. Add a hip adduction wedge to the wheelchair seat

B. Increase the flexion of the hips and knees to greater than 90 degrees when seated in chair. If the chair is positioned in a manner to increase hip and knee flexion to greater than 90 degrees, then flexion flexion will be facilitated. This will help reduce tone and keep the child seated in the wheelchair, reducing the extensor posturing. Hip flexion less than 90 degrees will facilitate hip extension, as will positioning in an anterior tilt. Extensor posturing typically occurs with adductor spasticity, so this should not be promoted. An abductor wedge is indicated in this case.

Which of the following is NOT a physiologic response of the body system to local cold application? A. Decreased muscle spindle discharge, resulting in decreased spasticity B. Increased extensibility of collagen tissue and decreased tissue viscosity C. Decreased capillary permeability D. Increased pain threshold due to interruption of pain-spasm cycle.

B. Increased extensibility of collagen tissue and decreased tissue viscosity. Local cold application causes an increase in joint stiffness due to decreased extensibility of collagen tissue and increased tissue viscosity. The other answers are true physiological responses to cold application over a local region of the body.

The physics related to heat transmission for utilization of the superficial thermotherapy modalities include conduction, convection, and radiation. Which of the following is an example of heat transmission through radiation? A. Hot packs B. infrared lamp C. paraffin D. fluidotherapy

B. Infrared Lamp Radiation is the transfer of heat from a warmer object to a cooler object by means of transmission of electromagnetic energy without heating of an intervening medium. Infrared waves are absorbed by the cooler body. Fluidotherapy is an example of heat transfer through convection. Hot packs and paraffin are examples of heat transfer through conduction.

Which type of resistance exercise uses static muscle contraction with no change in muscle length? A. Eccentric B. Isometric C. Isotonic D. Isokinetic

B. Isometric Isometric resistance exercise is performed as a static muscle contraction, with no muscle length change. The patient holds a muscle contraction for at least 6 seconds. This is beneficial to strengthen a muscle at a specific point in the ROM. Isotonic exercises use dynamic muscle contraction. There is constant (free weights) or variable (machine) resistance and uses concentric and eccentric contractions. The patient moves a resistance through the available range of motion and speed can be variable. Isokinetic resistance exercise is dynamic and has a speed control for muscle shortening and lengthening. Resistance is accommodating and variable. The individual uses isokinetic equipment, and it provides maximum resistance at all points of the range of motion. Eccentric resistance exercise is performed with dynamic lengthening contraction. This is useful to prepare muscles for functional activities and for muscles unable to perform concentric contraction. Maximum eccentric contraction produces more force than maximal concentric contraction.

Which of the following conditions or presentations is joint mobilization NOT indicated? A. Joint pain B. Joint effusion C. Muscle spasm and guarding D. Joint hypomobility

B. Joint effusion Joint effusion is a contraindication for joint mobilization intervention. Other contraindications include joint hypomobility and inflammation. Precautions include malignancy, unhealed fracturem bone disease, hypermobility in adjacent joints, systemic connective tissue disease, and if the person is on blood-thinning medication. Idications fr joint mobilization include pain, muscle spasm and guarding, joint hypomobility, and functional limitation in joint range of motion.

Regarding principles of progression, which principle is NOT an accurate progression of intervention? A. Stable surface progressing to a moveable surface B. Large range of motion progressing to a small range of motion C. Lower resistance progressing to a higher resistance D. Slow movements progressing to fast movements

B. Large range of motion progressing to a small range of motion In order to progress an intervention, the therapist or patient will move from a small range of motion to a large range of motion. Moving from a large range of motion to a small range of motion is a regression. The other answer choices are appropriate progressions.

A prosthesis that is too long can cause all of the following gait deviations in a patient with an above-knee amputation EXCEPT for which deviation? A. Abduction in stance phase B. Lateral bending in stance phase C. Circumduction in swing phase D. Vaulting in swing phase

B. Lateral bending in stance phase Lateral bending in stance phase is caused by a short prosthesis (not long). Additionally, it can be caused by inadequate lateral wall adduction or a sharp and high medial wall. A long prosthesis can cause abduction in stance phase, circumduction in swing phase, vaulting and hip hike in swing phase.

A prosthesis that is too long can cause all of he following gait deviations in a patient with an above-knee amputation EXCEPT for which deviation? A. Abduction in stance phase B. Lateral bending in stance phase C. Circumduction in swing phase D. Vaulting in swing phase

B. Lateral bending in stance phase Lateral bending in stance phase is caused by a short prosthesis, not a long one. Additionally, it can be caused by in adequate lateral wall adduction or a sharp and high medial wall. A long prosthesis can cause abduction in stance phase, circumduction in swing phase, vaulting and hip hike in swing phase.

A patient has complaints of elbow pain during his activities, working in construction. He uses a hammer and other tools that require a strong grip with wrist extension. What is the MOST LIKELY condition? A. Posterior elbow dislocation B. Lateral epicondylitis C. Medial epicondylitis D. Elbow contracture

B. Lateral epicondylitis Lateral epicondylitis is also known as tennis elbow. It is a chronic inflammation of the extensor tendons of the forearm, most typically the extensor carpis radialis brevis (ECRB) tendon at its proximal attachment to the lateral epicondyle of the humerus. The onset is gradual and results from repetitive motions with wrist extension or strong gripping with wrist extension. This motion overloads the ECRB. Medial epicondylitis is an inflammation of the pronator teres and flexor carpi radialis tendons at their attachment to the medial epicondyle of the humerus. It is a result of chronic overuse in sports, such as baseball pitching, golfing, swimming, or in occupations that require a strong hand grip with pronation of the forearm. Elbow contractures result in loss of motion due to soft tissue restriction or loose body in the elbow. Elbow dislocations are typically the result of elbow hyperextension from a fall on the outstretched arm.

The physical therapy evaluation identifies a Trendelenburg gait on the left side. The plan of care is to include functional strengthening exercises. Which would be MOST appropriate to target the weakened musculature and address this gait deviation? A. Straight leg raises B. Lateral step-ups C. Wall squats D.Prone hip extension

B. Lateral step-ups A Trendelenburg gait pattern is due to weakness in the hip abductors, particularly the gluteus medius. Lateral step-ups will strengthen the gluteus medius muscle bilaterally. Straight leg raises target the hip flexors. Wall squats target the quadriceps and gluteus maximus muscles. Prone hip extension targets the gluteus maximus.

Which pediatric disorder is characterized by degeneration of the femoral head due to a disturbance in blood supply? A. Arthogryposis Multiplex Congenita B. Legg-Calve'-Perthes Disease C. Slipped Capital Femoral Epiphysis D. Osteogensis Imperfecta

B. Legg-Calve'-Perthes Disease Legg-Calve'-Perthes Disease is a condition of self-limiting degeneration of the femoral head due to disturbance in blood supply. It most commonly occurs in boys 4-7 years of age. It is less frequent in girls with onset at older ages. Osteogenesis Imperfecta is characterized by fragile bones that break easily and often for no reason. It is due to an abnormality in the collagen gene causing problems with the amount and quality of collagen in the body. There are 4 types with varying degrees of severity. Arthrogryposis Multiplex Congenita is a non-progressive neuromuscular disorder that causes multiple joint contractures at birth. There is variability in clinical picture but typically include severe joint contractures and absence of muscle development. Children will utilize splinting, surgery, and assistive devices to compensate and correct for deformities. Slipped Capital Femoral Epiphysis is a hip deformity related to the slippage of the femoral epiphysis. It occurs to children who are tall, with delayed skeletal maturity, obese, and near the onset of puberty. It is more common in boys than in girls. In mild cases, surgery is required to pin the hip and prevent further slippage. Severe cases require varus osteotomy.

Which of the following is a sign associated with right-sided heart failure? A. Cough B. Nausea C. Fatigue D. Orthopnea

B. Nausea Signs associated with right-sided heart failure include nausea, anorexia, weight gain, ascites, and right upper quadrant pain. Right heart failure (backward heart failure) is a condition in which the blood is not adequately returned from the systemic circulation to the heart. It is due to failure the right ventricle and increased pulmonary artery pressures. Signs associated with left-sided heart failure include fatigue, cough, shortness of breath, dyspnea on exertion, ortopnea, paroxysmal nocturnal dyspnea, and diaphoresis. Left heart failure (forward heart failure) is a condition in which blood is not adequately pumped into systemic circulation. It is due to the inability of the left ventricle to pump and increases in ventricular end-diastolic pressure and left atrial pressures.

Which is NOT a structure involved in an upper motor neuron lesion condition? A. Cortex B. Brainstem C. Cranial nerves D. Corticospinal tracts

C. Cranial nerves Upper motor neuron lesions include lesions located in the central nervous system. The structures involved may include the cortex, brainstem, corticospinal tracts, and spinal cord. Lower motor neuron lesions include lesions located in the peripheral nervous system. The structures involved may include the anterior horn cells, spinal roots, peripheral nerves, and cranial nerves.

Which type of cancer primarily affects the blood? A. Sarcoma B. Leukemia C. Lymphoma D. Carcinoma

B. Leukemia Leukemia affects the blood with unrestrained growth of leukocytes. A sarcoma is a malignant tumor that orignates in connective and mesodermal tissues (such as the muscle, bone, or fat). A carcinoma is a malignant tumor that originates from epithelial tissues (such as the skin, stomach, colon, breast, and rectum. A lymphoma is a malignant tumor that affects the lymphatic system (such as Hodgkin's disease and lymphatic leukemia).

The physical therapist assistant is treating a child with Down syndrome, and the plan of care includes promoting increased muscle tone. Which is an effective intervention to increase muscle tone, especially in the trunk? A. Perform passive stretching to the lower extremities B. Lightly bounce the child over an exercise ball C. Give a slow and rhythmic massage with effleurage technique D. Assist the child in performing headstands

B. Lightly bounce the child over an exercise ball Children with Down syndrome tend to have hypotonia, or low muscle tone throughout. Bouncing exercises and positioning over an unstable surface, such as an exercise ball, will help to stimulate muscle activity and increased muscle tone. Slow massage will promote relaxation. Passive stretching is typically not indicated for children with Down syndrome due to hypermobility. Headstands are not appropriate due to risk of atlantoaxial dislocation.

Which healthcare-associated infection is NOT characterized by diarrhea, fever, and abdominal pains? A. C diff - Clostridium Difficile B. MRSA - methicillin-resistant staphylococcus aureus C. Rotavirus D. Norovirus

B. MRSA - methicillin-resistant staphylococcus aureus MRSA is a bacteria found on healthy human skin and poses an infection concern for those with weakened immune systems. It can appear as a pustule or boil at the site of visible skin trauma and areas covered by hair. Transmission occurs via contact. People who work and visit healthcare facilities are susceptible. Norovirus, rotavirus, and C diff are all infections associated with signs and symptoms of fever, diarrhea, and abdominal pains.

Which hand deformity is caused from rupture or avulsion of the extensor tendon at its insertion into the distal phalanx of a digit? A. Boutonniere deformity B. Mallet finger C. Swan neck deformity D. Ape hand defrmity

B. Mallet Finger A mallet finger deformity is due to a rupture or avulsion of the extensor tendon at its insertion into the distal phalanx of a digit. The observed deformity is flexion of the distal interphalangeal joint. A swan neck deformity results from contracture of intrinsic muscles with dorsal subluxation of lateral extensor tendons. The observed deformity is flexion of the metacarpophalangeal and distal interphalageal joint with extension of the proximal proximal interphalangeal joint. A boutonniere deformity results from the rupture of the central tendonious slip of the extensor hood. The observed deformity is extension of the metacarpophalangeal and distal interphalangeal joint with flexion of the proximal interphalangeal joint. An ape hand deformity is observed as thenar muscle wasting, with the first digit moving dorsally in line with the second digit. It is a result of median nerve dysfunction.

A patient has nerve entrapment resulting in aching pain with weakness in the forearm muscles and paresthesia into digits 1-4. What nerve is entrapped? A. Tibial nerve B. Median nerve C. Ulnar nerve D. Radial nerve

B. Median nerve Median nerve entrapment occurs within the pronator teres muscle and under the superficial head of the flexor digitorum superficialis. It occurs with repetitive gripping activities and sporting activities. Clinical signs include aching pain with weakness in forearm muscles and paresthesia into digits 1-4. Ulnar nerve entrapment is caused by direct trauma to the cubital tunnel, traction due to laxity in the medial elbow, compression due to thickening of retinaculum, hypertrophy of flexor carpi ulnaris, or DJD of the cubital tunnel. Symptoms include median elbow pain and paresthesia into digits 4 and 5. Radial nerve entrapment occurs in the distal branches within the radial tunnel due to overhead activities and throwing. Clinical signs include lateral elbow pain, pain over supinator muscle, and paresthesia into the posterior aspect of the forearm. The tibial nerve is not a viable option, as the tibial nerve innervates the lower leg, and this question describes symptoms in the upper extremity.

Which degenerative disease of the central nervous system is characterized by demyelinating lesions that impair neural transmission and rapid nerve fatigue? A. Amyotrophic lateral sclerosis B. Multiple sclerosis C. Meningitis D. Parkinson's disease

B. Multiple sclerosis Multiple sclerosis is a chronic, progressive demyelinating disease of the central nervous system. The etiology is unknown but most likely an autoimmune response to a virus. The demyelinating plaques impair neural transmission. Patients display paresthesia, diplopia, fatigue, emotional disturbance, paresis, impaired coordination, and impaired motor function. Amyotrophic lateral sclerosis is a progressive upper and lower motor neuron disease in which the muscle fibers atrophy from peripheral nerve involvement. Degeneration and scarring of the motor neurons occur in the lateral aspect of the spinal cord, brainsten, and cerebral cortex. Parkinson's disease is a degenerative condition of the basal ganglia with dopamine deficiency. Meningitis is an infectious disease that impacts the meninges of the spinal cord or brain characterized by viral or bacterial infection.

Which traumatic nerve injury is characterized by cutting of the nerve with severance of all structures and complete loss of function? A. Entrapment B. Neurotmesis C. Neurapraxia D. Axontmesis

B. Neurotmesis Neurotmesis is a class-3 traumatic nerve injury. It is characterized by cutting of the nerve with severance of all structures and complete loss of function. Re-innervation fails fails without surgical intervention. Axontmesis is a class-2 traumatic nerve injury. It is an injury to nerve interrupting the axon and causing loss of function and degeneration distal to the lesion such as a crush injury. Regeneration is possible. Neurapraxia is a class-1 traumatic nerve injury. It is an injury to the nerve that causes a transient loss of function like a compression injury. Nerve dysfunction may be rapidly reversed or persist a few weeks. Entrapment is a condition of pressure on a nerve where it passes over a bony prominence or restricted opening. It results in motor and sensory disturbances in the area of nerve distribution.

Which is NOT a typical characteristic of an arterial ulcer? A. Well demarcated lesion B. No pain C. Pale base without granulation D. Located over toes, dorsum of foot, lateral malleolus

B. No pain Characteristics of an arterial ulcer include: location over the toes, dorsum of foot, and lateral maleous, often full thickness depth, edges that are well demarcated, a pale base without granulation, decreased pulses and often significant pain. An ulcer without pain is characteristic of a venous ulcer or diabetic ulcer.

Which lobe of the cerebrum is responsible for processing visual information? A. Frontal B. Occipital C. Parietal D. Temporal

B. Occipital The occipital region of the cerebrum functions for visual association; it processes visual information and applies meaning. If a patient suffers injury to this region of the brain, it is import to know which systems may be affected. The temporal lobe is responsible for auditory functions, comprehension of spoken word, long-term memory, perception of touch, perception of pain, and temperature. The frontal lobe functions for the primary motor cortex for voluntary movement, motor components of speech, cognition, judgment, attention, abstract thinking, and emotional control.

Which cranial nerve, involved in vision or eye function, does NOT function as a motor nerve? A. Oculomotor B. Optic C. Abducens D. Trochlear

B. Optic Cranial nerve II is the optic nerve. It is a sensory nerve responsible for the function of visual acuity. The oculomotor (cranial nerve III), trochlear (cranial nerve IV), and abducens (cranial nerve VI) are all motor cranial nerves. They are responsible for eye movement

Where would the stethoscope be placed to listen for the heart sounds of the tricuspid valve? A. Over the second left intercostal space at the sternal border B. Over the fourth left intercostal space at the sternal border C. Over the fifth left intercostal space at the midclavicular line D. Over the second right intercostal space at the sternal border

B. Over the fourth left intercostal space at the sternal border The auscultation landmark for the tricuspid valve is found over the fourth left intercostal space at the sternal border. The auscultation landmark for the aortic valve is found over the second right intercostal space at the sternal border. The auscultation landmark for the pulmonic valve is at the second left intercostal space at the the sternal border. The auscultation landmark for the mitral valve is found over the fifth left intercostal space at the midclavicular line.

For patients who have an acute condition, with pain at rest, an appropriate education intervention includes teaching PRICE. What is the correct explanation or the PRICE protocol? A. P- protect, R- rest, I- inspect, C- cold compress, E- elevation B. P- protect, R- rest, I- ice, C- compression, E- elevation C. P- protect, R- range of motion, I- inspect, C- compression, E- electrical stimulation D. P- prevent, R- range of motion, I- ice, C- compression, E- elevation

B. P- protect, R- rest, I- ice, C- compression, E- elevation In order to control for inflammatory processes, the physical therapist assistant should educate his patients in PRICE. PRICE stands for: P- protect, R- rest, I- ice, C- compression, E- elevation These strategies will help to reduce edema and prevent effusion as well as further injury.

In what position should a patient be placed to promote draining of the anterior segments of the upper lobes? A. Patient sits, leaning over a folded pillow at 30 degrees B. Patient is flat on back with pillow under knees C. Patient is on abdomen in a head-down position with a pillow under the hips D. Patient is prone on abdomen with two pillows under hips

B. Patient is flat on back with pillow under knees In order to promote postural drainage of the anterior segments of the upper lobes, the patient is on her back with a pillow under her knees. The therapist performs percussion and vibrations between the clavicle and nipple on each side. To clear the superior segments of the lower lobes, the patient is prone on the abdomen with two pillows under her hips. To clear the posterior basal segments of the lower lobes, the patient is on the abdomen in a head-down position with a pillow under her hips.

Which of following interventions is BEST indicated for a patient immediately following a cesarean delivery? A. Abdominal curl-ups B. Pelvic floor strengthening exercises C. Cardiovascular training D. Straight leg raises

B. Pelvic floor strengthening exercises For patients after pregnancy and cesarean section, the patient will benefit from pelvic floor exercises to strengthen the musculature of the pelvic floor. After pregnancy, the patient will likely have decreased muscle tone and decreased muscle strength and control. This will help to prevent or reverse incontinence. Cardiovascular training is too intense right after cesarean delivery. Abdominal curl-ups and straight leg raises are contraindicated due to the surgery and location of the incision. The patient will require time for healing.

The therapist suspects a patient has carpal tunnel syndrome. Which special test would identify structures involved in this condition? A. Finkelstein's test B. Phalen's test C. Bunnel-Littler test D. Adson's test

B. Phalen's test Both Tinel's sign and Phalen's test identify carpal tunnel compression of the medial nerve. Phalen's test is performed by having the patient hold his wrist in complete forced flexion by pushing the dorsal surfaces of the hands together. The patient holds this position for up to one minute. In this position, there is increased pressure on the carpal tunnel, pinching the median nerve. Positive signs include burning, tingling, or numbness into the first four digits. Adson's Test identifies pathology of the structures that pass through the thoracic inlet. It is used when identifying individuals with thoracic outlet syndrome. The test is positive when the examiner finds the loss of the radial pulse in the arm when the patient rotates the head to the ipsilateral side with neck extension, following deep inspiration. Finkelstein's test is used to indentify de Quervain's tenosynovitis. It is performed by grasping the patient's thumb and sharply moving the hand into ulnar deviation. A positive test will cause pain over the distal radius and forearm.

Which type of imaging study is BEST indicated to identify the integrity of bony tissues? A. MRI B. Plain film radiograph C. Arthrography D. Diagnostic ultrasound

B. Plain film radiograph The plain film radiograph, also known as an X-ray, is used to demonstrate the integrity of bony tissues. The X-ray beams pass through the tissues resulting in varying shades of gray on the film, depending on the tissue they pass through. The more dense the structure, such as a bone, the more white it appears. One negative side effect is the patient's exposure to radiation. MRI uses magnetic fields and offers excellent visualization of tissue anatomy. Arthrography is an invasive technique that injects water-soluble dye into an area while observing under radiograph. It is used to identify abnormalities within joints such as tendon ruptures. Diagnostic ultrasound uses high-frequency sound waves and assesses soft tissue dysfunctions.

The physical therapist assistant is treating a patient with a stroke who displays synergistic patterns. Which of the following is NOT part of a flexion synergy? A. Flexion of the knee B. Plantarflexion and inversion at the ankle C. Retraction and elevation of the scapula D. Abduction and external rotation at the shoulder

B. Plantarflexion and inversion at the ankle Plantarflexion and inversion at the ankle are part of an extension synergy. Flexion synergy patterns include: retraction and elevation of the scapula, shoulder abduction and external rotation, elbow flexion, forearm supination, wrist and finger flexion, hip flexion/abduction/external rotation, knee flexion, ankle dorsiflexion, and eversion and toe dorsiflexion. Extension synergy patterns include: protraction of scapula, shoulder adduction and internal rotation, elbow extension, forearm pronation, wrist and finger flexion, hip extension and adduction, knee extension, ankle plantarflexion and inversion, toe plantarflexion.

Which specific exercise regimen incorporates isotonic exercises that combine speed, strength, and functional activities typically reserved for later stages of rehabilitation to achieve high athletic performance? A. Progressive resistive exercise B. Plyometric training C. Circuit weight training D. Brief, repetitive isometric exercise

B. Plyometric training Plyometric training is an isotonic exercise regimen that combines speed, strength, and functional activities. It is used in later stages of rehabilitation to achieve high level of performance. It can include activities such as jumping on and off platforms, high-stepping, quick lunges, etc. Circuit weight training uses a sequence of exercise for total body conditioning. There is typically a rest period of 30 seconds to 1 minute between each exercise. Weight machines or free weights are used for resistance. Brief, repetitive isometric exercise incorporates up to 20 maximum contractions, which are held for 5-6 seconds and performed daily. A rest of 20 seconds between contractions is recommended to reduce the likelihood of increased blood pressure. Strength gains occur over six weeks. An example is Kegel exercises for the pelvic floor. Progressive resistive exercise (PRE) uses the repetition maximum (RM), which is the greatest amount of weight that a muscle can move through the range of motion a specific number of times (depending on protocol). For example, DeLorme uses 10 RM for baseline. In this protocol, exercise begins with 10 repetitions at 50% RM, followed by 10 repetitions at 75% RM, and 10 repetitions at 100% RM%.

In order to make a sharp turn to the right, how should the patient be instructed to propel the wheelchair? A. Push the right arm forward on drive wheel while pulling the left wheel backward B. Push the left arm forward on the drive wheel while pulling the right wheel backward C. Push the left arm forward on the drive wheel while engaging the right side brake D. Push the right arm forward on the drive wheel while engaging the right side brake

B. Push the left arm forward on the drive wheel while pulling the right wheel backward In order to make a wheelchair turn, the patient must push harder with one hand than the other. In order to make a sharp turn in a wheelchair, the patient should pull one wheel backward while pushing the other forward. In order to make a sharp right turn, the patient will push the left arm forward while pulling the right wheel backward.

Which specific exercise regimen incorporates isotonic exercises that combine speed, strength, and functional activities typically reserved for later stages of rehabilitation to achieve high athletic performance? A. Progressive resistive exercise B. Plyometrics C. Circuit weight training D. Brief, repetitive isometric exercise

B. Plyometrics Plyometric training is an isotonic exercise regimen that combines speed, strength, and functional activities. It is used in later stages of rehabilitation to achieve high level of performance. It can include activities such as jumping on and off platforms, high-stepping, quick lunges, etc. Circuit weight training uses a sequence of exercise for total body conditioning. There is typically a rest period of 30 seconds to 1 minute between each exercise. Weight machines and free weights are used for resistance. Brief, repetitive isometric exercise incorporates up to 20 maximum contractions, which are held for 5-6 seconds and performed daily. A rest of 20 seconds between contractions is recommended to reduce the likelihood of increased blood pressure. Strength gains occur over six weeks. An example is Kegel exercises for the pelvic floor. Progressive resistive exercise (PRE) uses the repetition maximum (RM), which is the greatest amount of weight that a muscle can move through the range of motion a specific number of times, depending on the protocol. For example, DeLorme uses 10 RM forbaseline. In this protocol, exercise begins with 10 repetitions at 50% RM, followed by 10 repetitions at 75% RM, and 10 repetitions at 100% RM.

What type of waveform characteristic is present in Russian and interferential current? A. Monophasic wave B. Polyphasic wave C. Galvanic wave D. Biphasic wave

B. Polyphasic wave Polyphasic wave characteristics of electrical stimulation describe waves that have biphasic current modified to produce three or more phases in a single pulse. Polyphasic waveform in medium frequency can be Russian or interferential current. Biphasic wave has a bidirectional flow of charged particles with one-half the cycle above the baseline and the second phase below the baseline. Monophasic waves have a unidirectional flow of charged particles and have either a positive or negative charge. Galvanic current is a type of monophasic waveform used for muscle stimulation and wound healing.

Physical therapy begins within a few days of a flexor tendon repair to the fingers in order to: A. It does not begin until 2-4 weeks following sergery to allow for healing B. Preserve tendon gliding C. Promote circulation D. Initiate strengthening

B. Preserve tendon gliding Early passive and active assistive exercises are performed within a few days following flexor tendon repair in order to promote collagen remodeling, prevent adhesions, and promote free tendon gliding. Early strengthening is not appropriate. Though circulation will be promoted with hand mobility, it is not the primary goal. Waiting several weeks to begin therapy will lead to adhesion formationand restriction to motion.

Which type of multiple sclerosis (MS) is characterized by progression of symptoms from onset with no or minimal episodes of plateau? A. Progressive-relapsing MS B. Primary-progressive MS C. Relapsing-remitting MS D. Secondary-progressive MS

B. Primary-progressive In regards to MS symptoms, a relapse is worsening of symptoms. A remission is a period of decreased or no symptoms. Primary-progressive MS progresses from onset with no or limited plateaus. These patients may experience temporary, minor improvements along their course of progression. Relapsing-remitting MS is characterized by cycles of exacerbation/remission with long periods of stability. These patients may have minimal long-term impairment. Secondary-progressive MS begins as relapsing-remitting MS and turns into progressive MS. Progressive-relapsing follows a progressive course with periodic acute relapses. In this type, there is a loss of function and progressive worsening with each exacerbation.

Which type of multiple sclerosis (MS) is characterized by progression of symptoms from onset with no or minimal episodes of plataeu? A. Progressive-relapsing MS B. Primary-progressive MS C. Relapsing-remitting MS D. Secondary-progressive MS

B. Primary-progressive MS In regards to MS symptoms, a relapse is a worsening of symptoms. A remission is a period of decreased or no symptoms. Primary-progressive MS progresses from onset with no (or limited) plateaus. These patients may experience temporary, minor improvements along their course of progression. Relapsing-remitting MS is characterized by cycles of exacerbation/remission with long periods of stability. These patients may have minimal long-term impairment. Secondary-progressive MS begins as relapsing-remitting MS and turns into progressive MS. Progressive-relapsing MS follows a progressive course with periodic acute relapses. In this type, there is a loss of function and progressive worsening with each exacerbation.

Which type of stretching is performed with serial casting? A. Manual static passive stretching B. Prolonged mechanical stretching C. Facilitated stretching D. Dynamic stretching

B. Prolonged mechanical stretching Prolonged mechanical stretching is a low-intensity external force (up to 10% body weight) applied over a prolonged period of time. This is done by positioning the patient with weighted pulleys, traction, or serial casting. Prolonged stretch may mean 20 minutes to 10 hours. Facilitated stretching refers to techniques in which the patient reflexively relaxes muscle to be elongated during stretching techniques. It utilizes active inhibition principles of PNF. Examples include hold-relax, hold-relax-active contraction, and contract--relax-active contraction. Dynamic stretching is an active or passive stretch performed actively by moving joints (either in a body weighted or eliminated position) through the available range of motion. Gradually, the individual increases the range and speed of movement as the musculature warms up. This motion is controlled and performed at a moderate speed and is appropriate to perform before sports activity. Manual static passive stretching stretches the structures beyond the free range of motion to elongate the muscles and soft tissues beyond their resting length. The stretch is typically held 30 seconds to 2 minutes and repeated several times. Intensity and duration depend on patient tolerance.

Which intervention is MOST appropriate to improve the breathing of a patient with chronic emphysema? A. Forceful inspiration B. Pursed-lip expiration C.Place patient in supine for easiest respiration D. Emphasis on utilization of accessory muscles

B. Pursed-lip expiration A patient with chronic emphysema has difficulty with air trapping. This causes over inflated lungs. Exhalation should be the focus of treatment to improve breathing. Therefore, pursed lip exhalation should be the intervention to empty the lungs. Seated positioning is best, leaning forward onto forearms (not supine). Utilizing accessory muscles and forceful inspiration will promote air trapping and should be avoided.

A patient with hemiplegia due to CVA requires strengthening of his shoulder girdle, trunk, and hips. Which position allows weight bearing through the shoulder girdle and hips and facilitates initial control of the lower trunk an hips? A. Supine on elbows B. Quadruped C. Prone on elbows D. Longsitting withhands in the lap

B. Quadruped Placing the patient in quadruped allows for weight bearing through the femur into the hip joint as well as through the humerus into the shoulder joint. It allows for recruitment of lower trunk, hip complex, and shoulder girdle muscle activity to work on stabilization. The other options do not allow weight bearing through the hips nor do they promote trunk stabilization.

Which of the following is NOT a characteristic of fibromyalgia syndrome? A. Muscle pain B. Recurring sore throat C. Multiple trigger points D. Persistent fatigue

B. Recurring sore throat Recurring or frequent sore throat is a symptom common in chronic fatigue syndrome, not fibromyalgia. Common characteristics of fibromyalgia include muscle pain, generalized aching, persistent fatigue, sleep disturbances, morning stiffness, trigger points, cold intolerance, headaches, irritable bowel, impaired memory, difficulty with concentration, restless legs, and commonly anxiety/depression.

The physical therapist assistant is working with a patient who has Alzheimer's disease. The patient becomes frustrated and agitated in performing a skill during treatment. What is the BEST strategy for the PTA to take? A. Increase the amount of verbal cues and instruction provided B. Redirect the patient to a less difficult task C. Discontinue the treatment session D. Increase the repetitions of the task and length of task practice

B. Redirect the patient to a less difficult task When working with a patient who has Alzheimer's disease, it is best to keep tasks simple and direct, aiming to minimize frustration, confusion, and agitation. Increasing the verbal cues and instruction will likely increase agitation. Increasing repetitions and time spent on task will also increase frustration and is not going to simplify the task task at all. There is no need to end the session at this point just because the task is frustrating.

Regarding ventilation, what is the volume of gas that remains in the lungs after expiratory reserve volume has been exhaled? A. Total lung capacity B. Residual volume C. Tidal volume D. Vital capacity

B. Residual volume Residual volume is the volume of gas that remains in the lungs after expiratory reserve volume has been exhaled. Tidal volume is the volume of gas inhaled or exhaled during a normal resting breath. Vital capacity is the amount of air under volitional control (inspiratory reserve volume + tidal volume + expiratory reserve volume). Total lung capacity is the total amount of air contained within the thorax during a maximum inspiratory effort (inspiratory reserve volume + tidal volume + expiratory reserve volume + residual volume).

What term describes the intensity of the current, with a long duration stimulus, required to produce a minimum muscle contraction? A. Chronaxie B. Rheobase C. Amperage D. Motor point

B. Rheobase Rheobase is the intensity of the current, with a long duration stimulus, required to produce a minimum muscle contraction. Rheobase is a measure of membrane excitability. Chronaxie is the pulse duration of the stimulus at twice the rheobase intensity. A motor point is an area of greatest excitability on the surface of the skin in which a small amount of current generates a muscle response. Amperage is the rate of flow of electrons.

Which primitive reflex is responsible for turning head toward a touch and opening the mouth when the cheek is stimulated with a brush or stroke? A. Moro B. Rooting C. Suck-swallow D. Babinski

B. Rooting The rooting reflex is an important reflex for early feeding, especially for breastfeeding. When the side of the baby's cheek is stroked or brushed, the head turns toward the touch and the mouth opens. The suck-swallow reflex is important for early feeding. When an object touches the roof of the mouth, the baby begins sucking. The Moro reflex is elicited with rapid neck extension. Upon rapid neck extension, the baby's arms abduct and extend followed quickly by arms flexing in as if grasping for a parent and crying. The Babinski reflex is elicited with stroking the lateral plantar surface of the foot. Upon stimulation, the greater toe extends and the other toes fan out.

Ultrasound treatment utilizes which form of energy to create thermal effects in the soft tissue structures? A. Magnetic energy B. Sound wave C. Electric current D. Infrared energy

B. Sound wave Ultrasound utilizes mechanical energy produced by sound waves at frequencies between 85 KHz and 3 MHz and delivered at intensities between 1 w/cm^2 and 3 w/cm^2. The sound waves are absorbed by the body tissues and changed into thermal energy.

In regards to arthrokinematics (joint accessory motion), what is the movement between the joint surfaces of the radial head with the humerus? A. Roll B. Spin C. Glide D. Extension

B. Spin Spin consist of a rotation of the movable components of a joint. An example is between the radial head and the humerus. Glide (or slide) consists of a pure translatory motion of one surface gliding on another surface. An example of glide is the proximal phalanx at the head of a metacarpal bone. Roll consists of one joint surface rolling on another such as a tire rolling over a road. An example of rolling is between the femoral and tibial articular surfaces in the knee. Extension is a term to describe osteokinematics, which is a physiological motion between two bones.

When performing soft tissue mobilization on a patient's neck, the therapist notices a flat red area on the patient's skin, which she is concerned may need further evaluation by a dermatologist. Which form of malignant tumor is characterized by a flat, red area with poorly defined margins commonly occurring in sun-exposed areas? A. Basal cell carcinoma B. Squamous cell carcinoma C. Kaposi's sarcoma D. Malignant melanoma

B. Squamous cell carcinoma Squamous cell carcinomas have a poorly defined margin. They present as flat red area, ulcer, or nodule. Squamous cell carcinoma grows more quickly and is common on sun-exposed areas including the face, neck, and back of hands. It can metastasize. A basal cell carcinoma is a low-growing epithelia basal cell tumor. It is characterized by a raised ivory colored patch with rolled border and indented center. It rarely matastasized and is common on the face in fair-skinned people. It is is associated with prolonged sun exposure. Malignant melanoma is a tumor that arises from melanocytes. Superficial spreading melanoma is the most common type. It is characterized by uneven edges, irregular borders, variations in color, larger than 6 mm, and typically elevated presentation. Kaposi's sarcoma is a lesion of endothelial cell origin with red or dark purple macules that progress to nodules or ulcers. it is associated with itching and pain. it is common in lower extremities.

The PTA is giving a community educational presentation on properly lifting strategies to avoid low back injury. Which of the following is an appropriate statement about safe lifting? A. Keep the object as far away from the body as possible B. Squat down to the object before trying to lift it from the floor C. Keep the low back flexed during the lift D. Increase the curve of the low back while lifting in order to promote stability

B. Squat down to the object before trying to lift it from the floor It is important to emphasize good body mechanics when lifting. Individuals should stoop down to the object first, allowing them to lift using the leg musculature. The back should be kept in neutral alignment with the trunk muscles engaged. Individuals should also be sure to keep the load close to the body and not away from the body.

A physical therapist assistant is working with a patient with a history of several shoulder dislocations and is referred for therapy for joint hyper mobility. What is the BEST treatment intervention? A. Passive range of motion B. Stabilization exercises and corrective taping C. End range shoulder strengthening exercises D. Manual joint mobilization, grade 2

B. Stabilization exercises and corrective taping The proper intervention for a joint that has excess movement is to stabilize the joint through functional stabilization strengthening exercises and corrective taping. No joint mobilization is appropriate for the hypermobile joint. Passive range of motion is not indicated for joint hypermobility. Additionally, end range strengthening is not appropriate as this could cause the shoulder to dislocated.

When teaching a caregiver to assist in a sit-to-stand transfer using a gait belt, the physical therapist assistant should teach the caregiver to achieve a lower center of gravity by: A. Keeping the hips flexed and the knees extended B. Standing with a wide base of support C. Bending at the waist D. Standing with a narrow base of support

B. Standing with a wide base of support In order to lower the center of gravity and improve balance and stability when assisting for a transfer, it is important to stand with a wide base of support. A narrow base of support raises the center of gravity. It is not appropriate to assist a transfer when bending at the waist and keeping the knees extended. This will promote injury and is an example of poor body mechanics.

The physical therapist assistant is testing a patient's sensation by placing an object in the patient's hand and asking him to identify the object. Which sensory test is being performed? A. Graphesthesia B. Stereognosis C. Barognosis D. Kinesthesia

B. Stereognosis Stereognosis is when an pbject is placed in a patient's hand, and the patients should be able to identify the object by touch only. Graphesthesia is a test of cortical sensation. It is done when the therapist draws a letter in the palm of the patient's hand. The patient should be able to identify the letter. Barognosis is when similar objects with varying weights are placed in the patient's hand. The patient should be able to identify differences in weight. Kinesthesis is tested by moving the patient's limb in various directions, and the patient identifies the direction of the motion the limb is being moved.

Which special test of the spine is used to identify spondylolisthesis? A. Slump test B. Stork standing test C. Anterior shear test D. Quadrant test

B. Stork standing test The stork standing test identifies spondylolisthesis. It is performed by having the patient stand on one foot and with back in extension. A positive test is pain in the back. The quadrant test identifies compression of neural structures at the intervertebral foramen and facet dysfunction. The slump test identifies dysfunction of the neurological structures that supply the lower limbs. The anterior shear test assesses the integrity of the integrity of the upper cervical spine ligaments and capsules.

The patient displays poor initiation of swing with diminished hip flexion during gait. Which intervention would be MOST appropriate for strengthening? A. Prone hip extension B. Straight leg raise C. Long arc quad D. Supine hip abduction

B. Straight leg raise Straight leg raises target the hip flexors. The described gait pattern is due to weakness in the hip flexors, so this exercise would be appropriate. Supine hip abduction targets the hip abductors. Long arc quads target the knee extensors. Prone hip extension targets the gluteus maximus (hip extensor).

Which of the following is NOT one of the four steps in the utilization of evidenced-based practice? A. A systematic literature review is conducted, and best evidence is collected B. The clinician utilizes all resources available via web search in order to make a comprehensive evaluation C. A clinical problem is identified, and an answerable research question is formulated D. The research evidenced is synthesized and applied to clinical ractice

B. The clinician utilizes all resources available via web search in order to make a comprehensive evaluation The clinician must avoid taking any and all resources of information that are found via an Internet search. The clinician must first analyze the quality of the sources and the level of evidence for each study or report. Only qualified sources and qualified studies should be considered. The four steps in evidence-based practice are: 1. A clinical problem is identified, and an answerable research question is formulated 2.A systematic literature review is conducted, and best evidence is collected. 3. The research evidenced is summarized and critically analyzed. 4. The research evidence is synthesized and applied to clinical practice.

The medical history reveals that a patient has sustained a first-degree burn to his back. What layer or layers are affected in a first-degree burn? A. Epidermis, dermis, nerve endings, hair follicles, and sweat glands B. The epidermis C. Epidermis, dermis, and subcutaneous tissue D. Epidermis and upper layers of dermis

B. The epidermis In a first-degree, superficial burn, the layer affected is the epidermis. In a second-degree, superficial partial thickness burn, the layers affected include the epidermis and upper layers of the dermis. In a second-degree, deep partial thickness burn, the layers affected include the epidermis, dermis, nerve endings, hair follicles, and seat glands. In a third-degree, full thickness burn, the layers affected include the epidermis, dermis, and subcutaneous tissue. In a fourth-degree, subdermal burn wound, the layers affected include the epidermis, dermis, subcutaneous tissue, muscle, and bone.

Which is NOT an appropriate point of control for gait training when guarding? A. The gait belt B. The forearm C. The shoulder D. The pelvis

B. The forearm The therapist should not grasp the patient's clothing or the patient's extremities for gaurding. Key points of control for guarding in gait training include the shoulder, bilateral or unilateral pelvis, and gait belt.

A patient has a CVA with right-sided weakness. If the PTA is assisting the patient to perform a stand pivot transfer from her wheelchair to the bed, how should the wheelchair be aligned? A. The wheelchair is placed perpendicular to the surface on the patients uninvolved side B. The wheelchair is placed parallel to the surface on the patient's uninvolved side C. The wheelchair is placed parallel to the surface on the patient's involved side D. The wheelchair is placed perpendicular to the surface on the patient's involved side.

B. The wheelchair is placed parellel to the surface on the patient's uninvolved side. An assisted stand pivot transfer is used for patients who cannot stand independently but can bear some weight in the lower extremities. If there is one-sided involvement, he wheelchair is placed parallel to the transfer surface on the stronger side. The transfer is completed to the strong side.

In regards to normal ventilation, which term describe the volume of gas inhaled or exhaled during a normal resting breath? A. Residual volume B. Tidal volume C. Inspiratory reserve volume D. Expiratory reserve volume

B. Tidal volume Tidal volume is the volume of gas inhaled or exhaled during a normal resting breath. Inspiratory reserve volume is the volume of gas that can be inhaled beyond a normal resting tidal inhalation. Expiratory reserve volume is the volume of gas that can be exhaled beyond a normal resting tidal exhalation. Residual volume is the volume of gas that remains in the lungs after expiratory reserve volume has been exhaled.

Which surgical repair requires the patient to utilize a wedge to prevent hip adduction when in bed? A. Total knee arthroplasty B. Total hip arthroplasty C. ACL reconstruction D. Open reduction internal fixation

B. Total hip arthroplasty After a total hip arthroplasty, the patient should avoid the position of hip flexion past 90 degrees with adduction and internal rotation. The patient should be positioned in bed with a wedge to prevent adduction of the lower extremity. The other conditions d not require a wedge to be used to prevent hip adduction.

Which wound dressings are BEST indicated for a patient with a stage 1 pressure ulcer? A. Foams B. Transparent films C. Hydrocolloids D. Hydrogels

B. Transparent films A transparent film is indicated for stage 1 and 2 pressure ulcers. It is a clear, adhesive, semipermeable membrane dressing. It is permeable to atmospheric oxygen and moisture vapor but impermeable to water, bacteria, and environmental contaminants. The other options are indicated for deeper wounds.

Which type of rigidity is common for a patient with traumatic brain injury in which the lesion is above the brainstem? A. UE and LE and extension B. UE in flexion, and LE in extension C. UE extension, and LE in flexion D. UE and LE in flexion

B. UE in flexion, and LE in extension Decorticate rigidity is a form of rigidity common with lesions above the brainstem. It is characterized by UE in flexion and LE in extension. Decerebrate rigidity is a form of rigidity common with lesions of the brainstem. The UE and LE are held in extension. The other options are not typical forms of rigidity.

The physical therapist assistant is working with a patient that has class II congestive heart failure. What is the maximal METS for physical activity for this patient? A. Up to 1.5 METS B. Up to 4.5 METS C. Up to 6.5 METS D. Up to 3 METS

B. Up to 4.5 METS Patients with congestive heart failure (CHF) demonstrate significant ventricular dysfunction, decrease cardiac output, and low functional capacities. CHF can be classified as class I-IV. METS are metabolic equivalents of oxygen consumption. Class I: Mild CHF; no limitation in physical activity; up to 6.5 METS. Patients are cmfortable at rest; ordinary activity does not cause undue fatigue, dyspnea, or angina pain. Class III: Moderate CHF; marked by limitation of physical activity; up to 3 METS. PAtients are comfortable at rest, but less than ordinary activities cause fatigue, palpitation, dyspnea, or angina pain. Class IV: Severe CHF; patients are unable to carry out any physical activity, 1.5 METS, without discomfort. Symptoms of ischemia, dyspnea, and angina pain are present even at rest and symptoms increase with exercise.

The PTA is working with a patient who suffered a myocardial infarction 36 hours ago. Immediately following a myocardial infarction, which exercise is not appropriate? A. Exercise that increases heart rate 10-15 beats over resting heart rate B. Upper extremity strengthening with a 7-pound weight C. Paced walking D. Short duration exercise sessions

B. Upper extremity strengthening with a 7-pound weight The other options are appropriate, provided the patient is stable for at least 24 hours. They are low intensity and will not place unnecessary stress on the heart

Which strategy for therapeutic intervention is LEAST appropriate for an individual with a left hemisphere injury? A. Give frequent feedback and support B. Use verbal cues C. Assess level of understanding D. Develop communication plan of words and gestures

B. Use verbal cues The use of verbal cues is appropriate for an individual with a right hemisphere injury. For an individual with a left hemisphere injury, verbal cues may be difficult to understand due to impairment with speech and language as well as communication difficulties. It is hard for those patients to process verbal cues. For a left hemisphere injury, appropriate strategies for intervention include developing a communication plan of word and gestures, assessing the level of understanding, giving frequent feedback and support as well as not underestimating the patient's ability to learn.

A patient with multiple sclerosis has persistent diplopia, which leaves her nauseated, imbalanced, and restricts her tolerance for walking. Which intervention is MOST appropriate to assist in mobility? A. Turn lights off in the room B. Utilize a patch over one eye C. Have the patient close her eyes D. Use a soft cervical collar

B. Utilize a patch over one eye Diplopia is a condition of double vision. It can be managed by patching over one eye. Many patients alternate the eye patch on a schedule. Patching will negatively impact depth perception. Closing the eyes and turning off the lights with not diminish the diplopia. A cervical collar to limit neck motion will not address diplopia either.

A patient presents with impaired plantar flexion range of motion and weakness in the plantar flexor muscle group. Which task will be MOST difficult? A. Walking over even surface B. Walking down a ramp C. Ascending stairs D. Descending stairs

B. Walking down a ramp With limited plantar flexion range of motion and weakness in the gastrocnemius/soleus, walking down a decline will be difficult. When walking down a ramp, plantar flexion range is needed for the person to achieve foot flat position. Additionally, the plantar flexor muscle group is activated to hold back the tibia. Walking up a ramp requires sufficient dorsiflexion range of motion and strength in the anterior tibialis. Descending stairs relies more on proper sequencing and strength of the hip and knee. Walking over even surfaces would be the easiest of these tasks.

A patient presents with impaired dorsiflexion range of motion and weakness in the anterior tibialis muscle. Which task will be MOST difficult? A. Walking over even ground B. Walking up a ramp C. Walking down a ramp D. Descending stairs

B. Walking up a ramp With limited dorsiflexion range of motion and weakness in the anterior tibialis, walking up an incline will be difficult. Walking up a ramp requires sufficient dorsiflexion range of motion and strength in the anterior tibialis. When walking down a ramp, plantar flexion range is needed for the person to achieve foot flat posiion. Additionally, the plantar flexor muscle group is activated to hold back the tibia. Descending stairs relies more on proper sequencing and strength of the hip and knee. Walking over even surfaces would be the easiest of all the tasks.

A geriatric patient suffered a Colle's fracture from a fall. Upon further examination, she was diagnosed with osteoporosis. Which exercise would be BEST indicated for this patient once the fracture is well healed? A. Treadmill walking and lower extremity resistive band exercises B. Wall push-ups and upper extremity resistive band exercises C. Passive stretching to the upper and lower extremities D.Open chain upper extremity active range of motion in all shoulder planes

B. Wall push-ups and upper extremity resistive band exercises Colle's fracture is a fracture of the distal radius, which is a common injury in individuals who have osteoporosis. In order to stimulate bone mineral density, exercises that incorporate weight-bearing positions and resistive exercises. Wall push-ups incorporate weight bearing through the upper extremities and resistive band exercises utilize resistance strengthening. Treadmill walking and lower extremity resistance will not address the bone mineral density in her injured upper extremity. Passive stretching and active range of motion do not accomplish weight bearing either.

When applying electrodes to a patient, the space between the active and dispersive electrodes should be how large? A. They should always be one inch apart B. At least twice the diameter of the active electrode C. At least the diameter of the active electrode D. Half the diameter of the active electrode

C. At least the diameter of the active electrode The electrodes should be placed so that the space between the active and dispersive electrodes is at least the diameter of the active electrode. The greater the space between the electrodes, the lesser the current density in the superficial tissue, which reduces the risk of skin irritation and burns. The electrodes should be moved closer together if deep penetration causes unwanted contraction of undesired muscles.

A patient s/p CVA presents with left-sided sensory and motor loss to her lower extremity with no involvement in her upper extremity. She has confusion and slow, delayed movement. Additionally, she has behavioral changes and confusion. Which CVA syndrome is likely? A. Posterior cerebral artery syndrome B. anterior cerebral artery syndrome C. middle cerebral artery syndrome D. vertebrobasilar artery syndrome

B. anterior cerebral artery syndrome CVA syndrome are characterized by the impacted cerebral artery. The anterior cerebral artery supplies the medial part of the frontal and parietal lobe, basal ganglia, and corpus collosum. With anterior cerebral artery syndrome, the patient can experience contralateral sensory and motor loss with lower extremities affected more than upper extremities. The patient can have mental impairment, urinary incontinence, apraxia, slow movement, and behavioral changes. Middle cerebral artery syndrome is characterized by contralateral sensory motor loss, with face and upper extremity affected more than lower extremity. These patients can have perceptual deficits, homonymous hemianopsia, Broca's aphasia, Wernicke's aphasia, and global aphasia. The middle cerebral artery supplies the lateral cerebral hemispheres, including the frontal, temporal, and parietal lobes. The posterior cerebral artery supplies the occipital lobe, medial and inferior temporal lobe, thalmus, and midbrain. With posterior cerebral artery syndrome, the patient can experience contralateral sensory and motor loss, homonymous hemianopsia, visual agnosia, oculomotor nerve palsy, involuntary movement, Pusher syndrome, and thalmic pain syndrome. The vertebrobasilar artery supplies the medulla, the pons, and cerebellum. Patients with vertebrobasilar artery syndrome have a wide variety of symptoms, ipsilaterally, and contralaterally. They can have cranial nerve involvement, ataxiam Wallenburg's syndrome, and locked-in syndrome.

Which of the following conditions is considered a precaution to electrical stimulation (not a contraindication)? A. pacemaker B. impaired mentation C. active bleeding D. superficial metal implants

B. impaired mentation Contraindications: pacemakers, unstable arrhythmias, suspected epilepsy or seizure disorders, transcerebrally or transthoracically in the presence of active bleeding or infection, superficial metal implants, pharyngeal or laryngeal muscles, and over the carotid sinus, thrombosis, eyes, thoracic region, phrenic nerve, urinary bladder simulators, and abdomen and low back during pregnancy. Precautions: cardiac disease, impaired mentation, areas of impaired sensation, malignant tumors, open wounds, hypotension, hypertension, excessive edema, bleeding disorders, menstrating uterus, and pregnancy during labor and delivery.

As a result of multiple sclerosis, a patient is displaying impaired dynamic standing balance deficiencies. Which intervention is BEST indicated for this patient? A. Standing with narrow base of support in tandem stance B. Reaching for cones outside base of support in standing C. Standing with eyes closed on even surface D. Alternating isometrics in standing

B.Reaching for cones outside base of support in standing Reaching for objects outside the patient's base of support will help increase the patient's dynamic balance. The other answer choices involve activities that challenge a patient's static balance.

What is the BEST age to screen boys for scoliosis? A. 8 to 10 years of age B. 14 to 16 years of age C. 11 to 13 years of age D. 16 to 18 years of age

C. 11 to 13 years of age Scoliosis screening should be performed right before the pubescent growth spurt. During this growth spurt, the spinal curvature can greatly increase. For boys, this screening should occur between 11 to 13 years of age. Boys reach puberty later than girls, so girls are screen earlier. Girls are screened between 9 and 11 years of age.

When measuring for a cane or crutches, what is the desirable degree of the elbow flexion for a patient? A. 40-50 degrees elbow flexion B. 5-10 degrees elbow flexion C. 20-30 degrees elbow flexion D. No flexion, the elbow should be extended

C. 20-30 degrees elbow flexion The assistive device should be measured to fit the patient's height and arm length. In checking for accuracy, measure that the patient achieved 20-30 degrees of elbow flexion when standing and holding the assistive device appropriately.

Regarding environmental considerations for wheelchair accessibility, what is the minimum recommended clear width for doorways and halls? A. 28 inches minimum and ideally 32 inches B. 24 inches minimum and ideally 26 inches C. 32 inches minimum and ideally 36 inches D. 42 inches minimum and ideally 46 inches

C. 32 inches minimum and ideally 36 inches The minimum clear width for doorways and halls is 32 inches. Ideally, the clear width is 36 inches. The standard adult wheelchair width is 24 to 26 inches from rim to rim. There must be additional clearance to allow space for the patient's hands for propulsion. In order to perform a 90-degree turn in space, the standard wheelchair will require at minimum 36 inches of space.

When initially instructing a patient in ambulation, what is the BEST gait pattern to use for a patient using a prothesis for the first time after undergoing a transtibial amputation? A. 2-point gait B. swing through gait C. 4-point gait D. 3-point gait

C. 4-point gait When first instructing a patient to walk with a new prosthesis, the 4-point gait will be the most stable gait pattern as there is always three points of contact with the ground and it has a slower cadence. The other three options will not be as stable as a 4-point gait.

What age is MOST effective when screening girls for scoliosis? A. 12 to 14 years of age B. 7 to 9 years of age C. 9 to 11 years of age D. 16 to 18 years of age

C. 9 to 11 years of age The most effective time to screen girls for scoliosis is when they are between 9 to 11 years of age. This is just before the pubescent growth spurt when the curve of scoliosis can greatly increase. The best time to screen boys of scoliosis is between 11 to 13 years of age because boys tend to reach puberty later than girls.

The physical therapist assistant is working with a patient with know chronic obstructive pulmonary disease. The patient has oxygen saturation readings between 95-96% during low-intensity strengthening. The physical therapist assistant should discontinue treating the patient and notify the physical therapist if the oxygen readings are below ____. A. 80% B. 88% C. 90% D. 94%

C. 90% Normal oxygen saturation levels in an adult are 96-100%, If oxygen saturation is below 90% then the body systems are deprived of oxygen and the patient must have a rest to reoxygenate. The options 88% and 80% are also alarming. but they are not the first point at which the session should stop.

Regarding evidence-based practice, which is the correct definition for specificity? A. The factor believed to bring about the change in dependent variable B. The change or difference that is the result of the intervention C. A measure of the test's ability to correctly identify the portion of those who do not have the condition, disease, or impairment D. A measure of the test's ability correctly identify the proportion of those that truly have the condition, disease, or impairment being measured

C. A measure of the test's ability to correctly identify the portion of those who do not have the condition, disease, or impairment Specificity is a measure of a true negative. It measures the test's ability to correctly identify those who do not have the condition, impairment, or disease being measured. Sensitivity is a measure of true positive. It measures the test's ability to correctly identify those who truly have the condition, impairment, or condition being measured. Sensitivity should be analyzed for standardized outcome measurement tools. And independent variable is the factor believed to bring about the change in the dependent variable, the cause or treatment. The dependent variable is the change or difference that is the result of the intervention.

In regards to evidence-based practice, it is essential that clinicians are able to distinguish between levels of evidence to determine quality research studies. Which is an example of a research study that will be considered level 1 evidence? A. An individual cohort study B. A systematic review of case-control studies C. A systematic review of multiple randomized control trials D. Expert opinion without explicit critical appraisal

C. A systematic review of multiple randomized control trials There are 5 levels of evidence. Level 1 evidence is the highest level of evidence and considered the best and most reliable. Level 1 studies include: systematic review of multiple randomized control trials, individual randomized control trials with narrow confidence level, and all-or-none case series. An individual cohort study is an example of a level 2 type of evidence. An expert opinion without explicit critical appraisal is a level 5 type of evidence. A systemic review of case-control studies is a level 3 type of evidence.

Which condition is characterized by a restriction in shoulder motion as a result of inflammation and fibrosis of the shoulder capsule? A. Bicipial tendinitis B. Impingement syndrome C. Adhesive capsulitis D. Thoracic outlet syndrome

C. Adhesive capsulitis Adhesive capsulitis is also referred to as frozen shoulder. It is characterized by restriction in the shoulder motion as a result of inflammation and fibrosis of the shoulder capsule. This usually follows disuse after an injury or repetitive microtrauma. Restriction follows a capsular pattern of limitation, with thegreatest limitation in external rotation followed by abduction and flexion. Bicipital tendinitis is an inflammation of the long head of the biceps. It results from mechanical impingement of the proximal tendon between the anterior acromion and the bicipital groove of the humerus. Impingement syndrome is characterized by soft tissue inflammation of the shoulder from impingement against the acromion with repetitive overhead motion. Thoracic outlet syndrome is compression of the neurovascular bundle in the thoracic outlet between bony and soft tissue structures. The bundle includes the brachial plexus, subclavian artery and vein, vagus and phrenic nerves, and sympathetic trunk.

Which type of amputation describes a Syme's amputation? A. Amputation through the tibia, below the knee B. Amputation of the entire lower limb C. Amputation through the ankle joint D. Amputation through the knee

C. Amputation through the ankle joint The Syme's amputation is an ankle disarticulation. The amputation is through the ankle joint. The heel pad is preserved and attached to the distal end of the tibia in order to allow for weight bearing. A transtibial amputation is below the knee and through the tibia. A knee disarticulation is an amputation through the knee joint. A hip disarticulation is an amputation of the entire lower limb while preserving the pelvis.

A patient is positioned on her left side with her head down and a pillow under her knees for postural drainage. What is being drained? A. Lingular segments of the left upper lobe B. Right middle lobe C. Anterior basal segments of the right lower lobe D. Anterior basal segments of the left lower lobe

C. Anterior basal segments of the right lower lobe The anterior basal segments of the lower lobes are drained by positioning the patient on her side, head down with a pillow under her knees. Because this patient was positioned on the left side, it is the right lower lobe being drained. If she was positioned on her right side, it would be her left lower lobe being drained. The left upper lobe lingular segments are drained by positioning the patient head down on the right side and rotated 1/4 turn backward. The right middle lobe is drained by positioning the patient head down on her left side and rotated 1/4 turn backward.

The physical therapist assistant is working with a patient with rheumatoid arthritis of the hands. Which is likely? A. Short lasting stiffness in fingers B. Bone spurs C. Bilateral and symmetrical hand deformities D. Radial drift

C. Bilateral and symmetrical hand deformities Bilateral symmetrical deformities in the hands are characteristic of rheumatoid arthritis. Rheumatoid arthritis is characteristic of morning stiffness with lasting stiffness, joint erythema, and swelling. Deformities of the hand include swan neck deformity, boutonniere deformity, and thumb deformities. Short duration stiffness in the hands is typical of osteoarthritis, as is bone spur formation. The patient with rheumatoid arthritis often displays ulnar drift, not radial drift, of the metacarpophalangeal joints.

Which is NOT a desired shape for a residual limb? A. Smooth End B. Conical C. Bulbous end D. Cylindrical

C. Bulbous end As part of the care for a patient after amputation, the physical therapy team should monitor the residual limb healing. The shape should be smooth, cylindrical, or conical. The therapy team should monitor for abnormalities including a bulbous end, dog ears, and adductor roll. Abnormalities will make it difficult for the residual limb to fit properly in a prosthetic socket and may pose a risk for skin breakdown and pain.

In regards to physics related to aquatic exercise, what is the upward force of water on an immersed or partially immersed body or body part? A. Density B. Hydrostatic pressure C. Buoyancy D. Cohesion

C. Buoyancy Buoyancy is the upward force of the water on an immersed (or partially immersed) body or body part. It is equal to the weight of the water that it displaces. The buoyancy creates an apparent decrease in body weight and joint unloading, which allows for easier movement in the water. Cohesion is the tendency of water molecules to adhere to each other. The resistance that is felt while moving through water is due to cohesion. Force is required to separate the water molecules. This can create resistance for exercise purposes. Density is the mass per unit volume of a substance. The density of water is proportional to its depth. The deeper the water, the greater the density. This can be incorporated to challenge or regress exercise. Hydrostatic pressure is the circumferential water pressure exerted on an immersed body or body part. A pressure gradient is established between the surface water and deeper waters due to the increased density in the deeper water. Increased pressure reduces edema and enhances peripheral circulation.

What is the highest level of spinal cord injury in which it may be possible for an individual to be able to live independently? A. C2 B. C4 C. C7 D. T1

C. C7 With SCI at level C7, the patient has muscle innervation to triceps, latisimus dorsi, extrinsic finger extensors, and flexor carpi radialis; as well as everything above level C7 innervation. With this, the patient can be independent with pressure relief, LE dressing, LE self-ROM, transfers, and independent with manual wheelchair propulsion. A SCI at level C4 has respiration and scapular elevation intact. The patient can use a power wheelchair with mouth or chin control but is dependent for transfers and therefore, unable to live alone. At the level of C1-C3, the patient has movements available at the mouth and head but has total dependence with transfers and mobility. At the level of T1, the patient would be able to live independently , but it is a lower lesion level than C7.

Which specific exercise regimen is appropriate for total body conditioning, incorporating a rest period of 30-60 seconds between each exercise, and incorporating many body parts for strengthening? A. Progressive resistive exercise B. Plyometric training C. Circuit weight training D. Brief, repetitive isometric exercise

C. Circuit weight training Circuit weight training uses a sequence of exercise for total body conditioning. There is typically a rest period of 30 seconds to 1 minute between each exercise. Weight machines or free weights are used for resistance. Progressive resistive exercise (PRE) uses the repetition maximum (RM), which is the greatest amount of weight that a muscle can move through the range of motion a specific number of times (depending on the protocol). For example, DeLorme uses 10 RM for baseline. In this protocol, exercise begins with 10 repetitions at 50% RM, followed by 10 repetitions at 75% RM, and 10 repetitions at 100% RM. Plyometric training is an isotonic exercise regimen that combines speed, strength, and functional activities. It is used in later stages of rehabilitation to achieve a high level of performance. It can include activities such as jumping on and off platforms, high-stepping, quick lunges, etc. Brief, repetitive isometric exercise incorporates up to 20 maximum contractions, which are held for 5-6 seconds and performed daily. A rest of 20 seconds between contractions is recommended to reduce the likelihood of increased blood pressure. Strength gains occur over six weeks. An example is Kegel exercises for the pelvic floor.

When performing manual lymphatic drainage for an upper limb lymphedema, which is he correct technique? A. Clear proximal upper extremity ans then distal upper extremity, followed by the trunk, with the direction of flow away from the trunk. B. Clear trunk quadrant first, followed by distal upper extremity and the proximal upper extremity, with the direction of flow toward the trunk. C. Clear trunk quadrants first, followed by proximal upper extremity and then distal upper extremity, with direction of flow toward the trunk. D. Clear trunk quadrants first, followed by proximal upper extremity and then distal upper extremity, with the direction of flow away from the trunk.

C. Clear trunk quadrants first, followed by proximal upper extremity and then distal upper extremity, with direction of flow toward the trunk. Drainage is performed by clearing the trunk quadrants first, followed by limb drainage starting at the proximal segments first and continuing to the more distal segments in a successive manner. The direction of strokes is in the direction of flow, which is toward the trunk.

Which of the following is FALSE in regards to closed chain exercises? A. Loading of muscles, bones, joints, and soft tissues occurs B. Closed chain exercises occur when the body moves over a fixed distal segment C. Closed chain exercises occur when the distal segment moves freely in space D. Mechanoreceptors are stimulated, adding to join stability

C. Closed chain exercises occur when the distal segment moves freely in space Closed chain exercises occur when the body moves over a fixed distal segment. For example, stair climbing, or lunging activities are closed chain exercises. Open chain exercises occurs occur when the distal segment moves freely in space, such as when the arm lifts and lowers a weight. The other options are true for closed chain exercises. Closed chain are excellent for functional weight-bearing and functional mobility strength.

Which term describes the joint position in which the capsule and other soft tissues are maximally tensed? A. Closed chain B. Hard end feel C. Closed-packed position D. Loose-packed position

C. Closed-packed position The closed-packed position is that in which the capsule and other soft tissues are maximally tensed. There is maximal contact between joint surfaces. Joint mobilization cannot be performed properly in this position. The loose-packed position or resting position is the joint position in which the capsule and other soft tissues are in the most relaxed position. There is minimal joint surface contact. The therapist may choose to perform joint mobilization techniques in this position. End feel refers to the way that the joint feels at the end range of motion. A normal end feel that is firm occurs with capsular and ligamentous stretching. A normal end feel that is hard occurs when bone and/or cartilage meet. Closed chain refers to a form of exercise in which the distal segment of a limb is fixed.

The physical therapist assistant is working with a patient who has paraplegia with no sensory or motor function below the level of the lesion. What is the likely injury? A. Complete spinal cord injury at C6 B. Incomplete spinal cord injury at C5 C. Complete spinal cord injury at T8 D. Incomplete spinal cord injury at L1

C. Complete Spinal Cord Injury at T8 The lesion level indicates the most distal uninvolved nerve root segment with normal function. Quadriplegia involves all four extremities and trunk and occurs with an injury between C1 and C8. Paraplegia involves both lower extremities and varying levels of trunk and occurs with an injury between levels T1 and T12-L1. A complete cord injury results in no sensory or motor function below the level of the lesion. An incompletelesion has preservation of sensory or motor function below the level of injury. There are spotty sensations and some muscle function. Therefore, in this patient example, there is involvement of both legs but not upper extremity, so it must be T8 and not C6. Additionally, there is no motor or sensory function, which means it is a complete lesion.

The physical therapist assistant is working with a patient who has quadriplegia with no sensory or motor function below the level of the lesion. What is the likely injury? A. Incomplete spinal cord injury at level T12 B. Incomplete spinal cord injury at level C6 C. Complete spinal cord injury at level C5 D. Complete spinal cord injury at level T8

C. Complete spinal cord injury at level C5 The lesion level indicates the most distal uninvolved nerve root segment with normal function. Quadriplegia involves all four extremities and trunk and occurs with an injury between C1 and C8. Paraplegia involves both lower extremities and varying levels of trunk and occurs with an injury between T1 and T12-L1. A complete cord injury results in no sensory or motor function below the level of the lesion. An incomplete lesion has preservation of sensory or motor function below the level of the injury. There are spotty sensations and some muscle function. Therefore, in this patient example, there is involvement of all extremities (must be cervical level). Additionally, there is no motor or sensory function, which means it is a complete lesion.

Which of the following is NOT considered a contraindication for postural drainage intervention? A. Intracranial pressure > 20 mm Hg B. Active hemorrhage with hemodynamic instability C. Congestive heart failure D. Embolism

C. Congestive heart failure Congestive heart failure is a precaution to the use of the Trendelenburg position, with the head of the bed tipped down 15-18 degrees). It is not a contraindication for postural drainage. Additionally, it is only a precaution with head down position and not with supine, prone, or side-lying positions. The other options are all contraindications for all positions of postural drainage. The percussion and vibration should not be used for patient.

The physical therapist assistant is working with a geriatric patient in her home environment. The patient had a recent total knee replacement and is not complaining of new onset of discomfort in her low back and groin. The physical therapist assistant should recognize the new complaint could be an indication of: A. Neuropathy B. Diarrhea C. Constipation D. Inflammatory bowel syndrome

C. Constipation Constipation can cause discomfort in the low back and groin, as well as abdominal discomfort. The patient who is homebound is likely taking pain medication and has a reduced activity level, both of which can contribute to constipation. Inflammatory bowel syndrome and diarrhea would be associated with abdominal discomfort but not discomfort in the low back and groin. Additionally, the decreased activity level and pain medication would not typically associated with inflammatory bowel syndrome or diarrhea. Neuropathy would be associated with tingling or decreased sensation over the lower legs and feet.

Pelvic floor weakness due to pregnancy and childbirth can lead to organ prolapse. Which is a herniation of the bladder into the the vagina? A. Kegal B. Uterine prolapse C. Cystocele D. Rectocele

C. Cystocele Weakness and laxity in the pubococcygeal muscles from pregnancy, childbirth, and aging can lead to partial or total organ prolapse. A cystocele is the herniation of the bladder into the vagina. A rectocele is the herniation of the rectum into the vagina. A uterine prolapse is the bulging of the uterus into the vagina. Kegal exercises are a strengthening exercise intervention for the pelvic floor muscles.

Which of the following is NOT a physiologic effect of general heat application? A. Increased pulse rate B. Vasodilation C. Decreased cardiac output D. Decreased blood pressure

C. Decreased cardiac output General heat application is when large areas of the body surface are exposed to heat modalitiy such as with a whirlpool and Hubbard tank. Physiological effects of general heat application include increased cardiac output, decreased blood pressure, increased pulse rate, vasodilation, decrease stroke volume, and increased metabolic rate.

Reduction in resting membrane potential is called ____. A. Repolarization B. Pulse duration C. Depolarization D. Hyperpolarization

C. Depolarization Depolarization is a reduction in the resting membrane potential. An action potential is generated when the influx of sodium causes a reduction in resting membrane potential. During depolarization, a positive charge is generated inside the cell, and a negative charge outside the cell is produced due to the flow of ions. Repolarization is when the potassium channels are fully open at the same time the sodium channels are closed and potassium rushes rapidly out of the cell. This makes the transmembrane potential progressively more negative. Hyperpolarization is when the potassium channels remain open long enough to repolarize the membrane. Pulse duration is a characteristic of electrical stimulation modality that describes the length of time the electrical stimulus is present. Short pulse durations stimulate sensory nerves, and longer pulse durations stimulate motor nerves.

Which special test of the shoulder identifies a full rupture of the rotator cuff? A. Speed's test B. Neer's impingement test C. Drop arm test D. Clunk test

C. Drop arm test The drop arm test identifies a tear of the rotator cuff. It is performed by passively abducting the patient's shoulder to 90 degrees. The therapist then drops the arm, and the patient is supposed to control the the descent or hold the arm stable. Of the patient experiences pain or is unable to hold the arm in abduction, it is indicative of a rotator cuff tear. The speed's test identifies bicipital tendonitis. Neer's impingement test identifies impingement of soft tissue structures of the shoulder complex. The clunk test identifies glenoid labrum tears.

When assessing end feels for a typical joint, which would have a normal physiological end feel that would be described as bony or hard? A. Shoulder abduction in a patient with a torn rotator cuff B. Knee flexion C. Elbow extension D. Knee extension

C. Elbow extension Elbow extension would typically have a bony/hard end feel, which indicates when bone and/or cartilage meet. This bony end feel occurs when the humerus and the ulna meet. Knee extension would typically have a firm/capsular end feel, which indicates capsular and ligamentous stretching. This feels like stretching a leather belt. A soft end feel due tosoft tissue approximation occurs when two soft tissues, typically muscle and/or fat, meet and restrict any further movement. In knee flexion, the hamstring approximates with the gastrocnemius and prevents any further motion at the knee. If the patient has a torn rotator cuff and the clinician is assessing the end feel for shoulder abduction, then it would likely be an empty end feel. This occurs when the clinician is unable to bring the joint through the full range due to guarding by the patient, which is typically due to pain.

Which physical therapy exercise is BEST indicated for a patient with multiple sclerosis, who is experiencing an acute relapse of symptoms? A. Progressive resistance strengthening with dumbbells B. Low impact cardiovascular, such as the NuStep C. Exercise should be avoided D. High impact cardiovascular, such as treadmill jogging

C. Exercise should be avoided When working with patients with MS experiencing an acutes relapse, exercise should be avoided. An acute relapse is a direct contraindication to exercise. The other options may be indicated for treatment when the patient is not in in a state of acute relapse, depending on the patient's physical condition and presentation.

A patient is recovering from surgery to correct for chronic anterior shoulder dislocations. The patient is participating in therapy to improve shoulder girdle strength. Which movement should be avoided in the early stages of therapy? A. Flexion B. Abduction C. External rotation D. Internal rotation

C. External rotation External rotation moves the head of the humerus anteriorly. This motion would place stress on the repaired area of the shoulder and risk dislocation. Abduction moves the humeral head in an inferior direction. Flexion and internal rotation move it posteriorly.

Which type of stretching utilizes principles of PNF in which the patient reflexively relaxes the muscle to be elongated prior to or during the stretch technique? A. Manual static passive stretching B. Prolonged mechanical stretching C. Facilitated stretching D. Dynamic stretching

C. Facilitated stretching Facilitated stretching refers to techniques in which the patient reflexively relaxes muscles to be elongated during stretching techniques. It utilizes active inhibition principles of PNF. Examples include: hold-relax, hold-relax-active contraction, and contract-relax-active contraction. Dynamic stretching is an active or passive stretch performed actively by moving joints through the available range of motion, either in body weighted or eliminated position. Gradually, the individual increases the range and speed of movement as the musculature warms up. This motion is controlled and performed at a moderated speed and is appropriate to perform before sports activity. Prolonged mechanical stretching is a low-intensity external force that can be up to 10% of body weight, applied over a long period over a long period of time. This is done by positioning the patient with weighted pulleys, traction, or serial casting. Prolonged stretch may mean 20 minutes to 10 hours or more. Manual static passive stretching stretches the structures beyond the free range of motion to elongate the muscles and soft soft tissue beyond their resting length. The stretch is typically held 30 seconds to 2 minutes and repeated several times. Intensity and duration depends on patient tolerance.

Scar mobilization is safe to perform in which stage or stages of healing? A. Maturation phase only B. Granulation, fibroblastic, and maturation phases C. Fibroblastic and maturation phases D. Inflammatory and fibroblastic phases

C. Fibroblastic and maturation phases The stages of healing are inflammatory, granulation, fibroblastic, and maturation. During the inflammatory and granulation phases of healing, granulation tissue is being laid down and care should be taken to avoid damage, so scar tissue mobilization is not indicated. During the fibroblastic phase, collagen fibers are laid down and scar tissue mobilization can be initiated. Scar tissue mobilization can also be done in the maturation phase.

In regards to the FITT acronym for exercise training, which is NOT a correct term? A. Time B. Type C. Fitness D. Intensity

C. Fitness The FITT principle can be utilized to adjust, progress, and regress exercise. FITT stands for frequency, intensity, time, and type. Intensity is interrelated with both time and frequency. Frequency is the number of exercise sessions per week. Intensity is the overload and the primary way to improve cardiovascular endurance. Time is the duration of exercise. Type is the specific exercise being performed (aerobic, resistance, etc).

A physical therapist evaluated a female patient after a stroke and places her in Stage 1 on Brunnstrom's Stages of Motor Recovery scale. What motor presentation can the physical therapist assistant expect? A. Control and coordination near normal B. Voluntary movement possible but only in synergies, peak spasticity C. Flaccidity, no voluntary movement D. Emergence of spasticity, synergies, hyperreflexia

C. Flaccidity, no voluntary movement The Brunnstrom's Stages of Motor Recovery are as follows: Stage 1: Initial flaccidity, no voluntary movement Stage 2: Emergence of spasticity, hyperreflexia, synergies Stage 3: Voluntary movement possible but only in synergies, spasticity at its peak Stage 4: Voluntary control in isolated joint movements emerging, corresponding decline of spasticity and synergies Stage 5: Increasing voluntary control Stage 6: Control and coordination near normal

Which gait pattern provides the maximum stability with three points of support, indicated for patients with poor balance? A. Two-point gait B. Swing-to gait C. Four-point gait D. Three-point gait

C. Four-point gait A four-point gait pattern is a slow and stable pattern in which one assistive device is advanced forward and placed on the floor, followed by advancement of the opposite leg. Then the remaining assistive device is advanced forward followed by the opposite leg. Sequencing looks like this: Left crutch, right leg, right crutch, left leg. It requires using two assistive devices, cane or crutch. It provides the maximum stability with three points of contact while one limb is moving. It is appropriate for patients with poor balance. A two-point gait pattern allows for natural arm and leg gait motion. With two-point gait, one assistive device and opposite extremity move together. Three-point gait is indicated for weight bearing restrictions for one lower extremity. Both assistive devices and involved leg are advanced together and then the uninvolved leg advances to or past the assistive devices. A swing-to pattern is used when both lower extremities are involved. Both crutches are advanced forward together, and weight is shifted on the hands for support and both legs are swung forward.

What is the term for the amount of air that resides in the lungs after a normal resting tidal exhalation? A. Residual volume B. Total lung capacity C. Functional residual capacity D. Vital capacity

C. Functional residual capacity The functional residual capacity is the amount of air that resides in the lungs after a normal residing tidal exhalation. It is the addition of expiratory reserve volume plus residual volume. Residual volume is the volume of gas that remains in the lungs after expiratory reserve volume has been exhaled. Total lung capacity is the total amount of air contained within the thorax during a maximum inspiratory effort. Vital capacity is the amount of air under volitional control (Inspiratory reserve volume + tital volume + expiratory reserve volume).

In regards to arthrokinematics, joint accessory motion, what is the movement between the metacarpal of the hand and the proximal phalanx? A. Roll B. Spin C. Glide D. Extension

C. Glide Glide or slide consists of a pure translatory motion of one surface gliding on another surface. An example is the movement of the proximal phalanx at the head of a metacarpal bone. Roll consists of one joint surface rolling on another such as a tire rolling over a road. An example is the movement between the femoral and tibial articular surfaces in the knee. Spin consists of a rotation of the movement component of a joint. An example is between the radial head with the humerus. Extension is a term used to describe osteokinematics, which is physiological motion between two bones.

Which is the leading cause of death as a complication of a burn injury? A. Shock B. Cardiac complications C. Infection D. Pulmonary complications

C. Infection Common complications of a burn injury include infection, shock, pulmonary complications (smoke inhalation and poisoning, as well as restrictive lung disease from trunk burns), metabolic complications (weight loss, negative nitrogen balance, low energy), and cardiac and circulatory complications (fluid and plasma loss resulting in decreased cardiac output). Infection is the leading cause of death, dependent on the amount of skin burned. Gangrene may develop.

The physical therapist is delivering joint mobilization intervention at the patient's shoulder. The therapist delivers 3 sets of 30 seconds of small amplitude oscillation near the end of passive range, intended to increase capsular mobility. Which grade of joint mobilization is being performed? A. Grade 1 B. Grade 3 C. Grade 4 D. Grade 5

C. Grade 4 Grade 4 joint mobilization is being performed in this example. Grades 3 and 4 are intended for increasing capsular mobility. Grades 1 and 2 are intended for joint nutrition and pain reduction. The grades of joint mobilization are determined by dosages of oscillation techniques. Grade 1: small amplitude oscillation before the point of tissue resistance Grade 2: large amplitude oscillation before the point of tissue resistance Grade 3: large amplitude oscillation that pushes into tissue resistance Grade 4: small amplitude oscillation near the end of passive range Grade 5: small amplitude, high-velocity manipulation past the end of passive range

The physical therapist assistant is utilizing join mobilization to increase joint nutrition and decrease pain. Which grade or grades work to accomplish this goal? A. Grade 2 B. Grade 3 C. Grades 1 and 2 D. Grades 3 and 4

C. Grades 1 and 2 Grade 1 and grade 2 joint mobilization are intended to maintain joint nutrition and to decrease pain. Grades 3 and 4 are primarily used for stretching of he joint capsule. Grade 5 is a manipulation past the end of passive range.

Regarding general principles of progression, which principle is NOT an accurate progression of an exercise? A. Large base of support progresses to a small based of support B. Lots of feedback progresses to no feedback C. Higher center of gravity progresses to a lower center of gravity D. Closed environment progresses to an open environment

C. Higher center of gravity progresses to a lower center of gravity In order to progress an intervention, the patient will mover from a lower center of gravity to a higher center of gravity. Moving from a high center of gravity to a lower center of gravity is a regression. The other answers are appropriate progressions.

The quadrilateral socket is the most commonly prescribed above-knee prosthetic socket. Which is NOT true regarding the quadrilateral socket? A. Reliefs provided for the adductor longus tendon, hamstring tendons, sciatic nerve, gluteus maximus, and rectus femoris B. Contains broad horizontal posterior shelf for seating of the ischial tuberosity and gluteals C. Higher medial and posterior walls in comparison to anterior and lateral walls D. Posterior directed force provided by anterior and lateral walls to ensure proper seating

C. Higher medial and posterior walls in comparison to anterior and lateral walls In a quadrilateral socket, the medial wall is the same height as the posterior walls, while the anterior and lateral walls are 2.5 to 3 inches higher. The anterior and lateral walls provide a posterior directed force for proper seating. Additionally, the socket has reliefs for adductor longus tendon, hamstring tendons, sciatic nerve, gluteus maximus, and rectus femoris. Scarpa's bulgeis an area in the anterior wall built up to distribute forces across the femoral triangle. The quadrilateral socket provides excellent stability while accommodating for sensitive components of the upper thigh and buttocks.

The physics related to heat transmission for utilization of the superficial thermotherapy modalities include conduction, convection, and radiation. Which of the following is an example of heat transmission through conduction? A. Hubbard tank B. Infrared lamp C. Hot pack D. Whirlpool

C. Hot pack Conduction occurs when heat is transferred from a warmer object to a cooler object by means of direct molecular interaction of objects in physical contact. Examples include hotpacks and paraffin. Whirlpool and Hubbard tanks utilize convection. Infrared lamps utilize radiation.

Which types of wound care dressings are characterized as water or glycerine-based gels that are insoluble in water, used for partial and full thickness wounds, and those with necrosis and slough? A. Foams B. Hydrocolloids C. Hydrogels D. Alginates

C. Hydrogels Hydrogels are water or glycerine-based gels insoluble in water. They are available as solid sheets, amorphous gels, or impregnated gauze. They are used for partial and full thickness wounds and those with necrosis and slough. They are soothing and cooling to the patient and rehydrate dry wound beds. Hydrocolloids are adhesive wafers that contain absorptive particles that interact with wound fluid to form a gelatinous mass over the wound bed. They can be occlusive or semi-occlusive. They are indicated for protection of partial thickness wounds and wounds with mild exudate, maintaining a moist wound environment. Alginates are soft, absorbent, nonwoven dressings derived from seaweed and often used for wounds with moderate to large amounts of exudate. Alginates react with wound exudate to form a viscous hydrophillic gel mass over the wound area. They are able to absorb up to 20 times their weight in drainage. Foams are semiperimeable membranes that are either hydrophillic or hydrophobic. They vary in thickness, absorptive capacity, and adhesive properties. They are indicated for partial and full thickness wounds with minimal or moderate exudate.

Which term correctly identifies the problems in a body function of structure such as significant deviations or loss? A. Performance B. Capacity C. Impairment D. Health condition

C. Impairment According to The World Health Organization International Classification of Functioning, Disability, and Health Resources, impairments are the problems in body function or structure such as a significant deviation or loss. Impairment examples can include: active shoulder range of motion to 115 degrees flexion, manual muscle strength of quadriceps femoris grade 3/5, and elbow flexion contracture. Health condition is an umbrella term for disease, disorder, injury, or trauma and may include other circumstances such as aging, stress, congenital abnormality, or genetic predisposition. Capacity is an individual's ability to execute a task or action. Performance describes what an individual does in his/her current environment.

In regards to research and evidence-based practice, which term correctly describes the cause or treatment, which is believed to bring about a change? A. Hypothesis B. Sampling C. Independent Variable D. Dependent variable

C. Independent variable The independent variable is the activity or factor believed to bring about a change in the dependent variable. It is the cause or treatment. The dependent variable is the change or difference that results from the intervention (independent variable). It is the outcome being evaluated. The hypothesis is a tentative and testable explanation between the relationship between variables. The results of the experiment determine whether the hypothesis is accepted rejected. Sampling is the selection of individuals for a study from a population.

The physical therapist assistant is working in the patient's home, educating the family on pressure ulcers. The patient is wheelchair bound and has been diagnosed with Alzheimer's disease. The PTA should educate the family that the most likely location of a pressure ulcer for this patient would be: A. greater trochanter B. lateral malleous C. Ischial tuberosity D. Femoral Condyle

C. Ischial Tuberosity In sitting, the greatest pressure and weight are supported by the ischial tuberosities. Thus, for a wheelchair-bound patient, the most likely area of pressure ulcer formation would be the ischial tuberosities. The greater trochanter, lateral malleous, and femoral condyle would all be susceptible to breakdown if the patient was in prolonged positioning of side-lying.

In a transfemoral residual limb, which is NOT a pressure sensitive area? A. Perineal area B. Distolateral end of the femur C. Ischial tuberosity D. Pubic symphysis

C. Ischial tuberosity The ischial tuberosity is a pressure-tolerant area of the typical transfemoral residual limb. The pressure-sensitive areas include the distolateral end of the femur, pubic symphysis, and perineal area. It is important to understand pressure sensitive areas in order to monitor for potential skin breakdown and patient comfort when designing and wearing the socket of the prosthetic.

Which form of resistance exercise uses dynamic muscle contraction with constant or variable resistance moved through a range of motion, with shortening and/or lengthening of the muscle? A. Eccentric B. Isometric C. Isotonic D. Isokinetic

C. Isotonic Isotonic exercises use dynamic muscle contraction. There is constant (free weights) or variable (machine) resistance and uses concentric and eccentric contractions. The patient moves a resistance through the available range of motion and speed can be variable. Isometric resistance exercise is performed as a static muscle contraction, with no muscle length change. The patient holds a muscle contraction for at least 6 seconds. This is beneficial to strengthen a muscle at a specific point in the ROM. Isokinetic resistance exercise is dynamic and has a speed control for muscle shortening and lengthening. Resistance is accommodating and variable. The individual uses isokinetic equipment, and it provides maximum resistance at all points of the range of motion. Eccentric resistance exercise is performed with dynamic lengthening contraction. This is useful to prepare muscles for functional activities and for muscles unable to perform concentric contraction. Maximum eccentric contraction produces more force than maximal concentric contraction.

A patient presents with an acute synovitis of the temporomandibular joint. Which intervention is NOT indicated for this patient? A. Postural re-education B. Phonophoresis C. Joint mobilization D. Instruction to eat soft foods

C. Joint mobilization Joint mobilization is not indicated with an acute inflammation of the temporomandibular joint (TMJ). Phonophoresis will reduce an acute inflammatory situation, as will eating soft foods. Postural re-education will normalize the postural and muscular tension related to poor alignment of the TMJ.

When performing soft tissue mobilization to the gastrocnemius/soleus complex, the therapist notices several dark purple lesions, which the patient reports as itching. Which form of malignant tumor is characterized by lesions of endothelial cell origin that present as dark purple or red and are associated with itching and pain? A. Basal cell carcinoma B. Squamous cell carcinoma C. Kaposi's sarcoma D. Malignant melanoma

C. Kaposi's sarcoma Kaposi's sarcoma is a lesion of endothelial cell origin with red or dark purple macules that progress to nodules or ulcers. It is associated with itching and pain. It is common in lower extremities. Squamous cell carcinomas have a poorly defined margin. They present as a flat red area, ulcer, r nodule. Squamous cell carcinoma grow more quickly and is common on sun-exposed areas including the face, neck, and back of hands. It can metastasized and is common on the face of fair-skinned people. It is associated with prolonged sun exposure. Malignant melanoma is a tumor that arises from melanocytes. Superficial spreading melanoma is the most common type. It is characterized by uneven edges, irregular borders, variations in color, larger than 6 mm, and typically elevated presentation.

The physical therapist assistant should educate patients with lymphedema in management strategies. Which of the following is NOT a strategy to manage lymphedema? A. Wear compressive garments when traveling B. Maintain water intake C. Keep limbs in dependent position for easier circulation D. Avoid sitting cross-legged

C. Keep limbs in dependent position for easier circulation The limbs should not be placed in a dependent position, as gravity promotes edema. Strategies for lymphedema management include avoid dependent positions for extended periods of time, avoid sitting cross-legged, take breaks when traveling, maintain water intake, maintain ideal body weight, perform muscle-pumping exercises regularly, avoid lifting and carrying heavy loads, avoid wearing restrictive clothing, wear compressive garments when traveling, and monitor sodium intake.

Which nerve root is tested with the hamstrings deep tendon reflex examination? A. C7-C8 B. L2-L4 C. L5-S3 D. S1-S2

C. L5-S3 The hamstring deep tendon reflex testing examines the function of the nerve root level L5-S3. A deep tendon reflex is tested with the muscle positioned at mid-range. The tendon is tapped with a reflex hammer. Deep tendon reflexes are also called a stretch reflex. S1-S2 is tested with the Achilles deep tendon reflex. The L2-L4 nerve root level is tested with quadriceps deep tendon reflex. The C7-C8 nerve roots are tested with the triceps deep tendon reflex.

The initial evaluation notes that a patient has a weak gluteus medius. Which gait deviation can the physical therapist assistant expect to see with this patient during gait? A. Forward trunk lean B. Lateral trunk bending to the opposite site C. Lateral bending of trunk to the same side D. Backward trunk lean

C. Lateral bending of trunk to the same side Weakness in the gluteus medius muscle will cause a gait deviation of lateral trunk bending to the same side. A lateral trunk lean to the opposite side is associated with a weak hip flexor. A forward trunk lean is associated with weakness in the quadriceps or a hip/knee flexion contracture. A backward trunk lean is associated with weakness in the gluteus maximus.

In utilization of iontophoresis, which agent is indicated for pain relief? A. Copper sulfate B. Magnesium sulfate C. Lidocaine D. Dexamethasone phosphate

C. Lidocaine Lidocaine is utilized when the treatment indication is analgesia. Lidocaine has a positive polarity. Dexamethasone phosphate is indicated for inflammatory conditions. Copper sulfate is indicated for fungal infections. Magnesium sulfate is indicated for muscle spasm.

The physical therapist assistant is educating a patient in a work conditioning program to utilize proper body mechanics for his job that includes lifting boxes repeatedly. What does the therapist need to teach the patient to utilize proper lifting techniques and reduce risk of injury to his back? A. Maintain a posterior pelvic tilt B. Maintain an anterior pelvic tilt C. Maintain neutral spinal alignment D. Keep box away from the body when lifting

C. Maintain neutral spinal alignment The back should be maintained in the neutral spinal alignment in order to ensure best body mechanics for lifting. Increased pelvic tilt in either anterior or posterior directions puts the spine in a position of weakness and places the body at greater risk of injury. When lifting, the box or load should be kept close to the body and not away from the body.

Which of the following is NOT considered a facilitation technique when providing proprioceptive sensory intervention techniques? A. Resistance B. Quick stretch, tapping of muscle belly C. Maintained touch/pressure D. Joint traction

C. Maintained touch/pressure A sustained touch or maintained pressure is an inhibitory technique. This technique produces a calming effect and generalized inhibition. Facilitation techniques include all other options. Quick stretch and tapping of muscle belly facilitates agonist muscle and inhibits antagonist. Resistance recruits motor units and facilitates and strengthens agonist contraction. Joint contraction enhances joint awareness and actions of flexors.

A patient presents with pain in his elbow with a 3-month duration. In reviewing his evaluation, the therapist discovers he is an avid golfer and runner. What is the likely condition? A. Elbow dislocation B. Lateral epicondylitis C. Medial epicondylitis D. Elbow contracture

C. Medial epicondylitis The patient has a social history of playing alot of golf and has chronic elbow pain. Medial epicondylitis is known as "golfer's elbow" due to the fact that inflammation occurs commonly in golfers. Medial epicondylitis is an inflammation of the pronator teres and flexor carpi radialis tendons at their attachment to the medial epicondyle of the humerus. It is a result of chronic overuse in sports, such as baseball pitching, golfing, swimming, or in occupations that require a strong hand grip with pronationof the forearm. Lateral epicondylitis is also knwn as tennis elbow. It is chronic inflammation of the extensor tendons of the forearm, most typically the extensor carpi radialis brevis (ECRB) tendon at its proximal attachment to the lateral epicondyle of the humerus. The onset is gradual and results from repetitive motions with wrist extension or strong gripping with wrist extension. This motion overloads the ECRB. Elbow contractures result in loss of motion dues to soft tissue restriction or loose body in the elbow. Elbow dislocations are typically the result of elbow hyperextension from a fall on the outstretched arm.

In reviewing the physical therapist's evaluation, the physical therapist assistant notes his patient tested positive for McMurray's test. What does this test indicate? A. Tear of anterior cruciate ligament B. Patellofemoral dysfunctional C. Meniscal tear D. Tear of posterior cruciate ligament

C. Meniscal tear McMurray's test identifies meniscal tears. The examiner passively applies a medial or lateral rotation to the knee while passively moving the knee from flexion to extension. Pain or an audible click is positive for meniscal involvement. Lachman's stress examines the integrity of the anterior cruciate ligament. The posterior drawer test identifies the integrity of the posterior cruciate ligament. Clarke's sign identifies patellofemoral dysfunction.

A patient suffered a peripheral nerve injury that results in foot drop of the left lower extremity during gait. The physical therapist has included functional electrical stimulation for orthotic substitution. Which phase of gait should the PTA set up the electrical stimulation to contract the appropriate muscles? A. This is not an appropriate intervention B. Heel-off C. Midswing D. Foot flat

C. Midswing Stimulation of the dorsiflexor muscle group during the swing phase (during midswing) will prevent the toes from dragging on the ground and allow for appropriate foot clearance. Heel-off and foot flat are incorrect because the gastrocsoleous complex is activated during theses times, not the dorsiflexor group. Functional electrical stimulation is an appropriate intervention for drop foot as long as there are no other contraindications.

The physical therapist assistant is working with a patient who has peripheral vascular disease. Which is NOT a sign or symptom of chronic ARTERIAL insufficiency? A. Severe muscle pain over calf and lower leg B. Loss of hair C. Minimal aching pain in the lower leg D. Pale, shiny, dry skin

C. Minimal aching pain in the lower leg This is a sign of chronic venous insufficiency. Signs of chronic arterial insufficiency include severe muscle pain, intermittent claudication, resting pain, muscle fatigue, and pain in the calf and lower leg. There is decreased or absent pulses, and the skin is dry, pale, and shiny. There is loss of hair and nail changes. Signs of chronic venous insufficiency include minimal to moderate steady aching pain in the lower leg. There is muscle compartment tenderness and venous dilation or varicosity is present. Patients have dark, cyanotic thickened skin, and it may lead to static dermatitis.

A physical therapist assistant is working with a patient with Parkinson's disease and performing a gait analysis. What can be expected? A. Internal rotation of the hips and flexed knees B. Decreased stance time unilaterally and slow cadence C. Minimal trunk rotation and decreased arm swing D. Weakness on one side of the body with instability

C. Minimal trunk rotation and decreased arm swing In patients with Parkinson's disease, common gait deviations include minimal trunk rotation, decreased arm swing, decreased step length, shuffling gait, forward-stooped posture, and instability. These gait deviations are more prevalent with further disease progression. The other options are more characteristic of other conditions. Weakness on one side of the body with instability is common for an individual after a stroke. Decreased stance time unilaterally and slow cadence is common for an individual after orthopedic surgery such as a total hip arthroplasty. Internal rotation of the hips and flexed knees is common for an individual with spastic cerebral palsy.

Which of the following is NOT a contraindication or precaution for joint mobilization? A. Joint hypermobility B. Joint effusion C. Muscle spasm and guarding D. Bone disease

C. Muscle spasm and guarding Muscle spasm and guarding is an indication for joint mobilization. Other indications for joint mobilization include pain, joint hypomobility, and functional limitation in joint range of motion. Contraindications for joint mobilization include joint hypermobility, joint effusion, and inflammation. Precautions include malignancy, unhealed fracture, bone disease, hypermobility in adjacent joints, systemic connective tissue disease, and if the patient is on blood-thinning medication.

Which is the MOST common form of temporomandibular joint dysfunction? A. Rheumatoid arthritis B. Osteoarthritis C. Myofascial pain D. Internal derangement of the joint

C. Myofascial pain There are three diagnostic categories of temporomandibular joint dysfunction. The three categories are degenerative joint disease (DJD), which can include osteoarthritis or rheumatoid arthritis, myofascial pain is the most common form of dysfunction, which is discomfort or pain the muscles that control the jaw function, and internal derangement of the joint refer to a dislocated jaw, displaced jaw, displaced articular disc,or injury to the condyle.

Which type of nerve injury is characterized as an injury to the nerve that causes a transient loss of function such as a compression injury? A. Entrapment B. Neurotmesis C. Neurapraxia D. Axonotmesis

C. Neurapraxia Neurapraxia is a class-1 traumatic nerve injury. It is an injury to the nerve that causes a transient loss of function like a compression. Nerve dysfunction may be rapidly reversed or persist a few weeks. Neuromesis is a class-3 traumatic nerve injury. It is characterized by cutting of the nerve with severance of all structures and complete loss of function. Re-innervation fails without surgical intervention. Axonotmesis is a class-2 traumatic nerve injury, It is an injury to nerve interrupting the axon and causing loss of function and degeneration distal to the lesion such as a crush injury. Regeneration is possible. Entrapment is a condition of pressure on a nerve where it passes over a bony prominence or restricted opening. It results in motor and sensory disturbances in the area of nerve distribution.

A researcher expects to find no statistically significant differences between a geriatric population and an adolescent population in the outcomes of a new functional outcome tool. This is an example of what kind of hypothesis? A. This is not an appropriate hypothesis for a research design B. Experimental hypothesis C. Null Hypothesis D. Directional hypothesis

C. Null Hypothesis A null hypothesis states that there is no relationship or no difference between variables. A directional and experimental hypothesis predict there will be a relationship between the variables in an appropriate hypothesis to use in a research design.

Passive stretching would be inappropriate for a child with which condition? A. Juvenile Rheumatoid Arthritis B. Cerebral palsy C. Osteogenesis Imperfecta D. Spina bifida

C. Osteogenesis Imperfecta Osteogenesis Imperfecta is characterized by fragile bones that break easily as well as hypermobile joints. Passive stretching is not recommended in these patients due to risk of bone fracture and joint subluxation. Stretching is indicated for pediatric conditions of cerebral palsy, juvenile rheumatoid arthritis, and spina bifida.

What condition is characterized by decalcification of bones as a result of vitamin D deficiency? A. Osteomyelitis B. Ostegenesis imperfecta C. Osteomalacia D. Osteoporosis

C. Osteomalacia Osteomalacia is a condition of decalcification of bones due to vitamin D deficiency. Symptoms include severe pain, fractures, weakness and bone deformities. Patients are given calcium, vitamin D, and calciferol as medical treatment. Osteomyelitis is an inflammatory response within the bone caused by infection. The infection is usually caused by Staphylococcus aureus. Medical treatment is antibiotic medication. Osteogenesis imperfecta is an abnormality in the collagen gene that causes problems with the amount and quality of collagen in the body. It is characterized by fragile bones that break easily, dental problems, scoliosis, kyphosis, short stature, and hearing loss. Medical intervention includes orthotic devices, calcium, and treatment of fractures. Osteoporosis is a disease process that results in reduction of bone mineral density. It is a failure of bone formation to keep ace with bone reabsorption and destruction. Osteoporosis is more common in women, as there is a hormonal deficiency associated with menopause. Treatment includes calcium and vitamin D intake, as well as weight-bearing and strengthening exercises.

Which of the following is NOT a typical characteristic of rheumatoid arthritis affecting the hands? A. Boutonniere deformity B. Ulnar drift of metacarpophalangeal joints C. Osteophyte formation D. Swan neck deformity

C. Osteophyte formation Osteophyte, bone spur, formation is typical for osteoarthritis, not rheumatoid arthritis. The other options are all characteristics for rheumatoid arthritis. Bilateral symmetrical deformities in the hands are characteristics of rheumatoid arthritis. Rheumatoid arthritis is characteristic of morning stiffness with lasting stiffness, joint erythema, swelling, and ulnar drift of the metacarpophalangeal joints. Deformities of the hands include swan neck deformity, boutonnierre deformity, and thumb deformities.

If a patient is suspected to have bone cancer, possible stress fracture or osteoporosis, which type of imaging might be ordered? A. MRI B. Myelography C. Osteoscintigraphy (bone scan) D. Discography

C. Osteoscintigraphy (bone scan) Chemicals with radioactive tracers are injected into the patient. The isotopes settle in areas with high metabolic bone activity. Then, radiographs are taken and are able able to visualize any "hot spots" of activity. MRI is used to identify tissue anatomy, using magnetic fields. Discography identifies abnormalities within a disc annulus or nucleus. Myelography is used to observe the anatomy withing the vertebral canal.

Where is the auscultation landmark for the mitral valve located? A. Over the second left intercostal space at the sternal border B. Over the fourth left intercostal space at the sternal border C. Over the fifth left intercostal space at the midclavicular line D. Over the second right intercostal space at the sternal border

C. Over the fifth left intercostal space at the midclavicular line The auscultation landmark for the mitral valve is at the fifth left intercostal space at the midclavicular line. The auscultation landmark for the tricuspid valve is at the fourth left intercostal space at the sternal border. The auscultation landmark for the pulmonic valve is at the second left intercostal space at the sternal border. The auscultation landmark for the aortic valve is at the second right intercostal space at the sternal border.

What is the term to describe a patellar malalignment in which the patella tracks too far superiorly? A. Osgood-Schlatter B. Patella baja C. Patella alta D. Pes anserine bursitis

C. Patella alta Patella alta is a patellar malalignment in which the patella tracks superiorly in the femoral intercondylar notch. Patella baja is a patellar malalignment in which the patella tracks inferiorly in the femoral intercondylar notch. Osgood-Schlatter (jumper's knee) is a mechanical dysfunction resulting in traction apophysitis of the tibial tubercle at the patellar tendon insertion. Physical therapy goals include modification of activities to reduce stress. Pes anserine bursitis is typically caused by overuse or a contusion. It is an inflammatory condition of the medial knee, as a result of inflammation of the anserine bursa.

A physical therapy plan of care has directed the physical therapist assistant to provide exercise to a patient in the early stages of care for a shoulder injury. Which exercise should be implemented? A. Strengthening of the deltoid B. Strengthening of the middle trapezius and rhomboids C. Pendulum exercises D. Active range of motion abduction and external rotation exercises

C. Pendulum exercises During initial phases of rehabilitation, focus should be on protection of injured structures to allow for healing while promoting range of motion and eliminating pain. Pendulum exercises do not stress damaged soft tissue structures and provide movement of the joint for range of motion and joint nutrition. The other options would place too much stress on the shoulder in the initial rehabilitation phase. Active motion into abduction and external rotation is too strenuous when beginning shoulder rehabilitation. Additionally, resistance strengthening should be avoided until further healing has occurred.

For patients with burn injuries, it is important that the therapy team works to prevent or reduce complications of immobilization through positioning and splinting to avoid contractures. Which is the correct position to place a patient with an ankle burn? A. Position in ankle inversion B. Position in plantar flexion C. Position in neutral alignment D. Position in ankle eversion

C. Position in neutral alignment The common deformity with an ankle burn is plantar flexion contracture. Therefore, dorsiflexion should be stressed. The patient should have ankle positioned in a neutral alignment with a splint or plastic ankle foot orthosis.

A patient with a spinal cord injury presents with bilateral loss of proprioception, vibration, pressure, and two-point discrimination. She has preservation of motor function, pain, and light touch. Which clinical syndrome does she likely have? A. Brown-Sequard Syndrome B. Anterior Cord Syndrome C. Posterior Cord Syndrome D. Central Cord Syndrome

C. Posterior Cord Syndrome Posterior Cord Syndrome is a loss of dorsal columns bilaterally with bilateral loss of proprioception, vibration, pressure, epicritic sensations. There is preservation of motor function, pain, and light touch. Brown-Sequard Syndrome is a hemisection of the spinal cord. It typically is caused by penetration wounds from a gunshot or knife with aymmetrical symptoms. Characteristics include ipsilateral loss of dorsal columns with loss of tactile discrimination, pressure, vibration and proprioception as well as ipsillateral loss of corticospinal tracts with loss of motor function and spastic paralysis below the lesion. There is contralateral loss of spinothalamic tract with loss of pain and temperature below the level of the lesion. At the level of the lesion. there is bilateral loss of pain and temperature. Central Cord Syndrome is characterized by cavitation of central in cervical section, loss of ventral horn with bilateral loss of motor function, preservation of proprioceptive, and discriminatory sensation. Anterior Cord Syndrome is a loss anterior cord with loss of lateral corticospinal tracts with bilateral loss of motor function, spastic paralysis below the level of lesion, as well as loss of spinthalamic tracts with bilateral loss of pain and temperature. There is perservationof dorsal columns with proprioception, kinesthesia, and vibratory sense.

Which of the following is NOT a fine motor task typically present in a 12-month-old infant? A. Takes objects out of containers B. Displays emerging pincer grasp C. Stacks six blocks D. Puts objects into containers

C. Stacks six blocks The fine motor task of stacking six blocks does not typically occur until 16-23 months of age. An infant who is 12-15 months of age can typically build a tower of two blocks. At 10-11 months, the infant displays an inferior pincer grasp, and between 12-15 months, he uses a neat pincer grasp. At 10-11 months, he is able to put objects into a container. A 8-9 months, he is able to take objects out of a container.

A patient s/p CVA presents with right-sided motor and sensory loss, homonymous hemianopsia, visual agnosia, involuntary movement, oculomotor nerve palsy, thalamic pain syndrome, and Pusher syndrome. Which CVA syndrome is likely? A. Middle cerebral artery syndrome B. Anterior cerebral artery syndrome C. Posterior cerebral artery syndrome D. Vertebrobasilar artery syndrome

C. Posterior cerebral artery syndrome CVA syndromes are characterized by the impacted cerebral artery. The posterior cerebral artery supplies the occiptal lobe, medial and inferior temporal lobe, thalamus, and midbrain. With posterior cerebral artery syndrome, the patient can experience contralateral sensory and motor loss, homonymous hemianopsia, visual agnosia, oculomotor nerve palsy, involuntary movement, Pusher syndrome, and thalamic pain syndrome. The anterior cerebral artery supplies the medial part of the frontal and parietal lobe, basal ganglia, and corpus collosum. With anterior cerebral artery syndrome, the patient can experience contralateral sensory and motor loss with lower extremities affected more than upper extremities. The patient can have mental impairment, urinary incontinence, apraxia, slow movement, and behavioral changes. Middle cerebral artery syndrome is characterized by contralateral sensory motor loss, with face and upper extremity affected more than lower extremity. These patients can have perceptual deficits, homonymous hemianopsia, Broca's aphasia, Wernicke's aphasia, and global aphasia. The middle cerebral artery supplies the lateral cerebral hemispheres, including the frontal, temporal, and parietal lobes. The vertebrobasilar artery supplies the medulla, the pons, and cerebellum. Patients with vertebrobasilar artery syndrome have a wide variety of symptoms, ipsilaterally, and contralaterally. They can have cranial nerve involvement, ataxiam Wallenburg's syndrome, and locked-in syndrome.

What is the MOST commonly occurring disc disorder of the lumbar spine? A. Internal disc disruption B. Central posterior bulge/herniation C. Posterolateral bulge/herniation D. Anterior bulge/herniation

C. Posterolateral bulge/herniation This is the most commonly observed disc disorder of the lumbar spine due to three structure deficiencies that include: the posterior disc is narrower in height that the anterior disc, the posterior longitudinal ligament is not as strong and only centrally located in the lumber spine, and the posterior lamellae of the annulus are thinner. Internal disc disruption occurs when the internal structure of the disc annulus is disrupted, but external structures remain normal. This most commonly occurs in the lumbar region. Central posterior bulge/herniation is more commonly observed in the cervical spine but can be seen in the lumber spine. The anterior bulge/herniation is rare due to structural integrity of the anterior intervertebral disc.

Electrical potential is generated across the cell membrane due to the higher concentration of ____ and anions on the inside cell relative to the concentration of sodium on the outside. A. Zinc B. Oxygen C. Potassium D. Magnesium

C. Potassium The cell membrane is electrically excitable due to the higher concentration of potassium and and anions on the inside of the cell in comparison to the concentration of sodium on the outside. A negative charge is produced within the cell, and a positive change develops on the outside of the cell as the positively charged potassium diffuses from the cell. Resting membrane potential is maintained by an active sodium-potassium pump that takes in potassium and releases sodium.

Which specific exercise regimen incorporates the repetition maximum as a baseline for building muscle strength in completing three sets of 10 repetitions lifting weight? A. Plyometric training B. Circuit weight training C. Progressive resistive exercise (PRE) D. Brief, repetitive isometric excercise

C. Progressive resistive exercise (PRE) Progressive resistive exercise (PRE) uses the repetition maximum (RM), which is thegreatest amount of weight that a muscle can move through a range of motion a specific number of times, according to the protocol. For example, DeLorme uses 10 RM for baseline. In this protocol, exercise begins with 10 repetitions at 50% RM, followed by 10 repetitions at 75% RM, and 10 repetitions at 100% RM. Plyometric training is an isotonic exercise regimen that combines speed, strength, and functional activities. It is used in later stages of rehabilitation to achieve high level of performance. It can include activities such as jumping on and off platforms, high-stepping, quick lunges, etc. Circuit weight training uses a sequence of exercise for total body conditioning. There is typically a rest period of 30 seconds to 1 minute between each exercise. Weight machines or free weights are used for resistance. Brief, repetitive isometric exercise incorporates up to 20 maximum contractions, which are held for 5-6 seconds and performed daily. A rest of 20 seconds between contractions is recommended to reduce the likelihood of increased blood pressure. Strength gains occur over six weeks. An example is Kegel exercises for the pelvic floor.

A patient presents for physical therapy with a referral for chronic low back pain. After the evaluation, the physical therapist is concerned hat the patient may have spondylolisthesis. The patient is to have an imaging study performed but not for another week. Until this is confirmed or negated, which exercise should be avoided? A. Hamstring stretching B. Isometric strengthening of mutifidus C. Prone back extension D. Transverse abdominis activation

C. Prone back extension Spondylolisthesis is a defect with forward slippage of one vertebra on the one below it. Excessive motion with spinal extension could cause further damage. The patient should avoid extension, ipsilateral side-bending and contralateral rotation because these motions place excess stress on the affected structures. Exercise should focus on dynamic stabilization of the trunk. Transverse abdominis activation and isometric multifidus exercises are both appropriate and indicated. Stretching of the hamstrings is indicated as there is often hamstring tightness associated with spondylolisthesis.

Which arterial disease is marked by episodic spasm of small arteries and arterioles with an abnormal vasoconstrictor reflex, exacerbated by exposure to cold or stress? A. Diabetic angiopathy B. Arteriosclerosis obliterans C. Raynaud's disease D. Thrombangiitis obliterans

C. Raynaud's Disease Raynaud's disease is an episodic spasm of small arteries and arterioles. It is marked by abnormal vasoconstrictor reflex exacerbated by exposure to cold or emotional stress. The tips of fingers develop pallor, cyanosis, numbness, and tingling. Diabetic angiopathy is an inappropriate elevation of blood glucose levels and accelerated atherosclerosis. Neuropathy is a major proble. and patients may develoe neurtrophic ulcers. Thromboangiitis obliterans is a chronic, inflammatory vascular occlusive disease of small arteries and also veins. It commonly occurs in young adults who smoke. Patients have paresthesias, pain, cyanotic cold extremities, diminished temperature sensation, fatigue, and risk of ulceration. Arteriosclerosis obliterans is a chronic, occlusive arterial disease of medium-and large-sized vessels. It is the result of peripheral artherosclerosis and is associated with hypertension and hyperlipidemia.

Which form of multiple sclerosis (MS) is characterized by cycles of exacerbation/remission with long periods of stability? A.Progressive-relapsing MS B. Primary-progressive MS C. Relapsing-remitting MS D. Secondary-progressive MS

C. Relapsing-remitting MS Relapsing-remitting MS is characterized by cycles of exacerbation/remission with long periods of stability. These patients may have minimal long-term impairment. In regards to MS, a relapse is a worsening of symptoms. A remission is a period of decreased or no symptoms. Primary-progressive MS progresses from onset with no (or limited) plateaus. Secondary-progressive MS begins as relapsing-remitting MS and turns into progressive MS. Progressive-relapsing MS follows a progressive course with periodic acute relapses. In this type, there is a loss of function and progressive worsening with each exacerbation.

Amyotrophic lateral sclerosis (ALS) is a degenerative disease of the central nervous system. Death usually results in 2-5 years due to what? A. Fall B. Traumatic brain injury C. Respiratory compromise D. Infection

C. Respiratory Compromise ALS is a progressive upper and lower motor neuron disease. It is characterized by muscle fiber atrophy from peripheral nerve involvement. There is degeneration and scarring of the motor neurons in the lateral aspect of the spinal cord, brain stem, and cerebral cortex. Death typically results in 2-5 years after onset due to respiratory compromise. The patient presentation depends on the involvement of the upper and lower motor neurons. Cognition is not affected. Motor function progressively diminishes with eventual total dependency for mobility. The role of physical therapy is to maintain ROM and prevent disuse atrophy, teach energy conservation, maintain functional independence for as long as possible and continuously assess the need for equipment and teach caregiving roles.

The PTA is instructing a patient with a new transtibial amputation in positioning strategies to reduce the likelihood of contractures. Which position would promote contracture development? A. Sitting in a wheelchair with the use of a posterior board behind knee B. Lying supine with legs extended C. Sitting in a wheelchair with the limb dependent and knee flexed D. Prone-lying

C. Sitting in a wheelchair with the limb dependent and knee flexed. The positions to avoid are prolonged flexion and external rotation of the hip as well as knee flexion. Sitting in a wheelchair with the knee flexed would promote the contracture development. The other options are indicated positions that should be included in education, as they will reduce the likelihood of contracture development.

The initial evaluation reveals the patient is rated as Glasgow Coma Scale rating 4. Which behaviors can the physical therapist assistant expect to see? A. Eye opening to verbal command, inappropriate word usage, and decorticate posturing B. Appropriate eye opening, verbal orientation, and motor localization of pain C. Spontaneous eye opening, confused conversation, and motor withdrawal to pain D. Eye opening to pain stimulus, incomprehensible sounds, and decerebrate posturing

C. Spontaneous eye opening, confused conversation, and motor withdrawal to pain The Glasgow Coma Scale is a rating scale from 1-6. Consciousness is related to the patient's degrees of eye opening, motor response, and verbal response. A Glasgow Coma Scale rating of 4 would include spontaneous eye opening, confused conversation, and motor withdrawal to pain. Eye opening to pain stimulus, incomprehensible sounds, and decerebrate posturing is indicative of a Glasgow rating 2. Eye opening to verbal command, inappropriate word usage, and decoricate posturing indicate Glasgow rating 3. Appropriate eye opening, verbal orientation, and motor localization of pain indicate a Glasgow rating 5.

The physical therapist assistant wants to incorporate PNF techniques to promote improved stability at the trunk. Which PNF technique is BEST indicated? A. Contract-Relax B. Rhythmic rotation C. Stabilizing reversals D. Hold-relax

C. Stabilizing reversals Stabilizing reversals are indicated for decreased stability, poor antigravity control, and weakness. It is performed in an alternating pattern , and the agonist and antagonist muscles contract isometrically. In old terminology, it was referred to as alternating isometrics. Hold-relax is a PNF technique that is indicated for limited range of motion resulting from muscle tightness, spasm or pain. Rhythmic rotation is indicated for hypertonia and limited range of motion or function. Contract-relax is indicated for limited range of motion resulting from muscle tightness or spasticity.

The physical therapist assistant is working with a patient who received and began wearing a new AFO two weeks ago. The therapist inspects the skin at the ankle and notices an area of redness with slight blister formation over an area of friction with the new orthotic. What type of pressure ulcer is this? A. Stage I B. Stage III C. Stage II D. Stage IV

C. Stage II Stage II pressure ulcers present as an abrasion or blister, which is what this patient is displaying. Stage I pressure ulcers are non-blanchable redness of intact skin. Stage III pressure ulcers involve a crater that extends through the epidermis and dermis. Stage IV pressure ulcers have full thickness skin loss with extensive destruction and damage.

Which ambulatory aid is BEST indicated for a 60-year-old patient who has undergone an orthopedic surgery on his tibia, has poor balance, and has restriction for non-weight bearing status of his surgical limb? A. Single point cane B. Forearm crutches C. Standard walker D. Axillary crutches

C. Standard walker. Due to the patient's poor balance, the standard walker would be best indicated for ambulation. The walker provides increased lateral and anterior stability and allows for weight bearing through the upper extremities to maintain non-weight bearing status of the lower extremity. The axillary crutches would not be indicated because of the patient's poor balance, although they are appropriate for conditions of non-weight bearing status. The forearm crutches would not provide enough stability for non-weight bearing . The single point cane is not indicated for weight bearing restrictions.

A physical therapist assistant is working with an individual with Parkinson's disease. He displays difficulty with ambulation in freezing episodes and difficulty transitioning over thresholds. Which intervention would BEST address this limitation? A. Seated balance activity over the exercise ball B. Seated long arc quads and repeated sit-to-stand C. Standing marches and cone tapping with the lower extremity D. Passive stretching for trunk rotation

C. Standing marches and cone tapping with the lower extremity This intervention will help improve the patient's ability to shift weight and lift the lower extremity during gait. Cone tapping will work on coordination and the ability to lift the foot over a threshold. Balance activities over an exercise ball will help core strength and balance but not assist in specific gait deficits. Seated strengthening and repeated sit-to-stand will help in overall strength but not specific gait limitations. Passive trunk rotation will help with trunk rotation and gait but not foot advancement.

When analyzing the gait of a patient with a below-knee amputation, the PTA notices excessive knee flexion in initial contact of early stance. What is a likely contributing factor for this deviation? A. Excessive foot inset B. Soft heel cushion or low shoe heel C. Stiff heel cushion or high shoe heel D. Excessive foot outset

C. Stiff heel cushion or high shoe heel If the patient displays excessive knee flexion during initial contact of early stance, then the prosthetic can be the cause. This can occur due to a high shoe heel or a stiff heel cushion, if the socket is too far anterior or excessively flexed or if there is sufficient plantar flexion. The anatomical causes of this deviation include a knee flexion contracture and weakness in the quadriceps muscle. A soft heel cushion or low shoe heel will lead to insufficient knee flexion. Excessive foot inset will lead to excessive lateral thrust in midstance. Excessive foot outset will lead to excessive medial thrust in midstance.

In the acute (inflammatory) phase of a musculoskeletal condition, which intervention is NOT appropriate? A. Assisted movement of injured tissues B. Grade 1 and 2 joint oscillations C. Stretching of involved tissues D. Compression of the involved tissues

C. Stretching of involved tissues Stretching the involved tissues is contraindicated due to the weakness of the newly formed tissue. However, PRICE (protect, rest, ice, compression, elevation) should be utilized. Modalities, rest, relaxation, and soft tissue massage are indicated. Joint oscillation (grade 1 and 2) are indicated for pain relief. Assisted movement of injured structures will protect the soft tissue.

Regarding the cardiovascular hemodynamics, which is the term to describe the amount of blood ejected with each myocardial contraction? A. Cardiac output (CO) B. Ejection fraction (EF) C. Stroke volume (SV) D. Left ventricular end-diastolic pressure (LVEDP)

C. Stroke volume (SV) Stroke volume (SV) is the amount of blood ejected with each myocardial contraction, which is approximately 70 mL. Cardiac output (CO) is the volume of blood discharged from the left or right ventricle per minute, which is approximately 4-6 L per minute in a typical adult. Left ventricular end-diastolic pressure (LVEDP) is the pressure in the left ventricle during diastole. Ejection fraction (EF) is the percentage of blood emptied from the ventricle during systole.

A patient is one day post-operative from left total hip arthroplasty and the PTA is treating the patient in the hospital setting. Which exercise intervention is NOT appropriate to initiate strength of the lower extremities? A. Seated long arc quad B. Gluteal sets C. Supine active hip flexion, bringing knee to chest D. Supine hip abduction

C. Supine active hip flexion, bringing knee to chest After total hip replacement, patients should avoid the position of hip flexion past 90 degrees with adduction and internal rotation. Bringing knee to chest with active hip flexion would break the hip precaution of no hip flexion past 90 degrees. The other options are appropriate strengthening interventions for this patient.

A patient is one day post-op right total knee arthroplasty in a hospital setting. Which is an appropriate intervention to assign the patient to begin improving knee range of motion? A. Gluteus sets B. Wall squats C. Supine heel slides D. Forward lunges

C. Supine heel slides Supine heel slides promote knee flexion and extension range of motion and are appropriate in the acute stage of healing after a total knee arthroplasty. Forward lunges and wall squats are too strenuous for one day post-op. Gluteus isometric sets work to increase strength in the gluteal muscles and do not facilitate range of motion at the knee.

What type of wound debridement is indicated for a patient with a deep (stage III or IV) pressure ulcer with necrotic tissue and infection that is placing the patient's life at risk? A. Autolytic B. Enzymatic C. Surgical D. Mechanical

C. Surgical Surgical debridement is indicated for patients with deep or complicated pressure ulcers. It is the most efficient method of debridement and performed by a surgeon or physician in an operative procedure. This procedure removes most or all necrotic tissue. It is indicated when infection threatens the individual's life. The other options are not efficient enough methods for debridement due to the patient's infection.

Which of the following hand deformities is observed as flexion of the metacarpophalageal and distal interphalangeal joints with extension of the proximal interphalageal joint? A. Boutonniere defomity B. Mallet finger C. Swan neck deformity D. Ape hand deformity

C. Swan neck deformity A swan neck deformity results from contracture of intrinsic muscles with dorsal subluxation of lateral extensor tendons. The observed deformity is flexion of the metacarpophalangeal and distal interphalangeal joint with extension of the proximal interphalangeal joint. It is a result of trauma or rheumatoid arthritis. An ape hand deformity is observed as thenar muscle wasting, with the first digit moving dorsally in line with the second digit. It is a result of median nerve dysfuction. A mallet finger deformity is due to a rupture or avulsion of the extensor tendon at its insertion into the distal phalanx of a digit. The observed deformity is flexion of the distal interphalageal joint. It is usually a result of trauma. A boutonniere deformity results from the rupture of the central tendinous slip of the extensor hood. The metacarpophalangeal and distal interphalangeal joint with flexion of proximal interphalangeal joint. It is a result of trauma or rheumatoid arthritis.

Which specialized orthotic device is indicated to provide mild control for excessive hyperextension of the knee? A. Neoprene sleeve B. Craig-Scott KAFO (knee-ankle-foot-orthosis) C. Swedish knee cage D. Patellar stabilizing brace

C. Swedish knee cage The Swedish knee cage provides mild control for excessive knee hyperextension. It is hinged at the knee to track motion. A neoprene sleeve provides compression, protection, and proprioceptive feedback to the knee. A patellar stabilizing brace improves patellar tracking to maintain alignment. A Craig-Scott KAFO is commonly used for individuals wih paraplegia.

Which gait pattern is typically indicated for a patient with paraplegia or spina bifida? A. two-point gait with a cane B. modified three-point gait with canes C. Swing-to pattern with Lofstrand crutches D. Three-point gait with axillary crutches

C. Swing-to pattern with Lofstrand crutches A swing-to pattern is used when both lower extremities are involved. Both crutches are advanced forward together, weight is shifted on the hands for support, and both legs are swung or dragged forward to meet the crutches. This type of gait is indicated for patients with paraplegia, spina bifida, and when the trunk is unstable. A two-point gait pattern allows for natural arm and leg gait motion. With two-point gait, one assistive device and opposite extremity move together. Three-point gait is indicated for weight bearing restrictions for one lower extremity. Both assistive devices and involved leg are advanced together and then the uninvolved leg advances to or or past the assistive devices. A modified three-point gait is like three point but allows for some weight bearing through involved lower extremity.

An amputee wheelchair is modified to accommodate for patients with bilateral lower extremity amputations and altered center of gravity. Where are the drive wheels positioned? A. The drive wheels are positioned anterior to the vertical back supports B. The drive wheels are positioned in the same line as the vertical back supports C. The drive wheels are positioned posterior to the vertical back supports D. There is no modification required for the drive wheels

C. The drive wheels are positioned posterior to the vertical back supports Patients with bilateral lower extremity amputations have center of gravity located more posterior when seated in the wheelchair. The amputee wheelchair is modified by placing the drive wheels posterior to the vertical back supports, typically two inches backward. This feature increases the length of the base of support and posterior stability.

Which of the following BEST describes a gross motor behavior that would be established by 23 months of age? A. Hops on one foot B. Catches a large ball C. Throws a ball in standing D. Rides a tricycle

C. Throws a ball in standing Typically by 16-23 months, a child is able to throw a ball in standing. Riding a tricycle, catching a large ball, and hopping on one foot are all gross motor tasks not established typically until 24-36 months of of age.

Which mechanical agent is indicated if the therapist has a goal to gradually load the extremities, facilitate active head and trunk control, and facilitate postural drainage? A. Mechanical spinal traction B. Intermittent mechanical compression C. Tilt table D. Continuous passive motion

C. Tilt table A tilt table is an electrical or mechanical table designed to elevate a patient from a horizontal position to a vertical position in a controlled and incremental manner. It is indicated for patients who have be on prolonged bed rest, immobilization, with a spinal cord injury, traumatic brain injury, spasticity, and orthostatic hypotension. Using a tilt table can stimulate postural reflexes, facilitate postural drainage, load the extremities, facilitate head and trunk control and provide positioning for stretching lower extremities. Continuous passive motion is typically utilized for knee conditions or surgery in which the continuous passive motion device passively facilitates range of motion. Intermittent mechanical compression uses a pneumonic device applying external pressure to an extremity and is indicated for lymphatic and venous return systems. Mechanical spinal traction is indicated for vertebral traction to relieve pain.

What term describes a taut area of muscle that is tender to palpation and can refer pain to other parts of the body? A. Lymph node B. Muscle spindle C. Trigger point D. Motor point

C. Trigger point A trigger point is a taunt area of muscle also known as a muscle knot. It is tender to palpation and can refer pain to other parts of the body. Trigger point release is a massage technique in which the clinician uses a fingertip, knuckle, elbow, or trigger point release tool directly over the trigger point. The clinician places pressure to reproduce the pain and holds the pressure until the tension is released.

Which massage technique describes a shaking of tissue using rapid, quivering motion with the hands, which is used for postural drainage and to loosen adherent secretions? A. Tapotement B. Effleurage C. Vibration D. Petrissage

C. Vibration Vibration is often used in conjunction with cupping in order to promote postural drainage and loosen adherent secretions. It uses shaking of tissue with short, rapid quivering motions. Effleurage uses gliding movement of the hands over the surfaces of the skin. Tapotement is rapid striking of the surface of the palm, cupped hand, or edge of hand over the patient's skin. Petrissage uses kneading strokes to grasp and lift tissue.

Which physical property of water is utilized when a patient incorporates a paddle and increases his speed of movement in the water in order to increase resistance and drag for greater strength and endurance challenge? A. Density B. Hydrostatic pressure C. Turbulence D. Buoyancy

C. Turbulence Turbulence describes that the movement of a body part through water creates circular motion of the water near the surface of the part, producing a frictional drag. As the speed of movement increases, the resistance is increased. Using a paddle or boot increases resistance and drag, which is more challenging to the patient. Hydrostatic pressure is the circumferential water pressure exerted on an immersed body or body part. A pressure gradient is established between the surface water and deeper waters due to the increased density in the deeper water. Increased pressure reduces edema and enhances peripheral circulation. Buoyancy is the upward force of water on an immersed (or partially immersed) body or body part. It is equal to the weight of the water that is displaces. The buoyancy creates an apparent decrease in body weight and joint unloading, which allows for easier movement in the water. Density is the mass per unit volume of a substance. The density of water is proportional to its depth. The deeper the water, the greater the density. This can be incorporated to challenge or regress exercise.

A patient presents with nerve entrapment after a fall onto his elbow. He experiences pain at the medial aspect of his elbow and paresthesia into his 5th finger. Which nerve is entrapped? A. Tibial nerve B. Median nerve C. Ulnar nerve D. Radial Nerve

C. Ulnar Nerve Ulnar nerve entrapment is caused by direct trauma to the cubital tunnel, traction due to thickening of the retinaculum, hypertrophy of flexor carpi ulnaris, or DJD of the cubital tunnel. Symptoms include medial elbow pain and paresthesia into digits 4 and 5. Median nerve entrapment occurs within the pronator teres muscle and under the superficial head of the flexor digitorum superficialis. It occurs with repetitive gripping activities and sporting activities. Clinical signs include aching pain with weakness in forwarm muscles and paresthesia into digits 1-4. Radial nerve entrapment occurs in the distal branches within the radial tunnel due to over head activities and throwing. Clinical signs include lateral elbow pain, pain over supinator muscle, and paresthesia into the posterior aspect of the forearm. The tibial nerve is not a viable option, as the tibial nerve innervates the lower leg, and this question describes symptms in the upper extremity.

The physical therapist assistant is treating an 11-year-old girl who has pain and limited knee range of motion following repair of a posterior cruciate ligament. The plan of care includes modalities for pain relief and soft tissue healing. Which modality is NOT indicated? A. TENS B. Interferential current C. Ultrasound D. Ice pack

C. Ultrasound Ultrasound is contraindicated in patients who have active epiphyseal plates. The epiphyseal plates close after puberty. Therefore, ultrasound should be avoided for this patient. The other physical agents are appropriate to use and could improve pain control and soft tissue healing.

The physical therapist assistant is working with a patient that has class I congestive heart failure. What is the maximal METS for physical activity for this patient? A. Up to 1.5 METS B. Up 4.5 METS C. Up to 6.5 METS D. Up to 3 METS

C. Up to 6.5 METS Patients with congestive heart failure (CHF) demonstrate significant ventricular dysfunction, decreased cardiac output, and low functional capacities. CHF can be classified as class I-IV. Class I: Mild CHF; no limitation on physical activity, up to 6.5 METS. Patients are comfortable at rest; ordinary activity does not cause undue fatigue, dyspnea, or angina pain. Class II: Mild CHF; slight limitation in physical activity, up to 4.5 METS; Patients are comfortable at rest, and ordinary physical activity results in fatigue, palpitation, dyspnea, or angina pain. Class III: Moderate CHF; marked by limitation of physcial activity, up to 3 METS. Patients are comfortable at rest , but less than ordinary activities cause fatigue, palpitation, dyspnea, or angina pain. Class IV: Severe CHF; patients are unable to carry out any physical activity, 1.5 METS, without discomfort. Symptoms of ischemia, dyspnea, or angina pain are present even at rest and symptoms increase with exercise.

Which resistance training intervention would achieve the goal of improving muscular endurance? A. Using 90% of 1 RM and performing 1 set of 8 repetitions B. Using 80% of 1 RM and performing 2 sets of 12 repetitions C. Using 40% of 1 RM and performing 3 sets of 20 repetitions D. Using 75% of 1 RM and performing 3 sets of 10 repetitions

C. Using 40% of 1 RM and performing 3 sets of 20 repetitions Muscular endurance is the ability of an isolated muscle group to perform repeated contractions over time. To improve muscular endurance, low-load resistance exercise for many repetitions is performed. Using 40% of a patient's 1 repetition maximum is a low-load and performing 3 sets of 20 repetitions is a high amount of repetitions. Options B and D will both achieve a goal of muscle hypertrophy and strength. Option A will achieve maximum strength.

A PTA is providing initial gait training for a patient learning to walk with a hemi-walker after a stroke. What is the appropriate feedback? A. Verbal feedback is not given B. Verbal feedback is summed after four trials C. Verbal feedback after every trial D. Verbal feedback summed after 10 trials

C. Verbal feedback after every trial Feedback given frequently (after every trial) improves initial performance. In this example, frequent feedback is necessary to ensure proper sequencing and safety with ambulation. Feedback given less frequently(summed after a given number of trials or fading with decreasing frequency) improves retention of skills.

Which massage technique describes a shaking of tissue using rapid, quivering motion with the hands, which is used post postural drainage and to loosen adherent secretions? A. Tapotement B. Effleurage C. Vibration D. Petrissage

C. Vibration Vibration is often used in conjuction with cupping in order to promote postural drainage and loosen adherent secretions. It uses shaking of tissue with short, rapid quivering motions. Effleurage uses gliding movement of the hands over the surface of the skin. Tapotement is rapid striking of the surface of the palm, cupped hand, or edge of hand over patient's skin. Petrissage uses kneading strokes to grasp and lift tissue.

Which exercise is NOT an example of an open chain exercise? A. Sidelying hip abduction B. Short arc quad C. Wall squats D. Bicep curl with hand weight

C. Wall squats Wall squats are a closed chain exercise, as the body is moving over a fixed distal segment. In this case, the body is lowered and the feet are fixed to the ground. This strengthens the lower extremities. The other options are all open chain exercises. An open chain exercise is one in which the distal segment (hand or foot) moves freely in space.

In regards to wound assessment, what are the characteristics of wound exudate described as serous? A. Containing blood B. Containing eschar C. Watery serum D. Containing pus

C. Watery serum Wound exudate type should be determined in the assessment of wounds. Serous drainage is a watery serum. Sanguineous drainage contains blood. Purulent drainage contains pus. Eschar is dried necrotic tissue.

During the subacute phase of healing from a musculoskeletal condition, the patient will experience pain: A. At rest and with activities that stress the healing tissues B. Only with resisted movements C. With activities that overstress the healing tissue D. There will be no pain

C. With activities that overstress the healing tissue During the subacute phase, a patient should only have pain with activities that overstress healing structures. Patients in the acute phase often have pain at rest due to inflammation. These patients are in maximal protection phase for healing. During the functional restoration phase, patients should not have pain and should gradually return to previous activities.

A physical therapist assistant is working with a patient who has a rotator cuff injury. The plan of care includes "strengthening of shoulder complex" as part of the rehabilitation plan. The physical therapist reads the previous treatment note and finds that the assistant has progressed the exercise program to include strengthening with a resistance band. The physical therapist assistant is: A. Not working within the scope of practice B. In need of calling the physical therapist before adding resistance strengthening C. Working within the scope of practice D. Not allowed to progress exercises in this case

C. Working within the scope of practice The physical therapist assistant is able to progress a patient through an exercise program if it included in the plan of care by the evaluating physical therapist. If the plan of care detailed specific exercises, modalities, or activities, then the assistant would have needed consent from the physical therapist to modify. The assistant is not working outside the scope of practice in this case.

What skin color change is common with an individual who has liver disease? A. Blue B. Brown C. Yellow D. Cherry Red

C. Yellow A yellow color indicates jaundice or liver disease. It can also be associated with increased carotene intake. Cyanosis is a slighty bluish, grayish discoloration. It is indicative of a lack of oxygen and can indicate cogestive heart failure, advanced lung disease, congenitalheart disease, and venous obstruction. This is commonly seen as cyanosis of the lips, oral mucosa, tongue or nails, hands, and feet. Cherry red is indicative of carbon monoxide poisoning. Brown color indicates increased pigmentation associated with venous insufficiency.

A physical therapist assistant has just completed instructing a caregiver on how to transfer a patient from a bed to wheelchair and back to bed using a slide board. The assistant should document the caregiver's performance and ability to complete the task in safely in which section of the SOAP note? A. subjective B. objective C. assessment D. plan

C. assessment The assessment section is the appropriate area for documentation because the assistant is using multiple pieces of information to interpret the caregiver's ability to safely perform the transfer. Subject information is provided by the patient or caregiver. Objective data includes measurements, distances, and objective description of the technique. Plan includes identification of further training needed or plan for upcoming sessions.

When utilizing locomotor training with body weight support (BWS) and a motorized treadmill, which percentage of body weight support is contraindicated? A. 40% B. Less than 10% C. Less than 20% D. Greater than 55%

D. Greater than 55% An overhead harness is used to support the patient's body weight while stepping is facilitated over a motorized treadmill. Initially, support is high (40% of body weight) and then the support decreases progressively (30%, 20%, 10% to no body weight support). Body weight support greater than 55% is contraindicated because it negatively interferes with the gait cycle, as the patient is unable to achieve flat foot during stepping.

When utilizing intermittent mechanical compression over the full lower extremity, which portion of the sleeve has the GREATEST pressure? A. In the proximal compartments B. It alternates between distal and proximal compartments C. There is no pressure gradient D. In the distal compartments

D. In the distal compartments The compression units are designed to initiate and create pressure over the extremity. The pressure is greater in the distal compartments and lesser in the proximal compartments. This pressure creates increased pressure on the interstitial fluids, promoting lymphatic and venous return and reducing volume in the extremity.

You are treating a 45-year-old patient with a 4-month history of pain related to piriformis syndrome, with palpable piriformis muscle spasm. He has no contraindications or precautions for ultrasound modality, and it is included in the plan of care. Which is the appropriate setting for an ultrasound for this patient? A. 1 MHz pulsed at 1.0 w/cm.2 B. 3 MHz continuous at 3.0 w/cm.2 C. 3 MHz continuous at 1.0 w/cm.2 D. 1 MHz continuous at 1.0 w/cm.2

D. 1 MHz continuous at 1.0 w/cm.2 This patient requires ultrasound that will produce thermal effects to properly address the chronic pain and muscle spasm. 1 MHz of continuous ultrasound provides deep heating to 3-5 cm. Continuous ultrasound achieves thermal effects for chronic pain. The other options are incorrect because pulsed ultrasound is used for non-thermal effect, for acute cases. Ultrasound setting at 3 MHz would only address superficial soft tissue structures, and the piriformis is a deeper muscle.

What is the usual suctioning time when performing endotracheal suctioning for airway clearance? A. 30-40 seconds B. 20-30 seconds C. 50-60 seconds D. 10-15 seconds

D. 10-15 seconds Endotracheal suctioning is used only when other airway clearance techniques are not adequate. A catheter is fed into an artificial airway, oral airway, or the nares through the pharynx, larynx, and to the carina. When resistance is felt at the carina, the catheter is rotated and withdrawn. Suction is applied intermittently to avoid damage to the inner trachea. The typical suctioning time is 10-15 seconds.

Which patient would require the MOST concentration on safety measures and safety training with both patient and caregivers? A. 60-year-old female with left hemisphere lesion and right-sided hemiplegia, primarily impacting her lower extremity B. 85-year-old male with left hemisphere lesion and right-sided hemiplegia, primarily impacting his upper extremity C. 80-year-old male with right hemisphere lesion and left-sided hemiplegia, primarily impacting his upper extremity D. 80-year-old male with right hemisphere lesion and left-sided hemiplegia, primarily impacting his lower extremity

D. 80-year-old male with right hemisphere lesion and left-sided hemiplegia, primarily impacting his lower extremity Patients with right hemisphere lesions and left-sided hemiplegia tend to be impulsive, quick moving and overestimate their functional ability. Additionally, these patients tend to use poor judgement which leads to safety concerns. Patients with left hemisphere lesions and right-sided hemiplegia tend to be slow, cautious, insecure, and hesitant. Therefore, the patients with left-sided lesions can be eliminated from choices, as they would likely be cautious with their mobility. Of the two remaining choices with right hemisphere lesions, the patient who has more impact on the lower extremity would be at greater safety risk as compared with upper extremity due to the role of legs in walking, mobility, and balance.

For a patient with upper extremity involvement after stroke, which secondary musculoskeletal condition is LEAST likely? A. Impingement B. Adhesive capsulitis C. Shoulder subluxation D. A SLAP lesion

D. A SLAP lesion A SLAP lesion is a tear of the superior labrum, anterior to posterior. It commonly occurs in athletes with repeated overhead motions or in trauma. Typically, this does not occur to individuals s/p stroke. Subluxation is common, especially for individuals with flaccidity. It is caused by forces of gravity or traction, weakness of shoulder capsule, and shoulder musculature. Immobility can lead to adhesive capsulitis. Poor dynamic stabilizers can lead to impingement.

Regarding evidence-based practice, which is the correct definition for sensitivity? A. The change or difference in the behavior that is the result of the intervention B. The factor believe to bring about the change C. A measure of the test's ability to correctly identify the proportion of those who do not have the condition, impairment, or disease being measured D. A measure of the test's ability to correctly identify the proportion of those who truly have the condition, impairment, or disease being measured

D. A measure of the test's ability to correctly identify the proportion of those who truly have the condition, impairment, or disease being measured Sensitivity is a measure of true positive. It measures the test's ability to correctly identify those who truly have the condition, impairment, or disease being measured. Sensitivity should be analyzed for standardized outcome measurement tools. Specificity is a measure of a true negative. It measures the test's ability to correctly identify those who do not have the condition, impairment, or disease being measured. An independent variable is the factor believed to bring about the change in the dependent variable (the cause or treatment). The variable is the change or difference thatis the result of the intervention.

Regarding evidence-based practice, which is the most correct definition for sensitivity? A. The change or difference in the behavior that is the result of the intervention B. The factor believed to bring about the change C. A measure of the test's ability to correctly identify the proportion of those who do not have the condition, impairment, or disease being measured D. A measure of the test's ability to correctly identify the proportion of those who truly have the condition, impairment, or disease being measured.

D. A measure of the test's ability to correctly identify the proportion of those who truly have the condition, impairment, or disease being measured. Sensitivity is a measure of true positive. It measures the test's ability to correctly identify those who truly have the condition, impairment, or condition being measured. Sensitivity should be analyzed for standardized outcome measurement tools. Specificity is a measure of a true negative. It measures the test's ability to correctly identify those who do not have the condition, impairment, or disease being measured. And independent variable is the factor believed to bring about the change in the dependent variable, the cause or treatment. The dependent variable is the change or difference that is the result of the intervention.

A PTA is providing strengthening intervention for a patient with a painful arc of motion from chronically inflamed shoulder abductors. Which is the MOST efficient and appropriate exercise type? A. Isometric contraction at the end range B. Active concentric contraction in the full range C. Active concentric contraction in the pain-free range D. Active eccentric contraction in the pain-free range

D. Active eccentric contraction in the pain-free range For chronically inflamed musculature, the therapist should focus on working in a pain-free range of motion. Additionally, active eccentric contraction requires less stress on the contractile units in comparison to active concentric contractions. The other options could cause further inflammation of the muscle and are incorrect choices.

During which phase of healing will the patient experience pain at rest? A. Functional restoration phase B. Both acute and subacute phases C. Subacute phase D. Acute (inflammatory) phase

D. Acute (inflammatory) phase Patients in the acute phase often have pain at rest due to inflammation. These patients are in maximal protection phase for healing. During the subacute phase, a patient should only have pain with activities that overstress healing structures. During the functional restoration phase, patients should not have pain and should gradually return to previous activities.

Which types of wound dressings are characterized as a soft, absorbent, nonwoven dressings derived from seaweed and often used for wounds with moderate to large amounts of exudate? A. Hydrogels B. Hydrocolloids C. Foams D. Alginates

D. Alginates Hydrogels are water or glycerine-based gels insoluble in water. They are available as solid sheets, amorphous gels, or impregnated gauze.They are used for partial and full thickness wounds and those with necrosis and slough. They are soothing and cooling to the patient and rehydrate dry wound beds. Foams are semipermeable membranes that are either hydrophilic or hydrophobic.They vary in thicknes, absorptive capacity, and adhesive properties.They are indicated for partial and full thickness wounds with minimal or moderate exudate. Hydrocolloids are adhesive wafers that contain absorptive particles that interact with would fluid to form a gelatinous mass over the wound bed.They can be occlusive or semi occlusive. They are indicated for protection of partial thickness wounds and wounds with mild exudate, maintaining a moist wound environment

The physical therapist assistant is working to improve dynamic standing balance reactions in a patient who has multiple sclerosis. The patient is standing on a tile floor and stepping over 4-inch cones without difficulty. What is an appropriate progression to this activity? A. Place patient in sitting and perform cone taps B. Decrease the height of the cones to 2 inches C. Provide the patient with a balance bar for upper extremity support D. Ambulate over foam mats instead of tile floor

D. Ambulate over foam mats instead of tile floor In order to progress this exercise, the therapist can decrease the stability of the support surface. This would mean progressing from a level surface of the tile floor to an unstable surface of a foam mat. The other options are all ways to regress the exercise. This is performed by decreasing the number of segments involved, increasing the stability of the support surface, and decreasing the range of movement involved.

A long-distance runner is referred to physical therapy for pain in her bilateral lower limbs. She complains of tightness and a deep cramping paining in the region of the anterior tibialis muscles, which is chronic and made worse when she runs. What is the likely condition? A. Medial tibial stress syndrome B. Tibial plateau fracture C. Anterior tibial periostitis D. Anterior compartment syndrome

D. Anterior compartment syndrome Anterior compartment syndrome is a condition of increased compartmental pressure resulting in a local ischemic condition. It can be a result of direct trauma, fracture, overuse, and muscle hypertrophy. Symptoms can be chronic or brought on by exercise and are described as a deep cramping feeling. Acute ACS results after sudden trauma causing swelling in the compartment (with swelling, paresthesia, and severe pain) and is a medical emergency. Anterior tibial periostitis (also knwn as shin splints) is a musculotendinous overuse condition. It results from abnormal biomechanical alignment, poor conditioning, and improper training. Muscles involved include the anterior tibialis and extensor hallucis longus. Pain is elicited when palpating the lateral tibia and anterior compartment. Symptoms are a nagging pain on the inner edge of the shin, which is made worse with ankle dorsiflexion. Medial tibial stress syndromeis an overuse injury of the posterior tibialis and/or medial soleus resulting in periosteal inflammation at the muscular attachments. It is a result of excessive pronation and pain is elicited with palpation of the distal posteromedial border of the tibia. A tibial plateau fracture commonly occurs with a valgum and compression force to the knee when the knee is flexed. It often occurs with a medial collateral ligament injury.

Which hand deformity is the result of median nerve dysfunction? A. Boutonniere deformity B. Mallet finger C. Swan neck deformity D. Ape hand deformity

D. Ape hand deformity Ape hand deformity is observed as thenar muscle wasting, with the first digit moving dorsally in line with the second digit. It is a result of median nerve dysfunction. A mallet finger deformity is due to a rupture or avulsion of the extensor tendon at its insertion into the distal phalanx of the digit. The observed deformity is flexion of the distal interphalangeal joint. It usually is a result of trauma. A swan neck deformity results from contracture of intrinsic muscles with dorsal subluxation of lateral extensor tendons. The observed deformity is flexion of the metacarpophalangeal and distal interphalangeal joint with extension of the proximal interphalangeal joint. It is a result of trauma or rheumatoid arthritis. A boutonniere deformity results from the rupture of the central tendinous slip of the extensor hood. The observed deformity is extension of the metacarpophalangeal and distal interphalangeal joint with flexion of the proximal interphalangeal joint. It is a result of trauma or rheumatoid arthritis.

The physical therapist assistant is working with a patient with dysphagia. Upon assessing the oxygen saturation, the therapist finds it to be on 89% with an increased heart rate and noticeable cough. The patient complains of mild chest pain over the right side of her chest. The likely cause is: A. Heart attack B. Tuberculosis C. Chronic obstructive pulmonary disease D. Aspiration pneumonia

D. Aspiration pneumonia The patient has know dysphagia, which means she has impaired swallowing. The symptoms of aspiration pneumonia typically occur within hurs of the aspiration event. The symptoms include dry cough at onset progressing to productive cough, dyspnea, tachypnea, cyanosis, tachycardia, wheezes and crackles, hypoxemia, hypercapnea, chest pain over the involved area, fever, white blood cell count showing a varying degree of leukocytosis, and chest X-ray that shows pneumonitis.

When examining the integumentary system of a patient, it is important to understand risk factors for cancerous cell activity. Clinical examination of malignant melanoma can use the acronym ABCDEs. What does this stand for? A. Asymmetry, Bleeding, Cool, Dark, Elevation B. Active, Bleeding, Color, Diameter, Elevation C. Asymmetry, Brown, Cool, Diameter, Elevation D. Asymmetry, Border, Color, Diameter, Elevation

D. Asymmetry, Border, Color, Diameter, Elevation Asymmetry: look for uneven edges, lopsided appearance Border: look for an irregular border, poorly defined edges Color: look for variations in color, especially mixtures of black, blue, and red Diameter: larger than 6 mm Elevation: usually elevated, but may be flat as well

Which assistive device would be BEST indicated for a healthy 26-year-old male with left ankle fracture who is non-weight bearing in his left lower extremity? A. Single point cane B. Forearm crutches C. Hemi walker D. Axillary crutches

D. Axillary crutches Axillary crutches are intended to provide increased weight bearing through upper extremities, which is indicated for a condition of non-weight bearing restriction for the lower extremity. The crutches provide a moderate degree of stability. Forearm crutches are not indicated because they provide less stability than axillary crutches, which would make it difficult to maintain a non-weight bearing restriction. Neither a single point cane nor a Hemi walker would allow for mobility with the non-weight bearing restriction.

Which type of traumatic nerve injury is a crush injury? A. Entrapment B. Neurotmesis C. Neurapraxia D. Axonotmesis

D. Axonotmesis Axonotmesis is a class-2 traumatic nerve injury. It is an injury to nerve interrupting the axon and causing loss of function and degeneration distal to the lesion such as a crush injury. Regeneration is possible. Neurapraxia is a class-1 traumatic nerve injury. it is an injury to the nerve that causes a transient loss of function like a compression. Nerve dysfunction may be rapidly reversed or persist a few weeks. Neuromesis is a class-3 traumatic nerve injury. It is characterized by cutting of the nerve with severance of all structures and complete loss of function. Re-innervation fails without surgical intervention. Entrapment is a condition of pressure on a nerve where it passes over a bony prominence or restricted opening. It results in motor and sensory disturbances in the area of nerve distribution.

Which reflex is elicited with stroking the lateral plantar surface of an infant's foot? A. Moro B. Galant C. Rooting D. Babinski

D. Babinski The Babinski reflex is elicited with stroking to the lateral plantar surface of the foot. Upon stimulation, the great toe extends and the other toes fan out.

The initial evaluation notes that a patient has significant weakness in the gluteus maximus. How will this weakness demonstrate itself in a gait deviation? A. Forward trunk lean B. Lateral trunk lean to the same side C. Lateral trunk lean to the opposite side D. Backward trunk lean

D. Backward trunk lean A backward trunk lean is associated with weakness in the gluteus maximus. This patient will also demonstrate difficulty going up stairs and ramps. Weakness in the gluteus medius muscle will cause a gait deviation of lateral trunk bending to the same side. A lateral trunk lean to the opposite side is associated with a weak hip flexor. A forward trunk lean is associated with weakness in the quadriceps or hip/knee flexion contracture.

What temperature are hot packs generally heated to in a water-immersion tank? A. Between 100-105 degrees F B. Between 125-130 degrees F C. Between 145-150 degrees F D. Between 165-170 degrees F

D. Between 165-170 degrees F A hot pack is a canvas pack filled with silica gel and heated by immersion in water between 165 and 170 degrees Fahrenheit. The method of heat transmission is conduction. The clinician should add 6-8 layers of toweling between the patient and the hot pack to protect to skin.

A patient with emphysema has developed respiratory alkalosis. What signs should the therapist monitor for? A. Stupor B. Hyperventilation cyanosis C. Headache D. Blurred vision

D. Blurred vision A patient with respiratory alkalosis, which is diminished carbon dioxide and alveolar hyperventilation. Signs include tachypnea, dizziness, anxiety, numbness and tingling, blurred vision, diaphoresis, muscle cramps, twitching, weakness, arrhymias and convulsions. The other options are signs of respiratory acidosis, which is characteristic of carbon dioxide retention and impaired alveolar ventilation. Signs include dyspnea, hyperventilation cyanosis, restlessness, headache, and it can lead to disorientation, stupor, or coma, and death.

Which condition is an obstructive pulmonary disease that occurs in premature infants who have been on high pressures of mechanical ventilation? A. Respiratory distress syndrome B. Brochiectasis C. Cystic fibrosis D. Bronchopulmonary dysplasia

D. Brochopulmonary dysplasia Bronchopulmonary dysplasia is an obstructive pulmonary disease. It is often a sequela of premature infants with respiratory distress syndrome, who were given high pressures of mechanical ventilation, or infection. Lungs show pulmonary immaturity and hyperinflation. Respiratory distress syndrome is a condition of alveolar collapse in a premature infant. It is a result of lung immaturity and inadequate levels of pulmonary surfactant. Physical findings are within a few hours of birth and include respiratory distress, crackles, tachypnea, hypoxemia, cyanosis, accessory muscle use, expiratory grunting, and flaring nares. Brochiectasis is a chronic congenital or acquired disease characterized by abnormal dilation of the bronchi and excessive sputum production. Cystic fibrosis is a genetically inherited disease that affects the mucus-producing glands of the lungs, sweat glands, digestive tract, and genitourinary system. It is characterized by thickening of secretions of all exocrine glands, leading to obstruction.

At which level would a patient with a spinal cord injury still maintain the ability to self-feed? A. C2 B. C3 C. C4 D. C5

D. C5 At nerve root level C5, the biceps and deltoid remain intact. The patient has elbow flexion, shoulder external rotation and shoulder abduction to 90 degrees intact. Therefore, the patient is able to self-feed and perform some self-care with UE assistive devices. At level C4, the diaphragm and trapezius remain intact, and the patient still has respiration and scapular elevation available but would not be able to self-feed. At C1-C3, the key movements available are only of the mouth and head including talking, chewing, sipping and blowing, but the patient would not be able to self-feed

The therapist is working with a patient with known spina bifida who has a ventriculoperitoneal shunt. The child begins vomiting and shows decreased coordination and significant behavior change during the session. What is the MOST appropriate action? A. Discontinue therapy and reschedule for a future date B. Discontinue therapy and inform the parent to monitor this symptoms C. Discontinue the session and give the child water to prevent dehydration D. Call 911 for emergency medical services

D. Call 911 for emergency medical services Shunt precautions include: avoiding placing pressure on the shunt , avoiding stretching of the neck, and not placing the individual in Trendelenburg's position. Signs of shunt malfunction include: irritability, vomiting, fontanel bulging, lethargy, headache, and changes in behavior, coordination, or seizure activity. Shunt malfunction is a medical emrgency due to risk of increased cerebrospinal pressure. So, 911 should be called for emergency medical care.

Regarding neoplasms, which type of malignant tumor originates from the epithelial tissues? A. Sarcoma B. Leukemia C. Lymphoma D. Carcinoma

D. Carcinoma A carcinoma is a malignant tumor that originates from epithelial tissues, such as the skin, stomach, colon, breast, and rectum. A sarcoma is a malignant tumor that originates in connective and mesodermal tissues, such as the muscle, bone, or fat. A lymphoma is a malignant tumor that affects the lymphatic system, such as Hodgkin's disease and lymphatic leukemia. Leukemia affects the blood with unrestrained growth of leukocytes.

In regards to patient documentation, which of the following is considered false? A. All documentation entries must be dated B. All handwritten entries must be made in ink with original signatures C. Documentation is required for every patient visit D. Charting errors should be completely removed with white-out solution and rewritten for clarity

D. Charting errors should be completely removed with white-out solution and rewritten for clarity Charting errors should be corrected by drawing a single line through the error and initialing and dating the chart, clearly indicating that a change was made without deletion of the original record. The other options are true statements regarding documentation.

Which upper extremity special test would likely be positive for a patient who complains of shoulder pain, with a current history as a college level base ball pitcher? A. Drop arm test B. Yergason's test C. Adson's test D. Clunk test

D. Clunk test Glenoid labrum tears are commonly occurring injuries for an overhead throwing athelete. The clunk test identifies a glenoid labrum tear. Therefore, the clunk test would be an important special test in the evaluation of this patient. The other options are incorrect, as they do not evaluate structures most commonly identified with overhead throwing athlete injuries. Adson's test identifies a tear of the rotator cuff. Yergason's test tests the integrity of the transverse ligament and may identify bicipital tendonitis.

Which special shoulder test evaluates the integrity of the glenoid labrum? A. Speed's test B. Neer's impingement test C. Drop arm test D. Clunk test

D. Clunk test The clunk test evaluates the integrity of the glenoid labrum. It identifies the glenoid labrum tear. The examiner passively abducts and externally rotates the patient's arm arm overhead and applies an anterior force to the humerus. An audible clunk is indicative of a labrum tear. Speed's test examines for bicipital tendonitis. The drop arm test examines for a rotator cuff tear. Neer's impingement of soft tissue structures in the shoulder complex.

Which physical property of water describes the tendency of water molecules to adhere, relating to the resistance of water to movement? A. Density B. Hydrostatic pressure C. Buoyancy D. Cohesion

D. Cohesion Cohesion is the tendency of water molecules to adhere to each other. The resistance that is felt while moving through water is due to cohesion. Force is required to separate the water molecules. This can create resistance for exercise. Buoyancy is the upward force of water on an immersed or partially immersed body or body part. It is equal to the weight of the water that it displaces. The buoyancy creates an apparent decrease in body weight and joint unloading, which allows for easier movement in the water. Density is the mass per unit volume of a substance. The density of water is proportional to its depth. The deeper the water, the greater the density. This can be incorporated to challenge or regress exercise. Hydrostatic pressure is the circumferential water pressure exerted on an immersed body or body part. A pressure gradient is established between the surface waters and the deeper waters due to the increased density in the deeper water. Increased pressure reduces edema and enhances peripheral circulation.

A 75-year-old patient suffered an injury by falling forward onto an oustretched upper extremity. What is the likely injury? A. Boutonniere deformity B. Dupuytren's contracture C. Swan neck deformity D. Colles' fracture

D. Colles' fracture Colles' fracture is the most common wrist fracture resulting from a fall onto an outstretched upper extremity. The fractures are immobilized between 5-8 weeks. Median nerve compression may occur with increased edema. This fracture is characteristic of a "dinner form" deformity of the hand and wrist. Both Boutonniere deformity and Swan neck deformity are characteristic of rheumatoid arthritis deformities and affect the fingers. Dupuytren's contracture is a banding on the palm and digit flexion contractures due to contracture of the palmar fascia which adheres to the skin.

A 75-year-old patient suffered an injury by falling forward onto an outstretched upper extremity. What is the likely injury? A. Boutionniere deformity B. Depuytren's contracture C. Swan neck deformity D. Colles' fracture

D. Colles' fracture Colles' fracture is the most common wrist fracture resulting from a fall onto an outstretched upper extremity. The fractures are immobilized between 5-8 weeks. Median nerve compression may occur with increased edema. This fracture is characteristic of a "dinner form" deformity of the hand and wrist. Both Boutionniere deformity and Swan neck deformity are characteristic of rheumatoid arthritis deformities and affect the fingers. Duputrey's contracture is a banding on he palm and digit flexion contractures due to contracture of the palmar fascia which adheres to the skin.

In regards to wound assessment, what are the characteristics of wound exudate described as purulent? A. Containing blood B. Containing eschar C. Watery serum D. Containing pus

D. Containing pus Wound exudate type should be determined in the assessment of wounds. Purulent drainage contains pus. Serous drainage is watery serum. Sanguineous drainage contains blood. Eschar is dried necrotic tissue.

Which mechanical agent is MOST often indicated in the acute stage after a total knee replacement in order to accelerate joint range of motion gain and tendon and ligament healing, minimizing contracture risk? A. Mechanical spinal traction B. Intermittent mechanical compression C. Tilt table D. Continuous passive motion

D. Continuous passive motion The continuous passive motion (CPM) device creates uninterrupted passive motion of the joint through a controlled range of motion. The mechanical device provides this passive movement for an extended period of time. The indications are for post-immobilization fracture, tendon, or ligament repair as well as after total hip or knee replacement.

Which change in skin color is typical for a patient with poor oxygenation due to chronic advanced lung disease or congenital heart disease? A. Brown B. Pallor C. Yellow D. Cyanosis

D. Cyanosis Cyanosis is a slightly bluish, grayish discoloration. It is indicative of lack of oxygen and can indicate congestive heart failure, advanced lung disease, congenital heart disease, and venous obstruction. This is commonly seen as cyanosis of the lips, oral mucosa, tongue or nails, hands, and feet. Pallor is lack of color or paleness. It can indicate anemia, internal hemorrhage, or lack of exposure to sunlight. Temporary pallor is seen with arterial insufficiency and syncope, chills, shock, vasomotor instability, or nervousness. A yellow color indicates jaundice or liver disease. It can also be associated with increased carotene intake. Brown color indicates increased pigmentationassociated with venous insufficiency.

The physical therapist assistant incorporates PNF diagonal patterns into treatment to facilitate improved shoulder mobility in order to improve a patient's ability to reach back for a seatbelt in the car. Which PNF pattern is MOST appropriate? A. D1 extension B. D1 flexion C. D2 extension D. D2 flexion

D. D2 flexion D2 flexion for the upper extremity involves flexion-abduction-external rotation of the shoulder. The therapist cues the patient to "open your hand, turn, and lift your arm up and out." This motion closely facilitates the movement to reach for a seatbelt in the car. D1 flexion for the upper extremity involves flexion-adduction-external rotationof the shoulder. D2 extension for the upper extremity involves extension-adduction-internal rotation of the shoulder. D1 extension for the upper extremity involves extension-abduction-internal rotation of the shoulder.

Which is NOT a physiological effect of general (large surface area) cold application? A. Increased arterial blood pressure B. Increased cardiac output C. Increased stroke volume D. Decreased blood flow to internal organs

D. Decreased blood flow to internal organs General cold application over a large surface area has physiological effects on the body including increased blood flow to internal organs, increased cardiac output, increased stroke volume, and increased arterial blood pressure, as well as shivering.

Regarding research design, which term correctly defines the outcome being evaluated or the change that results from the intervention? A. Hypothesis B. Sampling C. Independent variable D. Dependent variable

D. Dependent variable The dependent variable is the change or difference in that results from the intervention (independent variable). It is the outcome being evaluated. The independent variable is the activity or factor believed to bring about a change in the dependent variable. It is the cause or treatment. The hypothesis is a tentative and testable explanation between the relationship between the variables. The results of the experiment determines whether the hypothesis is accepted or rejected. Sampling is the selection of individuals for a study from a population.

In regards to iontophoresis, which medium is indicated for musculoskeletal inflammatory conditions? A. Copper sulfate B. Calcium chloride C. Lidocaine D. Dexamethasone phosphate

D. Dexamethasone phosphate Dexamethasone phosphate and hydrocortisone sodium succinate are utilized for musculoskeletal inflammatory conditions for iontophoresis. Iontophoresis is the application of a continuous direct current to transport medicinal agents through the skin or mucous membranes for therapeutic purposes. Lidocaine is a pain reliever. Calcium chloride is indicated for muscle spasm, and copper sulfate is utilized for fungal infections.

Which statement is FALSE regarding manual lymphatic drainage (MLD)? A. Clearing trunk quandrants first B. Patient educated in self-massage techniques C. Short-stretch compressive bandages used during treatment phase D. Direction of strokes is away from the trunk

D. Direction of strokes is away from the trunk Manual lymphatic drainage is performed by a skilled clinician and performed on a daily basis. Drainage is performed by clearing the trunk quadrants first, then limb drainage staring at the proximal segments first and continuing to the more distal in a successive manner. The direction of strokes is toward the trunk. Lymphatic manual drainage uses light superficial strokes. Short-stretch compressive garments are used during treatment. Patient education on self-massage is included.

Which statement is FALSE regarding manual lymphatic drainage (MLD)? A. Clearing trunk quadrants first B. Patient educated in self-massage techniques C. Short-stretch compressive bandages used during treatment phase D. Direction of strokes is away from the trunk

D. Direction of strokes is away from the trunk Manual lymphatic drainage is performed by a skilled clinician and performed on a daily basis. Drainage is performed by clearing the trunk quadrants first, then limb drainage starting at the proximal segments first and continuing to more distal segments in successive manner. The direction of strokes is toward the trunk. Lymphatic manual drainage uses light superficial strokes. Short-stretch compressive garments are used during treatment. Patient education on self-massage is included.

Because of the property of buoyancy, when moving an immersed arm through the water, which direction of movement is MOST difficult? A. Upward B. Horizontal C. Direction does not matter D. Downward

D. Downward Movement of the arm horizontal to or upward toward the water surface is made easier due to buoyancy of water. Using a flotation device makes with motion easier as well. Movement downward into the water is more difficult because of the buoyancy of the water. Pushing a paddle or flotation device downward will further increase the resistance.

In regards to thermotherapy, which condition would be BEST suited for heat application utilizing paraffin? A. Rotator cuff tear B. 5 day s/p total knee arthroscopy C. Chronic lumbar back pain D. Rheumatoid arthritis of the hand and digits

D. Rheumatoid arthritis of the hand and digits Paraffin application is indicated for painful joints caused by arthritis or other inflammatory conditions in the late subacute or chronic phases. Additionally, it is indicated for joint stiffness and typically utilized over irregularly shaped areas such as the wrist, hand, and foot. It is contraindicated over sites with allergic rash, open wounds, recent sutures, or skin infections.

Which type of stretching is performed actively by moving joints gradually through an increasing range and speed of movement as the musculature warms up and is appropriate to use prior to sport activity? A. Manual static passive stretching B. Prolonged mechanical stretching C. Facilitated stretching D. Dynamic stretching

D. Dynamic stretching Dynamic stretching is an active or passive stretch performed actively by moving joints, either in a body weighted or eliminated position, through the available range of motion. Gradually, the individual increases the range and speed of movement as the musculature warms up. This motion is controlled and performed at a moderate speed and is appropriate to perform before sports activity. Prolonged mechanical stretching is a low-intensity external force, up to 10% of body weight, applied over a prolonged period of time. This is done by positioning the patient with weighted pulleys, traction, or serial casting. Prolonged stretch may mean 20 minutes to 10 hours. Facilitated stretching refers to techniques in which the patient reflexively relaxes muscle to be elongated during stretching techniques. It utilizes active inhibition principles of PNF. Examples include hold-relax, hold-relax-active contraction, and contract-relax-active contraction. Manual static passive stretching stretches the structures beyond the free range of motion to elongate the muscles and soft tissue beyond their resting length. The stretch is typically held 30 seconds to 2 minutes and repeated several times. Intensity and duration depend on patient tolerance.

Which type of stretching is performed actively by moving joints gradually through an increasing range and speed of movement as the musculature warms up and is appropriate to use prior to sports activity? A. Manual static passive stretching B. Prolonged mechanical stretching C. Facilitated stretching D. Dynamic stretching

D. Dynamic stretching Dynamic stretching is an active or passive stretch performed actively by moving joints, either in body weighted or eliminated positions, through the available range of motion. Gradually, the individual increases the range and speed of movement as the musculature warms up. This motion is controlled and performed at a moderate speed and is appropriate to perform before sports activity. Prolonged mechanical stretching is a low-intensity external force, up to 10% body weight,applied over a prolonged period of time. This is done by positioning the patient with weighted pulleys, traction, or serial casting. Prolonged stretching may mean 20 minutes or 10 hours. Facilitated stretching refers to techniques in which the patient reflexively relaxes the muscle to be elongated during stretching techniques. It utilizes active inhibition principles of PNF. Examples include hold-relax, hold-relax-active contraction, and contract-relax-active contraction. Manual static passive stretching stretches the structures beyond free range of motion to elongate the muscles and soft tissue beyond their resting length. The stretch is typically held 30 seconds to 2 minutes and repeated several times. Intensity and duration depend on patient tolerance.

Regarding cardiovascular hemodynamics, which is the term that describes the percentage of blood emptied from the ventricle during systole? A. Stroke volume B. Cardiac output C. Left ventricular end-diastolic pressure D. Ejection fraction

D. Ejection Fraction Ejection fraction is the percentage of blood emptied from the ventricle during systole. Stroke volume is the amount of blood ejected with each myocardial contraction, which is approximately 70 mL. Left ventricular end-diastolic pressure is the pressure in the left ventricle during diastole. Cardiac output is the volume of blood discharged from the left or right ventricle per minute. which is approximately 4-6 L per minute in a typical adult.

A physical therapist assistant is teaching the importance of proper seating to reduce the risk of pressure ulcers to a patient who has recently suffered a spinal cord injury. The therapist should educate the patient that the primary cause of this type of skin breakdown is due to: A. Nutrition B. Friction C. Incontinence D. Excess pressure

D. Excess Pressure Pressure ulcers are formed from excessive pressure in one area. For this patient, the pressure ulcer would form from pressure of prolonged sitting position. Skin breakdown from frictin typically occurs during transfers. Nutrition and incontinence can contribute to skin breakdown, but they are not the primary causeof pressure ulcers.

A patient is being seen in an acute setting after sustaining a motor vehicle accident. The initial evaluation reveals that the patient has a Glasgow Coma scale rating of 2. What can the physical therapist assistant expect to see from this patient? A. Eye opening to verbal command, inappropriate word usage, and decorticate posturing B. Spontaneous eye opening, confused conversation, and motor withdrawal to pain C. Appropriate eye opening, verbal orientation, and motor localization of pain D. Eye opening to pain stimulus, incomprehensible sounds, and decerebrate posturing

D. Eye opening to pain stimulus, incomprehensible sounds, and decerebrate posturing The Glasgow Coma Scale is a rating scale from 1-6. Consciousness is related to the patient's degree of eye opening, motor response, and verbal response. Eye opening to pain stimulus, incomprehensible sounds, and decerebrate posturing are indicative of Glasgow rating 2. A Glasgow Coma Scale rating of 4 would include spontaneous eye opening, confused conversation, and motor withdrawal to pain. Eye opening to verbal command, inappropriate word usage, and decorticate posturing indicate Glasgow rating 3. Appropriate eye opening, verbal orientation, and motor localization of pain indicate Glasgow rating 5.

The PTA is teaching a patient with a spinal cord injury at L1 to manage curbs in his wheelchair. The BEST way to instruct the patient is to have him: A. Throw head and trunk posteriorly to rise up on the back wheels B. Grab the hand rims anteriorly and then pull them forward abruptly and forcefully C. Throw head and trunk forward to lift the back wheels D. Grab the hand rims posteriorly and then pull them forward abruptly and forcefully

D. Grab the hand rims posteriorly and then pull them forward abruptly and forcefully In order to navigate over a curb, the patient must "pop a wheelie." This is done by having the patient grab the hand rims posteriorly and then pull them forward abruptly and forcefully. This will lift the casters and allow clearance over the curb. The other options are not appropriate. Moving the body forcefully will not lift the casters, as the rims need to be forcefully and quickly moved.

The physical therapist assistant is challenging a patient's balance by having her seated over an exercise ball with her arms elevated, while the PTA provides perturbations at the patient's trunk. The patient is having difficulty maintaining her balance and frequently requires assistance from the therapist assistant through guarding. The PTA decides to regress the exercise by: A. Throwing and catching a ball while continuing perturbations B. Having the patient close her eyes C. Having the patient elevate one lower extremity from the floor D. Having the patient place her hands on the side of the exercise ball, instead of elevated

D. Having the patient place her hands on the side of the exercise ball, instead of elevated By placing her hands on the side of the exercise ball, the patient has decreased the number of segments involved in the balance activity. She has increased the stability of the support surface as well. The other three options are ways to progress he exercise by decreasing the stability of the surface, decreasing the base of support, and increasing the range of movement involved.

Which federal law assures privacy and security of all personal health care information? A. Health Information Protection and Accountability Act B. Health Insurance Privacy Protection Act C. Health Information Protection Act D. Health Insurance Portability and Accountability Act

D. Health Insurance Portability and Accountability Act. The Health Insurance Portability and Accountability Act (HIPAA) is a federal law established in 1996 and applies to health information created or maintained by health care providers. It assures privacy of all health care information, especially that in electronic form. Information that is protected includes information entered in the medical chart, conversations among caregivers and between patient caregivers, and billing information. Information can be shared to provide appropriate care and coordinate care, reimbursement, protect public(such as reporting cases of flu or whooping cough), and complete required reports to authorities (such as gunshot wounds to police).

An 85-year-old patient is being seen in a sub-acute rehabilitation setting after an open reduction internal fixation of the hip, which was necessary due to a fall that resulted in a femoral neck fracture. What complication would be LEAST likely for this patient? A. Pulmonary embolism B. Deep vein thrombosis C. Fear of falling D. Hip Dislocation

D. Hip dislocation Hip dislocation is associated with total hip arthroplasty, not an open reduction internal fixation. However, complications of surgery are important to watch out for. Due to decreased mobility after the fall and surgery, the patient is at risk for development a deep vein thrombosis and/or pulmonary embolism. Because the patient sustained a hip fracture due to a fall, she will likely have a fear of falling again.

A physical therapist assistant is teaching a patient to descend a curb using a small base quad cane. The BEST sequencing for teaching this is: A. Hold cane on weaker side, descend with stronger extremity first B. Hold cane on strong side, descend with stronger extremity first C. Hold cane on weaker side, descend with weaker extremity first D. Hold cane on stronger side, descend with weaker extremity first

D. Hold cane on stronger side, descend with weaker extremity first When using a device in one hand, the device is used opposite the weaker lower extremity. When descending a stair or curb, the patient should lead with the weaker lower extremity and allow the stronger lower extremity to control the descent with eccentric control.

The physical therapist assistant chooses to incorporate PNF techniques to improve limited range of motion resulting from muscle spasm. Which PNF technique is BEST indicated? A. Rhythmic initiation B. Repeated stretch C. Stabilizing reversals D. Hold-Relax

D. Hold- Relax Hold--relax is a PNF technique indicated for limited range of motion resulting from muscle tightness, spasm or pain. It is performed with an isometric contraction of antagonist followed by relaxation and passive range of motion into new range. Rhythmic rotation is indicated in an inability to initiate movement, poorly coordinated movements and with communication disorders. Stabilizing reversals are indicated for decreased stability, poor antigravity control, and weakness. Repeated stretch is indicated for weakness, fatigue, and decrease ability to perform functional movements.

The physical therapist assistant is teaching a patient to descend stairs using a cane and one handrail. The patient has poor eccentric control of the left quadriceps. Which is the proper position for the physical therapist assistant for guarding? A. In front and slightly to the patient's right side B. Behind and slightly to the patient's right side C. Behind and slightly to the patient's left side D. In front and slightly to the patient's left side

D. In front and slightly to the patient's left side During descending, the therapist should be in the front (lower step) and slightly to the involved side. When ascending, the correct guarding position is always a behind (lower step) and slightly on the involved side.

The physical therapist assistant is working with a patient who has paraplegia with some sensation and mixed motor function below the level of the lesion. What is the likely injury? A. Complete spinal cord injury at C5 B. Complete spinal cord injury at T8 C. Incomplete spinal cord injury at C4 D. Incomplete spinal cord injury at T11

D. Incomplete spinal cord injury at T11 The lesion level indicates the most distal uninvolved nerve root segment with normal function. Qudriplegia involves all four extremities and trunk and occurs with injuries between C1 and C8. Paraplegia involves both lower extremities and varying levels of trunk and occurs with injury between levels T1 and L1. A complete injury results in no sensory or motor function below the level of the lesion. An incomplete lesion has preservation of sensory or motor function below the level of injury. There are spotty sensations and some muscle function. Therefore, in this example, there is movement of only lower extremities (cannot be a cervical level injury). Additionally, there is mixed motor and sensory function, which means it is an incomplete injury.

Two physical therapist assistants are asked to perform the same functional outcome measurement tool, the Berg Balance Assessment, on the same patient. The results show a difference in scoring between the two therapists. However when the same therapist repeats the test, his and her test scores remain the same between the two trails. This is an example of a problem with: A. Patient performance B. Clinic setup for the testing environment C. Intra-rater reliability D. Inter-rater reliability

D. Inter-rater reliability Inter-rater reliability is the degree to which two or more testers are able to obtain the same score or rating. In this case, the two therapist assistants found different scores with the same patient. However, they obtained the same score in their second trial, showing good interrater reliability. Intra-rater reliability is the degree to which the single tester will obtain the same score in subsequent testing.

Which form of resistance exercise uses dynamic muscle contraction that is speed controlled, with variable and accommodating resistance? A. Eccentric B. Isometric C. Isotonic D. Isokinetic

D. Isokinetic Isokinetic resistance exercise is dynamic and has a speed control for muscle shortening and lengthening. Resistance is accommodating and variable. The individual uses isokinetic equipment, and it provides maximum resistance at all points of the range of motion. Eccentric resistance exercise is performed with dynamic lengthening contraction. This is useful to prepare muscles for functional activities and for muscles unable to perform concentric contraction. Maximum eccentric contraction produces more force than maximal concentric contraction. Isometric resistance exercise is performed as a static muscle contraction, with no muscle length change. The patient holds a muscle contraction for at least 6 seconds. This is beneficial to strengthen a muscle at a specific point in the ROM. Isotonic exercises use dynamic muscle contraction. There is constant (free weights) or variable (machine) resistance and uses concentric and eccentric contractions. The patient moves a resistance through the available range of motion and speed can be variable.

Which is a FALSE statement regarding cerebral palsy? A. Insult to the brain B. Seizures and mental retardation C. Hypotonia, hypertonia, dystonia, ataxia D. It is a permanent and progressive condition

D. It is a permanent and progressive condition While cerebral palsy (CP) is a permanent condition, it is non-progressive. The insult to the brain can occur during development, at birth, or after birth and it results in permanent, non-progressive damage. CP often presents with mental retardation, visual impairment, speech deficits, seizures, hearing impairment, and behavior disorders. The individual with CP may have a variety of muscle tone classifications, including hypotonia, hypertonia, dystonia, ataxia and/or mixed tone

The physical therapist assistant is working with a patient dependent for mobility and primarily bed-bound with high muscle tone and areas of muscle spasm around the shoulder complex and hips. Which sensory facilitation technique would be BEST indicated to improve joint awareness while relieving muscle spasm around the joint? A. Inhibitory pressure B. Quick stretch, tapping of muscle belly C. Resistance D. Joint traction

D. Joint traction Joint traction is a facilitation technique that enhances joint awareness and actions of flexors. Additionally, it relieves muscle spasm. Inhibitory pressure also decreases muscle tone (and inhibits muscle), but it is an inhibitory technique and not a facilitation technique. The other two options are facilitation techniques, but they do not increase joint awareness and decrease muscle spasm. A quick stretch and tapping of muscle belly facilitates agonist muscle and inhibits the antagonist muscles. Resistance recruits motor units and facilitates and strengthens the agonist contraction.

The physical therapist is testing sensation in her patient. She moves the patient's limb in various directions (up/down, left/right) and asks the patient to identify the direction of motion. Which sensory test is being performed? A. Graphesthesia B. Stereognosis C. Barognosis D. Kinesthesia

D. Kinesthesia Kinesthesia is tested by moving the patient's limb in various directions. The patient identifies the direction of the motion the limb is being moved. It is a test of proprioceptive sensation. Graphesthesia is a test of cortical sensation. It is done when the therapist draws a letter in the palm of a patient's hand. The patient should be able to identify the letter. Barognosis is when similar objects with varying weights are placed in the patient's hand. The patient should be able to identify differences in weight. Stereognosis is when an object is placed in the patient's hand, and the patient should be able to identify the object by touch only.

When assessing end feels for a typical joint, which would have a normal physiological end feel that would be described as firm or capsular? A. Shoulder abduction in a patient with a torn rotator cuff B. Knee flexion C. Elbow extension D. Knee extension

D. Knee extension Knee extension would typically have a firm/capsular end feel, which indicates capsular and ligamentous stretching. This feels like stretching a leather belt. A soft end feel due to soft tissue approximation occurs when two soft tissues (typically muscle and/or fat) meet and restrict any further movement. In knee flexion, the hamstring approximates with the gastrocnemius and prevents any further motion at the knee. Elbow extension would typically have a bony/hard end feel, which indicates when bone and/or cartilage meet. If the patient has a torn rotator cuff and the clinician is assessing the end feel for shoulder abduction, then it would likely be an empty end feel. This occurs when the clinician is unable to bring the joint through the full range due to guarding by the patient, which is typically due to pain.

A physical therapist assistant is preparing to work with a patient who has suffered a traumatic brain injury. The evaluation states the patient is a Level III on the Rancho Los Amingos level of cognitive function (LOCF). What can the therapist expect of this patient? A. Automatic-appropriate: The patient appears appropriate and oriented within the hospital and home settings but has shallow recall of activities. Judgement is still impaired. B. Confused-agitated: The patient is in a heightened state of activity. His behavior is bizarre and non-purposeful relative to environment. He is unable to cooperate directly with treatment efforts. C. No response: Patient appears to be in a deep sleep and completely unresponsive to stimulation. D. Localized response: The patient reacts specifically but inconsistently to stimuli. The patient may follow simple commands in an inconsistent and delayed manner such as squeezing the hand or closing eyes.

D. Localized response: The patient reacts specifically but inconsistently to stimuli. The patient may follow simple commands in an inconsistent and delayed manner such as squeezing the hand or closing eyes. The Ranchos Los Amigos Levels of Cognitive Functioning (LOCF) outlines predictable sequence of cognitive and behavioral recovery. It has eight levels of behavior. Typically, individuals will progress through levels in sequence and can plateau at any point. Level I: No response Level II: Generalized response Level III: Localized response Level IV: Confused-agitated Level V: Confused-inappropriate Level VI: Confused-appropriate Level VII: Automatic-appropriate Level VIII: Purposeful and appropriate

Which term describes the joint position where capsule and other soft tissues are the MOST relaxed position? A. Open chain B. End feel C. Closed-packed position D. Loose-packed position

D. Loose-packed position The loose-packed (or resting) position is the joint position in which the capsule and other soft tissues are in the most relaxed position. There is minimal joint surface contact. The therapist may choose to perform joint mobilization techniques in this position. The closed-packed position is that in which the capsule and other soft tissues are maximally tensed. There is maximal contact between joint surfaces. Joint mobilization cannot be performed properly in this position. End feel refers to the way that joint feels at the end range of motion. A normal end feel that is soft occurs with soft tissue approximation. A normal end feel that is firm occurs with capsular and ligamentous stretching. A normal end feel that is hard occurs when bone and/or cartilage meet. Open chain refers to a form of exercise in which the distal segment of a limb is moved freely in space.

The PTA is instructing a patient with a new transfemoral amputation in positioning strategies to reduce the likelihood of contractures. What should the therapist stress? A. Lying supine with residual limb elevated over a pillow B. Sitting in the wheelchair with an appropriate seat cushion C. Lying on the side of the residual limb D. Lying prone with the residual limb in neutral rotation

D. Lying prone with the residual limb in neutral rotation The typical contracture for a transfemoral amputation is in hip flexion, which is typically the result of too much sitting in the chair. The residual limb also externally rotates. Positioning should emphasize hip extension and neutral rotation. Lying prone will stretch the hip flexors. Sitting in the wheelchair promotes hip flexion contractures. Lying on the side does not stretch the hip flexors. Lying supine with the residual limb propped on a pillow will also promote hip flexion.

A patient sustained an "unhappy triad" knee injury playing football. What anatomical structures are involved in an injury affecting the "unhappy triad"? A. MCL, PCL, and lateral meniscus B. LCL, PCL, and the medial meniscus C. MCL, PCL, and the medial meniscus D. MCL, ACL, and the medial meniscus

D. MCL, ACL, and the medial meniscus The medial collateral ligament, anterior cruciate ligament, and the medial meniscus are involved in an unhappy triad injury. This occurs with a combination of valgum, flexion, and external rotation forces to the knee when the foot is planted. It is commonly occurring in athletic events such as football.

When performing soft tissue mobilization on a patient's back, the therapist notices a dark, raised area with an area with an irregular border and asymmetrical presentation, which she is concerned may need further evaluation by a dermatologist. Which form of malignant tumor is characterized by asymmetry, irregular borders, variations in color, diameters larger than 6 mm, and elevation? A. Basal cell carcinoma B. Squamous cell carcinoma C. Kaposi's sarcoma D. Malignant melanoma

D. Malignant melanoma Malignant melanoma is a tumor that arises from melanocytes. Superficial spreading melanoma is the most common type. It is characterized by uneven edges, irregular borders, variations in color, larger than 6 mm, and typically elevated presentation. Kaposi's sarcoma is a lesion of endothelial cell origin with red or dark purple macules that progress to nodules or ulcers. It is associated with itching and pain. It is common in lower extremities. Squamous cell carcinoma grow more quickly and is common on sun-exposed areas including the face, neck, and back of hands. It can metastasize. A basal cell carcinoma is a low-growing epithelia basal cell tumor. It is characterized by a raised ivory color patch with a rolled border and indented center. It rarely metastasized and is common to the face in fair-skinned people. It is associated with prolonged sun exposure.

A PTA provides repeated sit-to-stand transfer training in a patient with Parkinson's disease with impaired balance, providing decreasing verbal cues with progressive trials. Which training strategy is being used? A. Compensatory strategy B. Remedial strategy C. Patient Strategy D. Motor learning strategy

D. Motor learning strategy Motor learning strategies assist the central nervous system in adaptation for movement control. It utilizes repetition for motor activities, as well as sensory cues and feedback. A compensatory strategy is used to promote safety and early resumption of functional skills. Remedial strategies focus on the use of the involved body segments, such as paretic limb in a person with a stroke. Patient strategy is not a training strategy.

Regarding electrical stimulation, which term describes the area of greatest excitability on the skin surface, in which a small amount current generates a muscle response? A. Muscle pump B. Sensory point C. Muscle origin D. Motor point

D. Motor point The motor point is an area of greatest excitability for generating a muscle response. It is an area on the skin surface in which electric stimulation can be delivered. For innervated muscles, the motor point is located where the motor nerve enters the muscle, typically over the muscle belly. For denervated muscle, the area of greatest excitability is usually located over the muscle distally toward the insertion.

Which disorder is a painful condition of abnormal calcification within a muscle belly? A. Gout B. Torticollis C. Bursitis D. Myositis ossificans

D. Myositis ossificans Myositis ossificans is a painful condition with abnormal calcification in a muscle belly. It is usually caused by direct trauma to the muscle, resulting in hematoma and calcification. It can also be induced by early aggressive physical therapy following direct muscle trauma. It most frequently affects the quadriceps, brachialis, and biceps brachii muscles. Gout is a disorder of purine metabolism characterized by elevated serum uric acid. The uric acid changes into crystals and deposits into peripheral joints and tissues. It is most frequently observed in the knee and great toe. Torticollis is a spasm or tightness of the sternocleidomastoid muscle. It is seen by side-bending toward and rotation away from the affected side. Physical therapy aims to correct for the SCM spasm or tightness and restore normal range and head motion. Bursitis is an inflammation of the bursa secondary to overuse , trauma, gout, or infection. It is characterized by pain at rest and limited range of motion due to pain, not capsular restriction. Physical therapy aims to reduce inflammation, reduce pain, and improve flexibility.

The physical therapist assistant is reviewing the physical therapist's evaluation for a patient with torticollis. The patient has torticollis with right sternocleidomastoid involvement. What will the patient present like? A. Neck side-bending toward the right and rotation to the right B. Neck side-bending toward the left and rotation to the left C. Neck side-bending toward the left and rotation to the right D. Neck side-bending toward the right and rotation to the left

D. Neck side-bending toward the right and rotation to the left Torticollis is spasm and/or tightness of the sternocleidomastoid (SCM) muscle. Dysfunction is seen as side-bending toward and rotation away from the affected muscle. In this patient case, with right side SCM invlvement, the patient would present with neck side-bending to the right and rotation toward the left.

In which patient population is foot-flat stepping gait pattern NOT considered a gait abnormality? A. Geriatric: 90 years old B. Adult: 45 years old C. Adolescent: 14 years old D. Pediatric: 13 months old

D. Pediatric: 13 months old Foot flat is when the entire foot contacts the ground and is a result of weakness in the dorsiflexsors and/or limited range of motion in the ankle. It is also a function of an immature gait pattern, which is considered normal during gross motor development in an infant. Therefore, a pediatric patient age 13 months would display a foot flat steeping, and this would be considered normal. In all other patient options, the foot-flat stepping would be considered a gait abnormality.

The physical therapist assistant notes that her patient has an inability to breathe when she positions her supine or in a reclined position. This condition is known as: A. Orthostatic hypotension B. Dyspnea on exertion C. Paroxysmal nocturnal dyspnea D. Orthopnea

D. Orthopnea Orthopnea is an inability to breathe when in a reclined position. Typically, this is due to the weight of gravity on the diaphragm and chest wall, making it difficult to fully expand the lungs. Paroxysmal nocturnal dyspnea is a sudden inability to breathe during sleep. Dyspnea on exertion is shortness of breath from exertion. Orthostatic hypotension is a sudden drop in blood pressure with a change in position.

Which pediatric disorder is the result of an abnormality in the collagen gene causing problems with the amount and quality of the collagen in the body, resulting in fragile bones that break easily? A. Arthogryposis Multiplex Congenita B. Legg-Calve-Perthes Disease C. Slipped Capital Femoral Epiphysis D. Osteogenesis Imperfecta

D. Osteogenesis Imperfecta Osteogenesis Imperfecta is characterized by fragile bones that break easily and often for no reason. It is due to an abnormality in the collagen gene causing problems with the amount and quality of collagen in the body. There are four types with varying degrees of severity. Arthogryposis Multiplex Congenita is a non-progressive neuromuscular disorder that causes multiple joint contratures at birth. There is variability in clinical picture, but typically include severe joint contractures and absence of muscle development. Children will utilize splinting, surgery, and assistive devices to compensate and correct for deformities. Slipped Captial Femoral Epiphysis is a hip deformity related to the slippage of the femoral epiphysis. It occurs in children who are tall, with delayed skeletal maturity, obese, and near the onset of puberty. It is more common in boys than in girls. In mild or moderate cases, surgery is required to pin the hip and prevent further slippage. Severe cases require various osteotomy. Legg-Calve-Perthes Disease is a condition of self-limiting degeneration of the femoral head due to a disturbance in the blood supply. It most commonly occurs in boys 4-7 years of age. It is less frequent in girls with onset at older ages.

When listening to heart sounds, where should the stethoscope be placed to locate the aortic valve? A. Over the second left intercostal space at the sternal border B. Over the fourth left intercostal space at the sternal border C. Over the fifth left intercostal space at the midclavicular line D. Over the second right intercostal space at the sternal border

D. Over the second right intercostal space at the sternal border The auscultation landmark for the aortic valve is found over the second right intercostal space at the sternal border. The auscultation landmark for the pulmonic valve is at the second left intercostal space at the the sternal border. The auscultation landmark for the tricuspid valve is found over the fourth left intercostal space at the sternal border. The auscultation landmark for the mitral valve is found over the fifth left intercostal space at the midclavicular line.

Which is a degenerative disease of the central nervous system characterized as a basal ganglia disorder with dopamine deficiency and degeneration of substantia nigra? A. Amyotrophic lateral sclerosis B. Multiple sclerosis C. Meningitis D. Parkinson's disease

D. Parkinson's disease Parkinson's disease is a chronic, progressive, degenerative disease of the CNS. It is a basal ganglia disorder characterized by dopamine deficiency abd degenraion of the substantia nigra. Patients present with rigidity, bradykinesia, resting tremor, and impaired postural reflexes. Multiple sclerosis is a degenerative disease of the CNS characterizedby chronic, progressive, demyelinating lesions that impair neural transmission and cause nerves to fatigue rapidly. Amyotrophic lateral sclerosis is a progressive upper and lower motor neuron disease in which the muscle fibers atrophy from peripheral nerve involvement. Degeneration and scarring of the of the motor neurons occur in the lateral aspect of the spinal cord, brainstem, and cerebral cortex. Meningitis is an infectious disorder characterized by inflammation of the meninges or the spinal cord or brain. It is due to a bacterial or viral infection.

A patient has known impairment of sensation over the L4 dermatome. Which area will have impaired sensation to touch? A. Thoracic region B. Groin region C. Gastroc-soleus area D. Patellar region

D. Patellar region The L4 dermatome is a large area that includes the low back around to the anterior thigh, over the front of the knee, and down the front of the lower leg as well as the bottom of the foot. The gastroc-soleus region is the S1 and S2 dermatomes. The groin area includes the L1 and L2 as well as S2 and S3 dermatomes. The thoracic region includes the thoracic dermatomes.

Which is NOT a pressure-sensitive area of the transtibial residual limb? A. Anterior tibial crest B. Fibular head and neck C. Fibular nerve D. Patellar tendon

D. Patellar tendon The patellar tendon is actually a pressure-tolerant area of the typical transtibial residual limb. Many sockets utilized are patellar tendon bearing (PTB) sockets. PTB sockets allow for moderate loading over the area of the patellar tendon. Pressure-sensitive areas of the transtibial residual limb include the anterior tibia, anterior tibial crest, fibular head and neck, and fibular nerve.

The BEST initial intervention to promote functional mobility after a four-week shoulder immobilization is: A. TENS for pain reduction B. Resisted strengthening with a resistive band C. Isometric shoulder strengthening in all planes D. Pendulum exercises

D. Pendulum exercises After a prolonged immobilization, the shoulder is prone to stiffness and exercises should focus on shoulder mobility. Pendulum exercises assist in preventing shoulder stiffness and are low-intensity means for shoulder joint mobilization. TENS is intended for pain relief, not functional mobility (though it may be an appropriate intervention). Isometrics may be used early in treatment, but it is for the goal of muscle strength, not mobility. Resistive exercises are not indicated in this early period of therapy.

The physical therapist assistant is working with an elderly patient who has poor abdominal and trunk extensor strength and has gastroesophageal reflux disease. The patient has frequent complaints of reflux. What is the BEST choice for intervention modification for this patient? A. Modify the intervention to include jogging while monitoring vitals B. Instruct the patient to eat a full meal prior to therapy C. Perform exercises lying supine D. Perform intervention prior to patient eating

D. Perform intervention prior to patient eating Individuals with gastroesophageal reflux disease (GERD) have discomfort due to stomach acids irritating he esophageal lining. Lying on the back may increase reflux due to positioning. Jogging can increase stomach churning and make symptoms worse. Eating right before exercise is not appropriate, as the patient will have increased stomach acid activity. The patient should avoid eating prior to therapy and my eat after his session.

Which form of massage technique describes a grasping and lifting of tissues, meant to loosen adhesions and increase venous return? A. Tapotement B. Effleurage C. Vibration D. Petrissage

D. Petrissage Petrissage is a kneading technique that grasps and lifts the tissues. It aids in loosening adhesions and increases venous return. The direction of the strokes should move from distal to proximal if the intention is to increase venous return. Effleurage is a technique by which the clinician provides gliding movement of the hands over the surface of the skin. Tapotement is rapid striking of the surface of the palm, cupped hand or edge of hand over the patient's skin. Vibration is shaking of the tissue with short, rapid motions.

A patient underwent a right-side transfemoral amputation four months ago and is now participating in outpatient physical therapy services. During the session, the patient complains of sharp pain in his right heel. What is this patient describing? A. Local pain B. Psychological dysfunction C. Phantom sensation D. Phantom pain

D. Phantom pain It is common for an individual to experience phantom limb sensation. Phantom pain may be local or diffuse and can be continuous or random. Phantom pain is usually described as knifelike, sharp, or electrically shocking in nature. Phantom sensation is the sensation that the patient feels the entire limb is present. Local pain refers to a specific pain at an injured area that can be pointed out. Phantom pain is a real experience and should not be considered psychological dysfunction. For most patients, phantom pain recedes in time.

Which of the following health care workers is a non-licensed provider? A. Physical therapist assistant B. Social worker C. Occupational therapist assistant D. Physical therapy aide

D. Physical therapy aide The physical therapy aide is a non-licensed worker. This worker is trained under the direction of a PT or PTA. They function within a physical therapy setting with continuous on-site supervision of a PT or PTA. They perform designated routine tasks related to the operations of the physical therapy setting. Job responsibilities can include patient transportation, equipment maintenance, secretarial and housekeeping duties. The physical therapist assistant, social worker, and ocupational therapist assistant are all licensedhealth care workers.

Which of the following is NOT an absolute contraindication to joint mobilization? A. Joint ankylosis B. Down syndrome C. Bone malignancy D. Pregnancy

D. Pregnancy Pregnancy is a relative contraindication, but not an absolute contraindication, to joint mobilization. Pregnancy is associated with increased joint laxity due to hormone changes. Therefore, joints may have increased mobility and should be accounted for during joint mobilization. Down syndrome is a condition that has impaired ligament integrity; therefore, it is an absolute contraindication to joint mobilization. Additionally, joint ankylosis and bone malignancy are also absolute contraindications to joint mobilizations.

The physical therapist assistant assesses her patient's blood pressure prior to intervention and finds it to be 130/85 mm Hg. What is this considered? A. Normal B. Hypertension Stage 1 C. Hypertension Stage 2 D. Prehypertension

D. Prehypertension Prehypertension is considered 120-139 mm Hg systolic; 80-89 mm Hg diastolic. Blood pressure should be assessed prior to, during, and after intervention. A normal adult blood pressure is < 120 mm Hg systolic; < 80 mm Hg diastolic. Hypertension Stage 1 is 140-159 mm Hg systolic; 90-99 mm Hg diastolic. Hypertension Stage 2 is >160 mm Hg systolic; >100 mm Hg diastolic. It is important to note that the majority of patients with hypertension are without symptoms, which is why monitoring by the therapist is so important. It is not within the scope of the PTA's practice to diagnose a patient as prehypertensive, but the physician should be informed of the blood pressure.

What is the appropriate progression for PNF technique interventions? A. Progress control from large range to small range and isotonic to isometric contractions. B. Progress control from large range to small range and isometric to isotonic contractions C. Progress control from small range to large range and isotonic to isometric contractions D. Progress control from small range to large range and isometric to isotonic contractions

D. Progress control from small range to large range and isometric to isotonic contractions In order to facilitate volitional control of movement,the therapist should progress control from small range to large range and isometric to isotonic contractions. The therapist will give proprioceptive inputs of resistance, weight bearing, stretching, and tapping. Additionally, the use of eye contact and appropriate verbal commands is necessary. Fatigue, pain, and heavy resistance are avoided, as they can decrease control.

Which intervention for urinary incontinence uses weighted vaginal cones for home exercises to target the pelvic floor muscles? A. Functional electrical stimulation B. Biofeedback C. Behavioral training D. Progressive strengthening

D. Progressive strengthening Progressive strengthening uses weighted vaginal cones for home exercises to increase the strength of the pubococcygeues muscles. Biofeedback uses pressure recordings to reinforce active contractions and relax the bladder. Functional electrical stimulation is used for muscle reeducation if a patient is unable to initiate to initiate active contractions in the pelvic floor. Behavioral training includes record keeping of voiding and education for lifestyle changes to avoid Valsalva's maneuver and heavy resistance exercises.

Which phase of burn healing is characterized by scar tissue formation by fibroblasts and wound contraction? A. Inflammatory phase B. Maturation phase C. Emergency phase D. Proliferative phase

D. Proliferative phase The three phases of burn healing include the inflammatory phase, proliferative phase, and maturation phase. In the proliferative phase, the fibroblasts form scar tissue, and there is wound contraction. Re-epithelialization may occur at the wound surface if there are still viable cells. The inflammatory phase is characterized by redness, edema, warmth, pain, and decreased range of motion, typically lasting 3-5 days. The maturation phase is characterized by scar tissue remodeling and lasts up to two years.

Part of the role of the physical therapy team is to prevent contractures in a patient after limb amputation. Which is NOT an appropriate position for a patient post-transtibial amputation? A. Prolonged knee extension B. Prolonged trunk extension C. Prolonged hip extension D. Prolonged knee flexion

D. Prolonged knee flexion Positioning for preventing contractures is important after amputation. Positions to avoid include prolonged hip flexion and external rotation as well as prolonged knee flexion. The therapist should utilize a posterior board to keep the knee straight when seated in a wheelchair. Additionally, the patient should have regularly scheduled prone-lying time in order to prevent hip flexion contracture.

Which is incorrect regarding the emergency care for a patient who has sustained a burn? A. Immersion in cold water B. Use of cold compresses C. Cover burns with sterile bandage D. Prompt application of ointment

D. Prompt application of ointment Emergency care for burn management is immersion in cold water. Cold compresses may also be used. The burn should be covered with sterile bandages or a clean cloth. No ointments or creams should be used.

A patient presents with thick, scaly plaques on his skin that have a silvery appearance. The patient reports that the plaques increase when he is stressed. What is the likely skin disorder? A. Eczema B. Ringworm C. Scleroderma D. Psoriasis

D. Psoriasis Psoriasis is an immune skin disorder caused by rapid skin cell production. It presents with scaly and silvery skin plaques. Ringworm is a fungal infection that forms ring-shaped patches with vesicles or scales. Scleroderma is a chronic disease of connective tissue that causes fibrosis of skin and other connective tissues. The skin appears taut and firm. Eczema is an inflammation of of the skin causing itching, redness, and lesions. It can be caused by allergies, photosensitivity, or with unknown cause.

Which of the following is NOT a pressure-tolerant area of the typical transfemoral residual limb? A. Lateral sides of residual limb B. Ischial tuberosity C. Gluteals D. Pubic symphysis

D. Pubic symphysis The pubic symphysis is a pressure-sensitive area of a typical transfemoral residual limb. Other pressure-sensitive areas include the distolateral end of the femur and the perineal area. Pressure-tolerant areas of a typical transfemoral residual limb include the ischial tuberosity, gluteals, lateral sides of the residual limb, and the distal end.

Which pulmonary condition is characterized by excessive seepage of fluid of from the pulmonary vascular system into the interstitial space? A. Atelectastis B. Pleural effusion C. Pulmonary emboli D. Pulmonary edema

D. Pulmonary Edema Pulmonary edema is excessive seepage of fluid from the pulmonary vascular system into the interstital space. It can eventually cause alveolar edema. There are cardiogenic and non-cardiogenic forms of pulmonary edema. Cardiogenic pulmonary edema results from increased pressure in pulmonary capillaries associated with left ventricular failure, aortic valve disease, or mitral valve disease. Non-cardiogenic results from increased permeability of the alveolar capillary membranes due to inhalation of toxic fumes, hypervolemia, narcotic overdose, or adult respiratory distress syndrome. Atelectasis is a collapsed or airless alveolar unit. It is caused by hypoventilation secondary to pain during ventilator cycle, internal bronchial obstruction, external bronchial compression, low tidal volumes, or neurologic insult. Plueral effusion is excessive fluid between the visceral and parietal pleura, caused by increased pleural permeability to proteins from inflammatory disease, neoplastic disease, or cogestive heart failure. A pulmonary emboli is a thrombus from the peripheral venous circulation that becomes embolic and lodges in the pulmonary circulation.

The PTA is teaching transfer training to a patient with a C8 complete spinal cord injury. Which type of transfer indicates the MOST independent type that this patient would be able to complete without assist? A. Dependent lift transfer B. Assisted stand pivot transfer C. Slide board transfer D. Push-up transfer

D. Push-up transfer The push-up transfer (pop-over transfer) is used for patients with good sitting balance who can lift their buttocks clear of the support surface. The patient with a C7 spinal cord injury or below can be independent in this technique. The patient uses the head hips principle to complete the transfer. The assisted sliding board transfer is used for patients with good sitting balance who can lift most but not all of their weight off the buttocks such as the individual with a complete C5 spinal chord injury. Patients with complete C6 level spinal cord injury can be independent with slide board transfers on even levels. This is not as independent of a transfer as the push-up transfer. Assisted stand pivot transfer is used for patients unable to stand independently and can bear some weight on the lower extremities such as a patient with CVA, imcomplete spinal cord injury, weakness, and orthopedic injury. The patient with a complete C8 spinal cord injury would not be able to actively bear weight through the lower extremities. Dependent lift transfer is indicated for patients with minimal or no active participation in the transfer, which is not the case in this patient example.

A patient has nerve entrapment related to a long history of playing baseball as a pitcher. He has pain at the lateral elbow and paresthesia into the posterior aspect of the forearm. Which nerve is involved? A. Tibial nerve B. Median nerve C. Ulnar nerve D. Radial nerve

D. Radial nerve Radial nerve entrapment occurs in the distal branches within the radial tunnel due to overhead activities and throwing. Clinical signs include lateral elbow pain, pain over supinator muscle, and paresthesia into the posterior aspect of the forearm. Median nerve entrapment occurs within the pronator teres muscle and under the superficial head of the flexor digitorum superficialis. It occurs with repetitive gripping activities and sporting activities. Clinical signs include aching pain with weakness in forearm muscles and paresthesia into digits 1-4. Ulnar nerve entrapment is caused by direct trauma to the cubital tunnel, traction due to laxity in the medial elbow, compression due to thickening of retinaculum, hypertrophy of flexor carpi ulnaris, or DJD of the cubital tunnel. Symptoms include median elbow pain and paresthesia into digits 4 and 5. The tibial nerve is not a viable option, as the tibial nerve innervates the lower leg, and this question describes symptoms in the upper extremity.

Which term describes a peripheral nervous system disorder that includes involvement of nerve roots due to skeletal changes and soft tissue injuries? A. Peripheral neuropathy B. Neurotmesis C. Entrapment syndrome D. Radiculopathy

D. Radiculopathy A radiculopathy is involvement of nerve roots due to skeletal changes and soft tissue injuries. Peripheral neuropathy is degenerative changes in peripheral nerves that produce sensory loss and motor weakness. Entrapment syndrome is when pressure on a nerve (from a bony prominence or restricted opening) leads to motor and sensory disturbances. Neurotmesisis a type of traumatic nerve injury in which the nerve is severed.

Which type of research study, considered level 1 evidence, is an experimental design in which participants are randomly assigned to either an experimental design in which participants are randomly assigned to either an experimental or control group to receive different interventions or a placebo? A. case-control study B. case report C. cohort study D. Randomized control trial

D. Randomized Control Trial A randomized control trial is an experimental study in which participants are randomly assigned to either an experimental or control group, receiving the variable intervention or placebo . This is considered level one evidence when utilizing research for evidence-based practice. A case report is a type of descriptive research in which only one patient is studied in depth, and a retrospective report is written about the case. A cohort study is a prospective (forward-in-time) study in which a cohort (group of participants) with a similar condition is followed for a defined amount of time and compared to a matched group without that condition. A case-control study is a retrospective (backward-in-time) study in which a group of individuals with a similar condition is compared with a group that does not have the condition to determine factors that could have played a role in the condition.

Pelvic floor weakness due to pregnancy and childbirth can lead to organ prolapse. Which is a herniation of the rectum into the vagina? A. Kegel B. Uterine prolapse C. Cystocele D. Rectocele

D. Rectocele Weakness and laxity in the pubococcygeal muscles from pregnancy, childbirth, and aging can lead to partial or total organ prolapse. A rectocele is the herniation of the rectum into the vagina. A cystocele is the herniation of the bladder into the vagina. A uterine prolapse is bulging of the uterus into the vagina. Kegel exercises are a strengthening exercise intervention for the pelvic floor muscles.

A physical therapist assistant is working in an assisted living facility, which is all on one single story. There is an emergency notification of a tornado warning. How should the therapist proceed? A. It is not the responsibility of the physical therapist assistant to help the residents B. Evacuate the residents from the building and into a building that has a basement level C. Discontinue treatment in the gym and return the resident to his or her room D. Remove the residents from their rooms and bring them into the hallways, closing the resident's room doors

D. Remove the residents from their rooms and bring them into the hallways, closing the resident's room doors In a tornado, i is important to remove the residents from windows and into an area free from furniture and areas prone to flaying debris and glass. By shutting the resident's door's, it will reduce risk of glass and flying debris. An interior hallway is safer than exterior rooms. The residents should not be brought outside if there is an active tornado warning, even if the intention is to move to another building. The residents should be kept in the interior of the building and not returned to their rooms, as it is easier to protect residents if they are all within the same area.

A researcher states that she expects there to be a statistically significant difference between the hamstring length of individuals who participated in a 12-week hamstring stretching program using static stretching versus those who participated in a yoga program. What kind of hypothesis is used here? A. Null hypothesis B. Non-directional hypothesis C. There is no hypothesis D. Research hypothesis

D. Research hypothesis A research hypothesis is a generalization that predicts an expected relationship between variables. Here, the researcher is predicting that those who participated in a static stretching program will have different outcomes in comparison to those who participated in a yoga program. A null hypothesis is one that states there is no relationship or difference between variables. A non-directional hypothesis means that a direction of change cannot be depicted

Which nerve root level is tested with the Achilles deep tendon reflex examination? A. C7-C8 B. L2-L4 C. L5-S3 D. S1-S2

D. S1-S2 The Achilles deep tendon reflex testing examines the function of the nerve root level S1-S2. A deep tendon reflex is tested with the muscle positioned at mid-range. The tendon is tapped with a reflex hammer. Deep tendon reflexes are also called stretch reflexes. L5-S3 is tested with the hamstrings deep tendon reflex. The L2-L4 nerve root level is tested with the quadriceps deep tendon reflex. The C7-C8 nerve roots are tested with the triceps deep tendon reflex.

A physical therapist assistant is educating family members on the positioning of a patient to prevent pressure ulcer formation. All of the following landmarks are susceptible to pressure ulcer formation when the the patient is positioned in side-lying EXCEPT: A. Lateral malleolus B. Lateral epicondyle of humerus C. Greater trochanter D. Scapulae

D. Scapulae The scapulae would have the most pressure placed on them when the patient is positioned supine. The greater trochanter, lateral epicondyle of the humerus, and lateral malleolus are most susceptible when the patient is positioned in side-lying. Bony prominences are susceptible to pressure ulcers. Risk factors for pressure ulcer formation include immobility and inactivity, sensory impairment, cognitive deficits, impaired circulation, poor nutrition, and incontinence.

A physical therapist assistant is educating family members on the positioning of a patient to prevent pressure ulcer formation. All if the following landmarks are susceptible to pressure ulcer formation when the patient is positioned in side-lying EXCEPT: A. Lateral malleolus B. Lateral epicondyle of humerus C. Greater trochanter D. Scapulae

D. Scapulae The scapulae would have the most pressure placed on them when the patient is positioned supine. The greater trochanter, lateral epicondyle of the humerus, and lateral malleolus are most susceptible when the patient is positioned in side-lying. Bony prominences are susceptible to pressure ulcers. Risk factors for pressure ulcer formation include immobility and inactivity, sensory impairment, cognition deficits, impaired circulation, poor nutrition, and incontinence.

The physical therapist assistant is teaching her patient with chronic obstructive pulmonary disease (COPD) strategies and positions to help to catch his breath. Which position is MOST effective for a patient with COPD to improve breathing? A. Prone B. Supine C. Seated leaning backward extension D. Seated leaning forward on the forearms

D. Seated leaning forward on the forearms The patient with COPD needs to utilize a position to decrease the amount of energy required to breathe. By sitting and leaning forward on the upper extremities, the patient can utilize the pectoralis and serratus anterior muscles as accessory muscles for improved ventilation. The other positions would increase the energy required to breathe and should not be used.

Which type of lymphatic disease occurs in some patients after radiations or malignancy? A. Primary lymphatic disease B. Acute lymphangitis C. Lymphadenopathy D. Secondary lymphatic disease

D. Secondary Lymphatic Disease Secondary lymphatic disease is acquired and can be due to trauma, surgery (radical mastectomy, femoral popliteal bypass), radiation, or disease. Lymphadenopathy is a condition of enlargement of nodes with or without tenderness. Acute lymphangitis is an acute bacterial infection spreading throughout the lymph system, usually streptococcal. Primary lymphatic disease is congenital.

Which type of lymphatic disease occurs in some patients after radiation or malignancy? A. Primary Lymphatic disease B. acute lymphangitis C. Lymphadenopathy D. Secondary lymphatic disease

D. Secondary lymphatic disease Secondary lymphatic disease is acquired and can be due to trauma, surgery (radical mastectomy, femoral popliteal bypass), radiation, or disease. Lyphadenopathy is a condition of enlargement of nodes with or without tenderness. Acute lymphangitis is an acute bacterial infection spreading throughout the lymph system (usually streptococcal). Primary lymphatic disease is congenital.

Which condition is considered a contraindication for electrical stimulation? A. Hypertension B. Malignant tumors C. Bleeding disorders D. Seizure disorder

D. Seizure disorder Contraindications for electrical stimulation include: pacemakers, unstable arrhythmias, suspected epilepsy or seizure disorders, transcerebrally or transthoracically in the presence of active bleeding or infection, superficial metal implants, pharyngeal or laryngeal muscles, and over the carotid sinus, thrombosis, eyes, thoracic region, phrenic nerve, urinary bladder stimulators, and abdomen or low back during pregnancy. Precautions include cardiac disease, impaired mentation, areas of impaired sensation, malignant tumors, open wounds, hypotension, hypertension, excessive edema, bleeding disorders, menstruating uterus, and pregnancy during labor and delivery.

In order to perform assisted quadriceps stretching in the pool, the patient is submerged to the umbilicus with a buoyant ankle weight around the ankle and performs which activity? A. Lunge forward and backward in the water B. Stand with the back to the wall and allow hip flexion C. Walk laterally along the pool wall D. Stand facing the wall and allow knee flexion

D. Stand facing the wall and allow knee flexion Standing and facing the wall while allowing the knee to flex (the bouyancy of the ankle weight will assist in flexing the knee, bringing the ankle up toward the buttocks) will promote stretching of the quadriceps. Standing with the back against the wall and allowing hip flexion will stretch the hamstrings. The lateral walking and forward/backward lunging will promote muscle strengthening of the lower extremities, not stretching.

Which document is intended to serve physical therapist assistants as a guide for ethical professional behavior and was put together by the American Physical Therapy Association? A. Code of Ethics B. Health Insurance Portability and Accountability Act (HIPAA) C. Board of Directors Guidelines for Documentation D. Standards of Ethical Conduct for the Physical Therapist Assistant

D. Standards of Ethical Conduct for the Physical Therapist Assistant The Standards of Ethical Conduct for the Physical Therapist Assistant was created by the American Physical Therapy Association and provides a framework by which physical therapist assistants should conduct themselves. It is comprised of ethical obligations and obligations of the profession. Standards include: Respect, Altruism, Sound Decisions, Supervision, Integrity in Relationships, Reporting, Exploitation, Colleague Impairment, Clinical Competence, Lifelong Learning, Organizational and Business Practices, Documentation Interventions, and Support-Health Needs.

A patient with a traumatic brain injury is displaying difficulty with her static standing balance. Which intervention is BEST indicated for this patient? A. Multi-directional lunging B. Reaching outside base of support for cones C. Standing over a tilt board and weight shifting D. Standing with eyes closed in tandem stance

D. Standing with eyes closed in tandem stance Standing with eyes closed with tandem stance will help to improve the patient's standing static balance. The other treatment options are all dynamic balance exercises that will help address dynamic balance.

What is being challenged when the therapist positions a patient in the Sharpened Rhomberg position? A. Dynamic standing balance B. Dynamic seated balance C. Static seated balance D. Static standing balance

D. Static standing balance The Sharpened Rhomberg position is when the patient is standing with heel-to-toe tandem stance. The patient folds his arms across the chest and stands statically for 30 seconds. Then, the patient is asked to repeat this stance for 30 seconds with his eyes closed. The test assesses and challenges static balance in standing.

When instructing a patient to ascend stairs with a step-to pattern after a right total knee replacement, which is the correct sequencing? A. Step up with the right leg followed by the cane and then meet the left leg on the same step. B. Step up with the right leg followed by the cane and then step the left leg onto the higher stair. C. Step up with the left leg followed by the cane and then step the right leg onto the higher stair. D. Step up with the left leg followed by the cane and then meet the right leg on the same step.

D. Step up with the left leg followed by the cane and then meet the right leg on the same step. When teaching stairs, it is important to teach proper sequencing. For ascending, the stronger or uninvolved leg steps up first followed by the assistive device and then the involved leg. For descending, the assistive device is first, then the involved leg steps down followed by the univolved leg. Step-to pattern sequencing means the individual meets both feet on the same stair. A reciprocal pattern is when the individual takes stairs step over step.

Which muscle or muscles are affected in an individual who has torticollis? A. Levator scapulae B. Upper trapezius C. Scalenes D. Sternocleidomastoid

D. Sternocleidomastoid Torticollis is marked by a spasm and/or tightness of the sternocleidomastoid. It has varied etiology but is common in infants. Dysfunction is observed as side-bending toward and rotationaway from the affected side. Physical therapy focuses on soft tissue mobilization and stretching of the neck musculature, as well as strengthening for head and neck control. The other options are incorrect, as the sternocleidomastoid is the affected muscle in torticollis.

Which handling technique is appropriate to use when working with a child with high tone or dystonia when the goal is to reduce motor output? A. Bouncing B. Brushing C. Vibration D. Stroking

D. Stroking Stroking is an appropriate handling technique when attempting to reduce motor output. Other appropriate techniques include rocking, firm touch, rhythmic movements, slow movements, warm water, and wrapping/swaddling. Bouncing, brushing, and vibration are all appropriate techniques to increase motor output when working with a child with low tone.

A patient presents with pain and tightness over the piriformis. The plan of care includes manual passive stretching. Which patient position is the BEST to stretch the piriformis? A. Prone with hip extended B. Supine with hip flexed to 70 degrees and knee extended C. Supine with hip abducted D. Supine with hip flexed to 70 degrees and adducted

D. Supine with hip flexed to 70 degrees and adducted Because of the anatomical position and attachments, the piriformis is placed in an elongated state and stretched when the patient's lower extremity is positioned in hip flexion and adducted. Placing the patient prone with hip extended will stretch the hip flexor muscle group. When the patient is placed supine with hip flexed and knee extended, the hamstrings will be stretched. When the leg is abducted, the adductor group is stretched.

The physical therapist assistant is working with a patient with bilateral lower extremity paralysis secondary to a spinal cord injury. The patient is wearing bilateral knee-ankle-foot orthoses and has not been able to perform reciprocal gait when in the parallel bars. The patient is to begin gait training with axillary crutches. Which is the most appropriate gait pattern to begin training? A. Swing-through B. 3 point C. 4 point D. Swing-to

D. Swing-to A swing-to gait pattern is appropriate for patients who have limited control for both lower extremities and impaired trunk stability. It is more stable and uses a slower cadence as compared to a swing-through. In a swing-to pattern, both crutches are advanced forward together. The weight is shifted to the hands for support and both legs are swung forward to meet the crutches. A swing-through pattern may be an appropriate progression if the patient masters a swing-to pattern. A 4-point gait is a reciprocal gait pattern, so not appropriate. A 3-point gait is indicated for a non-weight bearing lower extremity.

A PTA is aggressively working with a deconditioned individual to increase muscular strength. The patient complains of pain in the muscle in the next session, which is related to delayed onset muscle soreness (DOMS). Which statement is incorrect regarding DOMS? A. It peaks at 24-48 hours after exercise B. It can begin 12-24 hours after vigorous exercise C. Marked by muscle tenderness and stiffness D. Symptoms last up to 2-3 days

D. Symptoms last up to 2-3 days Muscle tenderness and stiffness can last up to 5-7 days. Soreness can begin 12-24 hours after vigorous exercise or muscular overexertion. It peaks at 24-48 hours after exercise. Usually, DOMS is greater after eccentric exercise.

Your patient has left leg weakness due to neurologic injury. As a PTA, how would you instruct your patient to descend stairs with a step to pattern? A. The assistive device and then the left leg steps down first followed by the right leg on the step below, never sharing the same step B. The assistive device and then the right leg steps down first followed by the left leg on the same step C. The assistive device and then the right leg steps down first followed by the left leg on the step below, never sharing the same step D. The assistive device and then the left leg steps down first followed by the right leg on the same step

D. The assistive device and then then left leg steps down first followed by the right leg on the same step. When teaching stairs, it is important to teach proper sequencing. For descending, the assistive device is first, then the weaker (involved) leg steps down followed by the uninvolved leg. For ascending, the stronger (uninvolved) leg steps up first followed by the assistive device and then the involved leg. Step-to pattern sequencing means the individual meets both feet on the same stair. A reciprocal pattern is when the individual takes stairs step over step.

In order to challenge postural stability to the greatest extent, which is TRUE? A. The higher the center of mass (COM) and larger the base of support (BOS), the greater the degree of postural challenge. B. The lower the center of mass (COM) and smaller the base of support (BOS), the greater the degree of postural challenge. C. The lower the center of mass (COM) and larger the base of support (BOS), the greater the degree of postural challenge. D. The higher the center of mass (COM) and smaller the base of support (BOS), the greater the degree of postural challenge.

D. The higher the center of mass (COM) and smaller the base of support (BOS), the greater the degree of postural challenge. If the center of mass is higher (such as standing) versus a low center of mass (in sitting), then postural stability is more challenged. Additionally, the smaller the base of support (such as single leg stance) versus a wider base of support (such as as standing with legs apart), postural stability is more challenged. Standing on one leg, for example is much more challenging to postural stability than sitting with legs firmly placed on the floor.

Physical therapist and physical therapist assistants are NOT mandated reporters of abuse in which population? A. Elders B. Children C. Disabled D. They are mandated reporters for all of these populations

D. They are mandated reporters for all of these populations PTs and PTAs are mandated reporters of neglect and/or abuse of children, elders, and the disabled in all 50 states. States have varying degrees of knowledge that trigger a report and all states of legislation that provides for immunity from prosecution arising out of the report. Victims of abuse are not likely to report abuse, as they are usually dependent on the abuser.

Which gait pattern is BEST indicated for an individual who is non-weight bearing in his right lower extremity due to an orthopedic injury? A. Two-point gait B. Four-point gait C. Swing-through gait D. Three-point gait

D. Three-point gait The therapist should instruct the describes patient in a three-point gait pattern in order to maintain non-weight bearing status. Both assistive devices and the involved leg advance together and weight is taken through the uninvolved limb. Then the uninvolved leg is advanced forward. This requires use of two assistive devices such as axillary crutches or a walker. Two-point gait and four-point gait are not indicated for non-weight bearing conditions. Swing-through gait is when both crutches advance forward together, weight is shifted onto the hands for support, and both legs are swung forward. This also creates weight bearing through the lower extremities.

Which arterial disease is marked by chronic inflammatory vascular occlusive disease of small arteries and also veins, primarily affecting young adult males who smoke? A. Diabetic angiopathy B. Arteriosclerosis obliterans C. Raynaud's disease D. Thromboangiitis obliterans

D. Thromboangiitis obliterans Thromboangiitis obliterans is a chronic, inflammatory vascular occlusive disease of small arteries and also veins. It commonly occurs in young adults who smoke. Patients have paresthesias, pain, cyanotic cold extremities, diminished temperature sensation, fatigue, and risk of ulceration. Arteriosclerosis obliterans is a chronic, occlusive aterial disease of medium- and large-sized vessels. It is the result of peripheral artherosclerosis and is associated with hypertension and hyper lipidemia. Diabetic angiopathy is an inappropriate elevation of blood glucose levels and accelerated atherosclerosis. Neuropathy is a major problem and patients may develop neurotrophic ulcers. Raynaud's disease is an episodic spasm of the small arteries and arterioles. It is marked by abnormal vasoconstrictor reflex exacerbated by exposure to cold or emotional stress. The tips of fingers develop pallor, cyanosis, numbness, and tingling.

25 degrees of flexion is the loose-packed position for which joint? A. Hip joint B. Talocrural joint C. Glenohumeral joint D. Tibiofemoral joint

D. Tibiofemoral joint The loose-packed position is the position in which the capsule and other soft tissues are maximally relaxed. The closed-packed position of a joint is the position in which the capsule and other soft tissues are maximally tensed. The loose-packed position for the tibiofemoral joint is 25 degrees of flexion of the knee. The loose-packed position for the hip joint is 30 degrees of flexion, 30 degrees of abduction, and slight external rotation. The loose-packed position for the talocrural joint is 10 degrees of plantar flexion of the ankle. The loose-packed position of the glenohumeral joint is 55-70 degrees of abduction, 30 degrees of horizontal abduction, and 0 degrees of rotation.

Which type of specialized wheelchair is BEST indicated for patients with extensor spasms and for pressure relief? A. Amputee chair B. One-arm drive C. Hemiplegic chair D. Tilt-in-space

D. Tilt-in-space A tilt-in-space wheelchair is a motorized wheelchair in which the entire seat and back may be tipped backward. It is indicated for patients with extensor spasms that may throw the patient out of the chair. It is also indicated for pressure relief. The one-arm drive wheelchair has drive mechanisms located on one wheel, usually with two outer rims. The patient propels the wheelchair by pushing on both rims with one hand. The hemiplegic chair is designed to be low to the ground and allows propulsion with the noninvolved upper extremity. An amputee chair is modified by placing the drive wheels posterior to the vertical back supports to increase posterior stability. It is intended for patients with bilateral lower extremity amputations.

The physical therapist assistant is working with a patient that has class III congestive heart failure. What is the maximal METS for physical activity for this patient? A. Up to 1.5 METS B. Up to 4.5 METS C. Up to 6.5 METS D. Up to 3 METS

D. Up to 3 METS Patients with congestive heart failure (CHF) demonstrate significant ventricular dysfunction, decreased cardiac output, and low functional. CHF can be classified as class I-IV. METS are metabolic equivalents of oxygen consumption. Class I: Mild CHF; no limitation in physical activity, up to 6.5 METS. Patients are comfortable at rest; ordinary activity does not cause undue fatigue, dyspnea, or angina pain. Class II: Mild CHF; slight limitation in physical activity, up to 4.5 METS. Patients are comfortable at rest, and ordinary activity results in fatigue, palpation, dyspnea, or angina pain. Class III: Moderate CHF; marked by limitation of physical activity, up to 3 METS. Patients are comfortable at rest, but less than ordinary activities cause fatigue, palpitation, dyspnea, or angina pain. Class IV: Severe CHF; patients are unable to carry out any physical activity (1.5 METS) without discomfort. Symptoms of ischemia, dyspnea, or angina pain are present even at rest and symptoms increse with exercise.

A patient with right-side hemiplegia is gait training with a hemi-walker. The patient is having difficulty advancing her right lower extremity forward. The BEST initial feedback that the physical therapist assistant should give is: A. Verbally cue to shift weight t the right B. Verbally cue the patient to shift weight posteriorly C. Physically lift the right lower extremity and place it forward D. Verbally cue to shift weight to the left

D. Verbally cue to shift weight to the left The patient must shift the weight toward the left in order to unweight the right lower extremity and allow for advancement of the right lower extremity. The most appropriate feedback is to cue the patient to weight shift left. Cuing the patient to shift weight to the right or posteriorly would hinder her ability to take a step forward. Physically lifting the right lower extremity forward would be indicated if the patient is not able to complete the task with a verbal cue.

A patient s/p CVA presents with symptoms on both right and left sides of the body. She has diplopia, dysphagia, and vertigo. She has ataxia, balance impairment, and nystagmus. Which CVA syndrome is likely? A. Middle cerebral artery syndrome B. Anterior cerebral artery syndrome C. Posterior cerebral artery syndrome D. Vertebrobasilar artery syndrome

D. Vertebrobasilar artery syndrome The vertebrobasilar artery supplies the medulla, pons, and cerebellum. Patients with vertebrobasilar artery syndrome have a wide variety of symptoms that can be on the same saide or opposite side of the infract. There is cranial nerve involvement presenting as diplopia, dysphagia, dysarthria, deafness, and vertigo. As well as ataxia. They can have Wallenberg's syndrome that presents with deficits in visual disturbances, balance, and gait or Locked-in syndrome where the patient is unable to speak or control any muscle beyond the eyes. The anterior cerebral artery supplies the medial part of the frontal and parietal lobes, basal ganglia and corpus callosum. Patients with anterior cerebral artery syndrome have contralateral sensory and motor loss with legs more affected than upper extremities. They have mental impairment, urinary incontinence, apraxia, slow delayed movement, and behavioral changes. The middle cerebral artery supplies the lateral cerebral hemispheres, including the frontal, temporal, and parietal lobes. Patients with middle cerebral artery syndrome have contralateral sensory and motor loss with face and upper extremities affected more than lower extremities. Additionally, they can have perceptual deficits, homonymous hemianopsia, Broca's and wernicke's aphasia, and global aphasia. The posterior cerebral artery suplies the occipital lobe, medial and inferior temporal lobe, thalamus, and midbrain. With posterior cerebral artery syndrome, the patient can experience contralateral sensory and motor loss, homonymous hemianopsia, visual agnosia, oculumotor nerve palsy, involuntary movement, Pusher syndrome, and thalamic pain syndrome.

In regards to ventilation, what is the term for the amount of air under volitional control, typically measured as forced expiratory vital capacity? A. Residual volume B. Total lung capacity C. Tidal volume D. Vital capacity

D. Vital capacity Vital capacity is the amount of air under volitional control (inspiratory reserve volume + tidal volume + expiratory reserve volume). Residual volume is the volume of gas that remains in the lungs after expiratory reserve volume has been exhaled. Tidal volume is the volume of gas inhaled or exhaled during a normal resting breath. Total lung capacity is the total amount of air contained within the thorax during a maximum inspiratory effort (inspiratory reserve volume + tidal volume + expiratory reserve volume + residual volume).

The physical therapist performs an evaluation for a 75-year-old woman who endured a stroke. She is classified as stage 3 on the Brunnstrom Stages of Motor Recovery scale. What motor presentation can the physical therapist assistant expect? A. Voluntary control is isolated joint movements, declining of spasticity and synergies B. Control and coordination near normal C. Flaccidity, no voluntary movement D. Voluntary movement possible but only in synergies, high spasticity

D. Voluntary movement possible but only in synergies, high spasticity The Brunnstrom's Stages of Motor recovery are as follows: Stage 1: Initial flaccidity, no voluntary movement Stage 2: Emergence of spasticity, hyperreflexia, synergies Stage 3: Voluntary movement possible but only in synergies, spasticity at it's peak Stage 4: Voluntary control in isolated joint movements emerging, corresponding decline of spasticity and synergies Stage 5: Increasing voluntary control Stage 6: Control and coordination near normal

Which of the following pieces of equipment LEAST supports an environment with a minimal lift guideline for employees? A. Slide board B. Sit-stand mechanical lift C. Hoyer lift D. Wheelchair

D. Wheelchair Many facilities now have minimal lift guidelines in place to minimize injuries to the worker, especially for the back, neck, or shoulders. Additionally, minimal lift guidelines are meant to minimize injury to the patient exceeds minimal assistance (requiring up to 25% assist) for transfers. Slide boards, Hoyer lifts, and sit-stand mechanical lifts are all examples of equipment that assists with lifting and transfers. A wheelchair does not assist offer any lifting assist.

The physics related to heat transmission for utilization of the superficial thermotherapy modalities include conduction, convection, and radiation. Which of the following is an example of heat transmission through convection? A. Infrared lamp B. Hot pack C. Paraffin D. Whirlpool

D. Whirlpool Convection occurs when heat is transferred by movement of air or fluid from a warmer area to a cooler area or moving past a cooler body part. Examples of convective modalities include whirlpool, Hubbard tank, and fluidotherapy. Hot packs and paraffin are examples of conduction heat transfer. Infrared lamp is an example of radiation heat transfer.

Which environmental situation is NOT appropriate for intervention when working to reduce tone in a child with spasticity or dystonia? A. Relaxing music B. Decluttered room C. Quiet voice D.Loud and fast-paced music

D.Loud and fast-paced music For a child with high tone, the goal is to decrease motor output. Environmental stimuli should provide should provide consistent sensory input, relaxing music, and quiet voice. A calm room free of clutter is appropriate. Increasing environmental or sensory stimulation may further increase tone. This is appropriate for a child with low tone.

Which transfer technique is MOST appropriate for an individual with a complete C5 spinal cord injury? A. Dependent lift transfer B. Assisted stand pivot transfer C. Dependent stand pivot transfer D. Assisted sliding board transfer

This transfer is used for patients with good sitting balance who can lift most but not all of the weight of the buttocks such as this individual with a complete C5 spinal cord injury. The sliding board is placed well under the patient's buttocks. The patient performs the transfer by doing a series of push-ups and lifts across the board. The therapist assists in the lifts. Patients with complete C6 level spinal cord injury can be independent with slide board transfer on even levels.


Ensembles d'études connexes

Chapter 15 - Water and Aqueous Systems

View Set

GCA - Geometry A Introduction to Transformations

View Set

2016 Mock ACLAM Practice Written/Practical Examination

View Set

Unit 5: Quadratic Equations and Functions

View Set

World History Unit 10: Major Changes After WWII

View Set